Select Transactions Flashcards

1
Q

A company should recognize goodwill in its balance sheet at which of the following points?

A
Costs have been incurred in the development of goodwill.
B
Goodwill has been created in the purchase of a business.
C
The company expects a future benefit from the creation of goodwill.
D
The fair market value of the company’s assets exceeds the book value of the company’s assets.

A

B

Explanation:
Goodwill arises when one entity purchases another entity, and is recognized as the excess of the acquisition-date fair value of the purchase price over the recognized amounts of assets, liabilities, and noncontrolling interests. Internally generated goodwill is not recognized or recorded.

How well did you know this?
1
Not at all
2
3
4
5
Perfectly
2
Q
DeeCee Co. adjusted its historical cost income statement by applying specific price indexes to its depreciation expense and cost of goods sold. DeeCee's adjusted income statement is prepared according to
A
Fair value accounting.
B
General purchasing power accounting.
C
Current cost accounting.
D
Current cost/general purchasing power accounting.
A

C-Current cost accounting

Explanation:
DeeCee adjusts the depreciation and cost of goods sold reported in the historical cost income statement by applying specific price indexes to these amounts. Therefore, DeeCee’s adjusted income statement is prepared using current cost accounting. The income statement is not prepared using fair value accounting because only depreciation expense and cost of goods sold are restated by applying specific price indexes. The income statement is not prepared using general purchasing power accounting because DeeCee’s historical costs are not remeasured into units of a currency with the same general purchasing power. The income statement is not prepared using current cost/general purchasing power accounting because amounts are not remeasured into units of a currency with the same general purchasing power.

How well did you know this?
1
Not at all
2
3
4
5
Perfectly
3
Q
On January 2, year 1, Union Co. purchased a machine for $264,000 and depreciated it by the straight-line method using an estimated useful life of eight years with no salvage value. On January 2, year 4, Union determined that the machine had a useful life of six years from the date of acquisition and will have a salvage value of $24,000. An accounting change was made in year 4 to reflect the additional data. The accumulated depreciation for this machine should have a balance at December 31, year 4, of
A
$176,000.
B
$160,000.
C
$154,000.
D
$146,000.
A

D- 146k remember the new salvage value

Explanation:
The change in the estimated useful life of the machine is a change in accounting estimate. The effect of the change in accounting estimate be accounted for prospectively in the period of change and future periods, because both are affected.

Accumulated depreciation, 1/1, year 4 [($264,000 - $0) / 8] × 3 years $ 99,000
Cost of machine $264,000
Accumulated depreciation, 1/1, year 4 (99,000)
Carrying amount of machine,1/1, year 4 $165,000
Less: Estimated salvage value (24,000)
Depreciable base of machine,1/1, year 4 $141,000
Divide by: Estimated remaining useful life (6 - 3) / 3
Depreciation for year 4 47,000
Accumulated depreciation, 12/31, year 4 $146,000

How well did you know this?
1
Not at all
2
3
4
5
Perfectly
4
Q

Retailer enters into a lease of warehouse space. The lease is for a nine-month noncancelable term, can be extended for four months, and does not include a purchase option. On the lease commencement date, Retailer concludes that it is not reasonably certain that it will renew the lease beyond the nine-month noncancelable period because (1) the noncancelable period coincides with the period in which Retailer expects to need the additional storage and (2) the monthly lease payments during the optional extension period are expected to be at market rates. How should the lessee recognize the lease?

A
As an operating lease with lease payments as expense on straight line basis.
B
As a finance lease with present value of lease payments as ROU asset.
C
As an operating lease with lease payments as expense on ROU asset.
D
As a sales type lease with lease payment as expense on straight line basis.

A

A- Operating lease

Explanation:
In this scenario, because Retailer only needs the warehouse space to support its operations for a nine-month period and the pricing for the optional period is expected to be consistent with the expected market rates, it would be reasonable to conclude that the lease term is limited to the nine-month cancelable period. Therefore, the lease term is 12 months or less and Retailer applies the short-term lease exemption in accounting for the lease (i.e., it recognizes lease payments as an expense on a straight-line basis over the lease term and does not recognize a lease liability or ROU asset on its balance sheet).

Option (B), (C) and (D) are incorrect as per above explanation.

How well did you know this?
1
Not at all
2
3
4
5
Perfectly
5
Q

At the commencement date of an operating lease, a lessor should

A
Subtract initial direct costs
B
Defer initial direct costs
C
Recognize initial direct costs
D
None of the above
A

Explanation:
The correct answer is (B).

At the commencement date, a lessor should defer initial direct costs.

Initial direct costs associated with the lease are deferred and amortized over the term of the lease as the lease income is recognized.

After the commencement date, a lessor should recognize all of the following:

The lease payments as income in profit or loss over the lease term on a straight-line basis.
Variable lease payments as income in profit or loss in the period in which the changes in facts and circumstances on which the variable lease payments are based occur
Initial direct costs as an expense during the lease period on the same basis as income from the lease.

How well did you know this?
1
Not at all
2
3
4
5
Perfectly
6
Q

Which of the following statements is false concerning subsequent events?

A
An entity shall not recognize events occurring between the time the financial statements were issued or were available to be issued and the time the financial statements were reissued unless the adjustment is required by GAAP or regulatory requirements.
B
Some nonrecognized subsequent events may be of such a nature that they must be disclosed to keep the financial statements from being misleading.
C
An entity shall not recognize subsequent events that provide evidence about conditions that did not exist at the date of the balance sheet but arose after the balance sheet date but before financial statements are issued or are available to be issued.
D
An entity is not permitted to disclose any estimates inherent in the process of preparing financial statements when disclosing the effects of subsequent events that provide additional evidence about conditions existing at the date of the balance sheet.

A

Explanation:
The correct answer is (D).

An entity shall recognize in the financial statements the effects of all subsequent events that provide additional evidence about conditions that existed at the date of the balance sheet, including the estimates inherent in the process of preparing financial
statements.

How well did you know this?
1
Not at all
2
3
4
5
Perfectly
7
Q

On October 1 of the current year, Mild Co., a U.S. company, purchased machinery from Grund, a German company, with payment due on April 1 of next year. If Mild’s current-year operating income included no foreign exchange transaction gain or loss, then the transaction could have
A
Resulted in a Foreign Currency translation gain to be reported in company’s income statement.
B
Been denominated in U.S. dollars.
C
Caused a foreign currency gain to be reported as a contra account against machinery.
D
Caused a foreign currency translation gain to be reported in other comprehensive income.

A

Explanation:
The correct answer is (B).

Foreign currency transactions are transactions denominated in a currency other than the entity’s functional currency. Hence, no foreign currency transaction gain or loss would occur if the purchase of the machinery by the U.S. company is denominated in U.S. dollars.

How well did you know this?
1
Not at all
2
3
4
5
Perfectly
8
Q

Which of the following statements correctly describes the proper accounting for nonmonetary exchanges that are deemed to have commercial substance?
A
It defers any gains and losses.
B
It defers losses to the extent of any gains.
C
It recognizes gains and losses immediately.
D
It defers gains and recognizes losses immediately.

A

C

Explanation:
In an exchange with commercial substance, the transaction is accounted for at the fair value of the asset received or the asset given up, whichever is more clearly evident, and a gain or loss is recognized on the exchange.

How well did you know this?
1
Not at all
2
3
4
5
Perfectly
9
Q

When valuing certain financial instruments, a company that has elected the fair value measurement option must apply the accounting measurement based on which of the following criteria?

A
A portion of an asset or liability
B
Instrument-by-instrument basis
C
Type-by-type basis
D
At the entity level
A

Explanation:
The correct answer is (B).

ASC 825-10-25-2 states the fair value measurement option is irrevocable, must be applied on an instrument-by-instrument basis, and must be applied to the entire instrument as a whole.

(A) is incorrect because the entire asset or liability must be measured at fair value.

(C) is incorrect because fair value must be measured on an instrument-by-instrument basis for each type that is to be valued under fair value measurement.

(D) is incorrect because fair value must be measured for each instrument and not at the entity level.

How well did you know this?
1
Not at all
2
3
4
5
Perfectly
10
Q
On January 1, year 1, a company capitalized $100,000 of costs for software that is to be sold. The company amortizes the software costs on a straight-line basis over five years. The carrying value of the software costs on January 1, year 3, was $60,000. As of December 31, year 3, the estimated future gross revenue to be generated from the sale of the software is $23,000, and the estimated future cost of disposing of the software is $8,000. What amount should the company expense related to the software costs for the year ended December 31, year 3?
A
$18,400
B
$20,000
C
$37,000
D
$45,000
A

D- 45k

Explanation:
Software production costs are capitalized and reported at the lower of unamortized cost or net realizable value (NRV) once technological feasibility has been met. The unamortized cost is $60,000 and the NRV is $15,000 ($23,000 − $8,000); therefore, the software should be written down by $45,000 (i.e., expensed) to the NRV of $15,000.

How well did you know this?
1
Not at all
2
3
4
5
Perfectly
11
Q

On December 12, year 1, Imp Co. entered into a forward exchange contract to purchase 100,000 euros in 90 days. The relevant exchange rates are as follows:

Spot rate Forward rate (for 3/12, yr 2)

December 12, year 1 $1.86 $1.80
December 31, year 1 $1.96 $1.83
Imp entered into the forward contract to hedge a commitment to purchase equipment being manufactured to Imp’s specifications. At December 31, year 1, what amount of foreign currency transaction gain should Imp include in income from this forward contract?

A
$0
B
$ 10,000
C
$ 5,000
D
$3,000
A

Explanation:
The correct answer is (D).

A forward exchange contract is an agreement to exchange different currencies at a specified future date and at a specified rate (the forward rate). A forward contract is a foreign currency transaction. The accounting for a gain or loss on a foreign currency transaction that is intended to hedge an identifiable foreign currency commitment (for example, an agreement to purchase or sell equipment) is considered a foreign currency hedge and works as a fair value hedge.

Gains and losses on this qualifying fair value hedge shall be recognized currently in earnings. The gain or loss realized on this forward exchange contract is computed by multiplying the foreign currency amount of the contract by the difference between the forward rate at the balance sheet date and the forward rate at the inception of the contract (or the forward rate last used to measure a gain or loss on that contract for an earlier period).

Foreign currency units to be purchased 100,000
Times: Excess of forward rate at the balance sheet date over the forward rate at the inception of the contract ($1.83 - $1.80) × $0.03
Foreign currency transaction gain $3,000
Journal Entries

12/12/Year 1:

J/E @90-day forward rate for the forward exchange contract (entered to hedge against a possible increase in the value of Euros to be paid when the invoice is due for the purchase commitment)

Dr. Euro receivable $180,000

Cr. US$ payable $180,000

12/31/20X0:

J/E for gain or loss on a forward exchange contract at the forward rate

Dr. Euro receivable $3,000

Cr. Gain on Forward Exchange Contract $3,000

Note: Foreign currency transactions can have operating transactions in the foreign currency such as buying inventory or a purchase commitment for equipment. Fluctuations in foreign exchanges (spot rate) will result in a gain or loss in inventory or receivable/payable. Such fluctuations always use the spot rate.

To hedge such fluctuations, a forward contract is used, which is an agreement to exchange two different currencies at a future date, at a specific rate.

Gains or losses in forward contracts use the forward rate.

In this particular question when spot rates increase from $1.86 to $1.96 my liability for purchase commitment will increase by $0.1 resulting in $10,000 loss. However, the forward exchange contract will have a gain of $1.83 - $1.80. i.e. $0.03 resulting in a $3,000 gain on the forward contract.

How well did you know this?
1
Not at all
2
3
4
5
Perfectly
12
Q

Certain balance sheet accounts of a foreign subsidiary of Rowan Inc., at December 31, have been translated into U.S. dollars as follows:
Translated at
Current Rates Historical Rates
Note receivable, long-term $240,000 $200,000
Prepaid rent 85,000 80,000
Patent 150,000 170,000
$475,000 $450,000The subsidiary’s functional currency is the currency of the country in which it is located. What total amount should be included in Rowan’s December 31 consolidated balance sheet for the above accounts?
A
$450,000
B
$455,000
C
$475,000
D
$495,000

A

Explanation:
Since the subsidiary’s functional currency is the currency of the country in which it is located, all of its assets are translated at the current rate (i.e., the exchange rate in effect at the balance sheet date).

How well did you know this?
1
Not at all
2
3
4
5
Perfectly
13
Q

Farm Co. leased equipment to Union Co. on July 1 of the current year and properly recorded the sales-type lease at $135,000, the present value of the lease payments discounted at 10%. The first of eight annual lease payments of $20,000 due at the beginning of each year was received and recorded on July 3 of this year. Farm had purchased the equipment for $110,000. What amount of interest revenue from the lease should Farm report in its current year income statement?

A
$0
B
$5,500
C
$5,750
D
$6,750
A

Explanation:
Interest revenue is calculated by adjusting the initial investment in the lease by the payment received on July 3, and then applying the interest rate and prorating for the last 6 months of the year.

Net investment in lease (PV), 7/1 $135,000
Less: Payment received, 7/3 (20,000)
Net investment in lease after 7/3 payment 115,000
Interest rate × 10%
Whole year interest revenue 11,500
July through December, 1/2 year × 0.5
Interest revenue $ 5,750

How well did you know this?
1
Not at all
2
3
4
5
Perfectly
14
Q
Which of the following is the characteristic of a perfect hedge?
A
No possibility of future gain or loss
B
No possibility of future gain only
C
No possibility of future loss only
D
The possibility of future gain and no future loss
A

Explanation:
Hedging is a risk management strategy to protect against the possibility of loss, such as from price fluctuations. Generally, the strategy involves counterbalancing transactions in which a loss on one financial instrument or cash flow stream would be offset by a gain on the related derivative. A perfect hedge would result in no possibility of future gain or loss.

How well did you know this?
1
Not at all
2
3
4
5
Perfectly
15
Q

In a lease that is recorded as a sales-type lease by the lessor, interest revenue

A
Should be recognized in full as revenue at the lease’s inception.
B
Should be recognized over the period of the lease using the straight-line method.
C
Should be recognized over the period of the lease using the effective interest method.
D
Does not arise

A

Explanation:
In a lease recorded as the sales-type lease by the lessor, the lessor recognizes two types of revenue: the gain or loss on the sale (lease) of the asset and the interest income from the collection of the lease payment. The unearned interest revenue is amortized over the life of the lease using the effective interest rate method on the receipt of the periodic lease payments.

Options (A), (B) and (D) are incorrect because interest revenue arises and that cannot be recognized immediately, it has to be earned over the period of the lease by effective interest method and not the straight-line method.

How well did you know this?
1
Not at all
2
3
4
5
Perfectly
16
Q

Which of the following items requires a prior period adjustment to retained earnings?
A
Purchases of inventory this year were overstated by $5 million.
B
Available-for-sale securities were improperly valued last year by $20 million.
C
Revenue of $5 million that should have been deferred was recorded in the previous year as earned.
D
The prior year’s foreign currency translation gain of $2 million was never recorded.

A

Explanation:
Errors in financial statements result from mathematical mistakes, mistakes in the application of accounting principles, or the oversight or misuse of facts that existed at the time the financial statements were prepared. Errors that occur in one accounting period and are discovered in a subsequent accounting period are more involved: the cumulative effect of each error on periods prior to the period of discovery is calculated and recorded as a direct adjustment to the beginning balance of retained earnings. Errors that occur and are discovered in the same accounting period may be corrected by reversing the incorrect entry and recording the correct one or by directly correcting the account balances with a single entry. Foreign currency gains and losses and available for sale securities are reported in other comprehensive income.

How well did you know this?
1
Not at all
2
3
4
5
Perfectly
17
Q

A company has an operating lease for its office space. The lease term is 120 months and requires monthly rent of $15,000. As an incentive for the company to enter into the lease, the lessor granted the first eight months’ rent at no cost. What amount of monthly rent expense should be recognized over the life of the lease?

A
$14,000
B
$14,062
C
$15,000
D
$16,072
A

Explanation:
The correct answer is (A).

In an operating lease, when there are uneven lease payments or there is a free rent period, the total rent expense payable for the entire lease term is divided evenly over each period in line with the matching principle. As the first eight months rent was free and the remaining 112 months the rent was $15,000, the total monthly rent expense to be recognized over the life of the lease is $14,000 (i.e. $15,000 x 112) / 120.

(B) is incorrect because this is a random number.

(C) is incorrect because this is the monthly rent to be paid as per the contract for 112 months which needs to be apportioned over the entire rental period of 120 months.

(D) is incorrect because here the calculation as per option (a) is reversed (i.e. $16,072 = $15,000 * 120 / 112).

How well did you know this?
1
Not at all
2
3
4
5
Perfectly
18
Q

Jay’s lease payments are made at the end of each period. Jay’s liability for a finance lease would be reduced periodically by the:
A
lease payment less the portion of the minimum lease payment allocable to interest.
B
lease payment plus the amortization of the related asset.
C
lease payment less the amortization of the related asset.
D
lease payment.

A

Explanation:
In a finance lease, the lease liability is amortized by effective interest rate method. Each lease payment is first applied to the interest on the basis of the interest rate x carrying value of the lease obligation. Any remaining portion of the lease payment is applied towards reduction of the lease liability.

Options (B), (C) and (D) are incorrect because finance lease is reduced by minimum lease payment less interest and not amortization of leased asset or the entire minimum lease payment.

How well did you know this?
1
Not at all
2
3
4
5
Perfectly
19
Q

Appointment of the IFRS Foundation Trustees is approved by which of the following?

A
The IFRS Advisory Council
B
The IFRS Interpretations Committee
C
The IFRS Foundation Monitoring Board
D
The Accounting Standards Advisory Forum
A

Explanation:
Appointment of the IFRS Trustees is approved by the IFRS Foundation Monitoring Board. The IFRS Advisory Council is the formal advisory body to the IASB and the Trustees. The IFRS Interpretations Committee reviews current IFRS accounting issues that have arisen and provides authoritative guidance (IFRICs) on those issues. It has 14 voting members appointed by the Trustees of the IFRS Foundation for their technical ability. The Accounting Standards Advisory Forum is an advisory group to the IASB providing technical advice and feedback. Its members consist of national accounting standard-setters and regional bodies with an interest in financial reporting and are selected by the Trustees.

How well did you know this?
1
Not at all
2
3
4
5
Perfectly
20
Q

Combined statements may be used to present the results of operations of

Companies under common management	Commonly controlled companies
A	Yes	No
B	No	Yes
C	Yes	Yes
D	No	No
A

Explanation:
The correct answer is Option (C).

Combined financial statements are used if consolidated financial statements are not suitable for the same purpose. This is done when a non-consolidated subsidiary or group of companies owned by a common shareholder is present.

Options (A), (B) and (D) are incorrect as per above explanation.

How well did you know this?
1
Not at all
2
3
4
5
Perfectly
21
Q

PQR Ltd. enters into a contract with a customer for the sale of a tangible asset on January 1, 20X7, for $1 million. The contract also gives the entity the right to repurchase the asset for $1.1 million on or before December 31, 20X7. The expected market value for the asset is $1.05 million. This option is:

A
A forward option accounted for as a lease.
B
A call option accounted for as a financing arrangement.
C
A put option accounted for as a sale with a right to return.
D
A forward option accounted for as a financing arrangement.

A

Explanation:
As PQR has the right to repurchase the asset this is a call option and as the repurchase price of 1.1 million is greater than the original price of $ 1 million, this is to be accounted for as a financing arrangement.

How well did you know this?
1
Not at all
2
3
4
5
Perfectly
22
Q

Which of the following statements describes the proper accounting for losses when nonmonetary assets are exchanged for other nonmonetary assets?
A
A loss is recognized immediately, because assets received should not be valued at more than their cash equivalent price.
B
A loss is deferred so that the asset received in the exchange is properly valued.
C
A loss, if any, which is unrelated to the determination of the amount of the asset received should be recorded.
D
A loss can occur only when assets are sold or disposed of in a monetary transaction.

A

Explanation:
In general, accounting for nonmonetary transactions (those involving nonmonetary assets or liabilities) should be based on the fair values of the assets involved. The acquisition is recorded at the fair value of the asset surrendered or the fair value of the asset received, whichever is more clearly determinable, and gains or losses should be recognized. The rationale associated with immediate gain/loss, recording the transactions at fair value, is that the exchange represents the culmination of the earnings process associated with the assets surrendered. The best statement describing the proper accounting for losses when nonmonetary assets are exchanged is that a loss is recognized immediately, because assets received should not be valued at more than their cash equivalent price.

How well did you know this?
1
Not at all
2
3
4
5
Perfectly
23
Q
Which of the following is (are) examples of contingent liabilities?
A
Obligations related to product warranties
B
Pending or threatened litigation
C
Both A. and B.
D
Neither A. nor B.
A

Explanation:
Examples of contingent liabilities include obligations related to product warranties and pending or threatened litigation. Since answers A. and B. are correct, answer C., both A. and B., is the best choice.

How well did you know this?
1
Not at all
2
3
4
5
Perfectly
24
Q
Conn Co. reported a retained earnings balance of $400,000 at December 31 of the previous year. In August of the current year, Conn determined that insurance premiums of $60,000 for the three-year period beginning January 1 of the previous year had been paid and fully expensed in that year. Conn has a 30% income tax rate. What amount should Conn report as adjusted beginning retained earnings in its current year statement of retained earnings?
A
$420,000
B
$428,000
C
$440,000
D
$442,000
A

B- 428k

Explanation:
The recognition of the effect of fully expensing the premiums for the three-year insurance policy represents the correction of an error of a prior period. The correction of the error should be reported as prior-period adjustment by restating the prior-period financial statements. The correction results in $40,000 (i.e., $60,000 × 2/3) less insurance expense recognized in the previous year. This increases the balance of retained earnings as corrected by the retrospective application. However, the balance of retained earnings cannot be increased by the full $40,000, because the reduction in insurance expense would have increased the amount of income tax expense previously recognized by $12,000 (i.e., $40,000 × 30%). Thus, the balance of retained earnings corrected by the retrospective application is increased by $28,000 (i.e., $40,000 - $12,000). Therefore, the amount to be reported as corrected by the retrospective application is $428,000 (i.e., $400,000 + $28,000).

How well did you know this?
1
Not at all
2
3
4
5
Perfectly
25
Q

On November 2, year 1, Platt Co. entered into a 90-day futures contract to purchase 50,000 Swiss francs when the contract quote was $0.70. The purchase was for speculation in price movement. The following exchange rates existed during the contract period:

 	30-day futures	Spot rate
November 2, year 1	$0.62	$0.63
December 31, year 1	0.65	0.64
January 30, year 2	0.65	0.68
What amount should Platt report as foreign currency exchange loss in its income statement for the year ended December 31, year 1?
A
$2,500
B
$3,000
C
$3,500
D
$4,000
A

A- 2.5k

Explanation:
A gain or loss on a speculative futures contract should be computed by multiplying the foreign cur­rency amount of the futures contract by the difference between the future rate available for the remaining matur­ity of the contract and the contract future rate (or the forward rate last used to measure a gain or loss on that contract for an earlier period). No separate accounting recognition is given to the discount or premium. The spot rate is the exchange rate used for immediate delivery of currencies exchanged.

Foreign currency units to be purchased

50,000

Times: Excess of futures rate available for the remaining maturity of the contract and the contracted futures rate ($0.70 – $0.65)

× $0.05

Loss on futures contract

$ 2,500

How well did you know this?
1
Not at all
2
3
4
5
Perfectly
26
Q

On January 1 of the current year, Babson, Inc., leased two automobiles for executive use. The lease requires Babson to make five annual payments of $13,000 beginning on this same date. At the end of the lease term, December 31 in five years, Babson guarantees the residual value of the automobiles will total $10,000 and the expected Residual Value is $9,000. The lease qualifies as a Finance Lease. The interest rate implicit in the lease is 9%. Present value factors for the 9% rate implicit in the lease are as follows:

For an annuity due with 5 payments 4.240
For an ordinary annuity with 5 payments 3.890
Present value of $1 for 5 periods 0.650
Babson’s recorded Finance Lease liability immediately after the first required payment should be

A
$42,770
B
$44,070
C
$35,620
D
$31,070
A

A- 42,770

Explanation:
At commencement, the initial measurement of the lease liability (regardless of lease classification) is calculated as the present value of the lease payments not yet paid by using the lease term and discount rate determined at lease commencement.

Lease Liability includes the present value of rental payments called for by the lease over the lease term (five payments of $13,000) and the amount that it is probable that the lessee will owe under the residual value guarantee at the end of the lease term ($10,000 - $9,000).

Since the annual rental payment is payable at the beginning of each lease year, the present value factor for an annuity due is used.

Present value of minimum rental payments called for over the lease term ($13,000 × 4.240) $55,120
Present value of the guarantee by the lessee of the residual value at expiration of lease ($1,000 × 0.650) 650
Present value of minimum lease payments at the inception of the lease, 1/1 $55,770
Immediate minimum rental payment made 1/1 13,000
Finance lease liability after first required payment, 1/1 $42,770

How well did you know this?
1
Not at all
2
3
4
5
Perfectly
27
Q

On January 2, Year 4, Raft Corp. discovered that it had incorrectly expensed a $210,000 machine pur­chased on January 2, Year 1. Raft estimated the machine’s original useful life to be 10 years and its salvage value at $10,000. Raft uses the straight-line method of depreciation and is subject to a 30% tax rate. In its December 31, Year 4, financial statements, what amount should Raft report as a prior period adjustment?

A
$102,900
B
$105,000
C
$165,900
D
$168,000
A

B- 105k

Explanation:
The company needs to reverse the impact of expensing the machine and account for depreciation for the machine. Depreciation expense is $20,000 {($210,000 - $10,000)/10 years} per year and $60,000 for 3 years. So, depreciation expense has been overstated by $150,000. Prior period adjustment would be $105,000 net of 30% taxes [$150,000 - ($150,000 x 30%)].

Options (A), (C) and (D) are incorrect as per the above explanation.

How well did you know this?
1
Not at all
2
3
4
5
Perfectly
28
Q

On January 2, 2018, Marx Co. as lessee signed a five-year non-cancellable equipment lease with annual payments of $200,000 beginning December 31, 2018. Marx treated this transaction as a finance lease. The five lease payments have a present value of $758,000 on January 2, 2018, based on an interest rate of 10%. What amount should Marx report as interest expense for the year ended December 31, 2018?

A
$0
B
$48,400
C
$55,800
D
$75,800
A

Explanation:
The correct answer is (D).

Interest is calculated on the lease liability carrying value. The first annual lease payment is made on December 31, 2018, one year would have elapsed since the inception of the lease. Interest for the one year would be calculated on the lease liability recorded at the beginning of the year at the present value of $758,000.

Interest = 10% of $758,000 = $75,800.

Options (A) is incorrect because interest would have accrued since one year would have elapsed from the date of inception of the lease to the date of the first annual payment.

Option (B) is incorrect because of inaccurate calculation.

Option (C) is incorrect because interest must be calculated on the original lease liability balance as the payment was made at the end of the year rather than after the first installment.

How well did you know this?
1
Not at all
2
3
4
5
Perfectly
29
Q

A company incurred the following costs to complete a business combination in the current year:

Issuing debt securities	$30,000
Registering debt securities	$25,000
Legal fees	$10,000
Due diligence costs	$1,000
What amount should be reported as Bond Issue Costs?
A
$1,000
B
$55,000
C
$36,000
D
$66,000
A

Explanation:
The correct answer is (B).

The costs to register and issue debt or equity securities shall be reported as Bond Issue Costs totaling to $30,000 + $25,000 = $55,000.

Acquisition-related costs are costs the acquirer incurs to effect a business combination.

Those costs include finder’s fees; advisory, legal, accounting, valuation, and other professional or consulting fees; general administrative costs; and costs of registering and issuing debt and equity securities.

The acquirer should account for acquisition-related costs as expenses in the periods in which the costs are incurred and the services are received.

How well did you know this?
1
Not at all
2
3
4
5
Perfectly
30
Q
What is the underlying concept governing the generally accepted accounting principles pertaining to recording gain contingencies?
A
Conservatism
B
Relevance
C
Consistency
D
Reliability
A

A- Conservatism

Explanation:
Gain contingencies should be disclosed in the financial statements in a footnote rather than being reflected in income because doing so could result in recognizing income prior to its realization. This treatment reflects the principle of conservatism which means that accountants who are selecting between two possible alternatives should choose the accounting alternative which is least likely to overstate assets and income.

How well did you know this?
1
Not at all
2
3
4
5
Perfectly
31
Q
Green Co. was preparing its year-end financial statements. Green had a pending lawsuit against a competitor for $5,000,000 in damages. Green’s attorneys indicate that obtaining a favorable judgment was probable and the amount of damages is reasonably estimated. Green incurred $100,000 in legal fees. The income tax rate was 30%. What amount, if any, should Green recognize as a contingency gain in its financial statements?
A
$0
B
$3,430,000
C
$3,500,000
D
$4,900,000
A

A-$0

Explanation:
Contingencies arise from events or circumstances occurring before the balance sheet date, the resolution of which is contingent on a future event or circumstance. Where the likelihood of a loss is considered probable and the loss can be reasonably estimated, the estimated loss should be charged to income and the nature of the contingency should be disclosed. Gain contingencies should be disclosed but not recognized as income.

How well did you know this?
1
Not at all
2
3
4
5
Perfectly
32
Q
Management can estimate the amount of loss that will occur if a foreign government expropriates some company assets. If expropriation is reasonably possible, a loss contingency should be
A
Disclosed but not accrued as a liability
B
Disclosed and accrued as a liability
C
Accrued as a liability but not disclosed
D
Neither accrued as a liability nor disclosed
A

A

Explanation:
Since the contingent loss from the expropriation of assets is judged to be reasonably possible, it should be disclosed in the footnotes, but not accrued. A contingent loss is accrued only in situations where the loss is probable and estimable.

How well did you know this?
1
Not at all
2
3
4
5
Perfectly
33
Q

An entity leases a highly specialized underwater vehicle with patented technology to another entity. The asset took three years to produce. The lessee uses the asset to search the deep ocean floor for buried treasure in a remote area of the Arctic Ocean. The cost of transporting the asset to the search site was approximately half the cost of the asset itself, and it is not expected that any other entity is going to want to use that asset in that specific location. The lessor should classify this transaction as:

A
Operating Lease
B
Sales-Type Lease
C
Direct Financing Lease
D
None of the above
A

B- Sales type

Explanation:
A Finance Lease must meet one of the following criteria:

Present Value equals or exceeds substantially all (90%) of the Fair Value
Option to Purchase (exercise is reasonably certain)
Economic Life - Major part (75%) of asset’s economic life is used
Transfer of Ownership at lease termination
Specialized Nature - No alternative use to the lessor at lease termination
Note: the implementation guidance for ASC 842 uses the 75/90 thresholds, even though the standard is principles-based.

The lessor does not believe that any other entities would be interested in a similar use (searching the Arctic Ocean for buried treasure), and the asset is designed for that particular environment and no other. As a result, the lessor in this example would meet the criterion for classifying the lease as a sales-type lease.

Option (A), (C) and (D) are incorrect as per above explanation.

How well did you know this?
1
Not at all
2
3
4
5
Perfectly
34
Q

Which of the following costs should not be included in research and development?

A
Facility costs.
B
Personnel costs.
C
Administrative costs.
D
Indirect costs
A

Explanation:
The correct answer is (C).

Following costs are included in research and development.

Materials, equipment, and facilities
Personnel Cost: Salaries, wages, and other related costs of personnel
Intangibles purchased from other
Contract services
Indirect costs.
General and administrative costs that are not clearly related to research and development activities shall not be included as research and development costs.

How well did you know this?
1
Not at all
2
3
4
5
Perfectly
35
Q

A company enters into a three-year operating lease agreement effective January 1, year 1. The amounts due at the end of each year are $25,000 in year 1, $30,000 in year 2, and $35,000 in year 3. The rate implicit in the lease is 10%.

What amount, if any, is the related liability on the first day of year 2?

Additional Information:

Present Value of $ 1 at 10% for 1 period = 0.9091
Present Value of $ 1 at 10% for 2 period = 0.8264
Present Value of $ 1 at 10% for 3 period = 0.7513
A
$0
B
$73,816
C
$56,198
D
$31,818

A

Explanation:
The correct answer is (C)

At commencement, the initial measurement of the lease liability (regardless of lease classification) is calculated as the present value of the lease payments not yet paid by using the lease term and discount rate determined at lease commencement. Rate Implicit in the Lease is 10%.

So lease liability will be recorded at present value of all minimum lease payments

$25,000	0.9091	$22,727
$30,000	0.8264	$24,793
$35,000	0.7513	$26,296
 	Total	$73,816
The Journal Entry would be:

Dr. ROU Asset $73,816

Cr. Lease Liability $73,816

After lease commencement, a lessee would recognize a single lease cost in the income statement on a straight-line basis. The total remaining lease cost on the commencement date would be $90,000 which is calculated as the total lease payments ($25,000 + $30,000 +$35,000).

This remaining lease cost is recognized on a straight-line basis over the remainder of the lease term (i.e., Lessee would recognize $30,000 in each period, which is calculated as $90,000 ÷ 3).

Lease Liability ROU Asset
Year Beg. Bal. Liab. Accretion Lease Pymt End. Bal. Beg. Bal. Asset Red. End. Bal. Lease Cost
1 73,816 7,382 (25,000) 56,198 73,816 (22,618) 51,198 30,000
2 56,198 5,620 (30,000) 31,818 51,198 (24,380) 26,818 30,000
3 31,818 3,182 (35,000) 0 26,818 (26,818) 0 30,000
The liability on the first day of Year 2 is $56,198.

How well did you know this?
1
Not at all
2
3
4
5
Perfectly
36
Q

On January 2, of the current year, Cole Co. signed an eight-year noncancelable lease for a new machine, requiring $15,000 annual payments at the beginning of each year. The machine has a useful life of 12 years, with no salvage value. Title passes to Cole at the lease expiration date. Cole used straight-line depreciation for all of its plant assets. Aggregate lease payments have a present value on January 2 of $108,000, based on an appropriate rate of interest. For the current year, Cole should record depreciation (amortization) expense for the leased machine at

A
$0
B
$ 9,000
C
$13,500
D
$15,000
A

Explanation:
The correct answer is (B).

A Finance Lease must meet one of the following criteria:

Present Value equals or exceeds substantially all (90%) of the Fair Value
Option to Purchase (exercise is reasonably certain)
Economic Life - Major part (75%) of asset’s economic life is used
Transfer of Ownership at lease termination
Specialized Nature - No alternative use to the lessor at lease termination
Note: the implementation guidance for ASC 842 uses the 75/90 thresholds, even though the standard is principles-based.

The title is transferred at the end of the lease and the leased asset is depreciated over the useful life of the asset, which is 12 years. Cole should record a depreciation expense of $108,000 / 12 = $9,000.

Option (A) is incorrect because, in a finance lease, depreciation is recorded over the useful life of the asset if the ownership transfers at the end of the lease term.

Option (C) is incorrect because depreciation has to be recorded over the useful life of the machine if there is an ownership transfer and not over the term of the lease.

Option (D) is incorrect because depreciation is not equal to the lease payments.

How well did you know this?
1
Not at all
2
3
4
5
Perfectly
37
Q

On December 31, year 1, Roe Co. leased a machine from Colt for a five-year period. Equal annual payments under the lease are $100,000 and are due on December 31 of each year. The first payment was made on December 31, year 1, and the second payment was made on December 31, year 2. The five lease payments are discounted at 10% over the lease term. The present value of lease payments at the inception of the lease and before the first annual payment was $417,000. The lease is appropriately accounted for as a Finance Lease by Roe. In its December 31, year 2 balance sheet, Roe should report a lease liability of

A
$317,000
B
$315,000
C
$285,300
D
$248,700
A

Explanation:
Roe’s annual lease payment is $100,000.

Balance before payment, 12/31, year 1 $ 417,000
Less MLP, 12/31, year 1 (100,000)
Balance after MLP, 12/31, year 1 $ 317,000
Minimum lease payment, 12/31, year 2 $ 100,000
Interest expense ($317,000 × 10%) ( 31,700)
Less: Principal Reduction ( 68,300)
Balance after MLP, 12/31, year 2 $ 248,700

How well did you know this?
1
Not at all
2
3
4
5
Perfectly
38
Q

Main, a pharmaceutical company, leased office space from Ash. Main took possession and began to use the building on July 1, year 1. Rent was due the first day of each month. Monthly lease payments escalated over the 5-year period of the lease as follows:

Period Lease Payment Note
July 1 - September 30, year 1 $0 Rent abatement during move-in, construction
October 1, year 1 - June 30, year 2 17,500
July 1, year 2 - June 30, year 3 19,000
July 1, year 3 - June 30, year 4 20,500
July 1, year 4 - June 30, year 5 23,000
July 1, year 5 - June 30, year 6 24,500
What amount would Main show as deferred rent expense on December 31, year 4?

A
$50,658
B
$52,580
C
$68,575
D
$71,550
A

Explanation:
The correct answer is (D).

When uneven lease payments are made, rent is recognized evenly over the term of the lease. Since the lease term is for 60 Months i.e., 5 years and the lease payments total to $1,201,500.

Rent to be recognized for each month is at $20,025 ($1,201,500/60 months).

For the period ending December 31, year 4, 42 lease payments are made.

Rent for 42 Months = 42 x $20,025 = $841,050.

Lease Expense for 42 months $841,050
Less: Actual amount of rent paid $769,500
Rental expense Deferred $ 71,550
Period Months Amount Paid per month ($) Total Amount Paid ($) Rent Expense
July Year 1 – Sept Year 1 3 $0 $0 $60,075
October Year 1 – December Year 1 3 $17,500 $52,500 $60,075
January Year 2 – June Year 2 6 $17,500 $105,000 $120,150
July Year 2 – December Year 2 6 $19,000 $114,000 $120,150
January Year 3 – June Year 3 6 $19,000 $114,000 $120,150
July Year 3 – December Year 3 6 $20,500 $123,000 $120,150
January Year 4 – June Year 4 6 $20,500 $123,000 $120,150
July Year 4 – December Year 4 6 $23,000 $138,000 $120,150
January Year 5 – June Year 5 6 $23,000 $138,000 $120,150
July Year 5 – December Year 5 6 $24,500 $147,000 $120,150
January Year 6 – June Year 6 6 $24,500 $147,000 $120,150

How well did you know this?
1
Not at all
2
3
4
5
Perfectly
39
Q
Mr. and Mrs. Dart own a majority of the outstanding capital stock of Wall Corp., Black Co., and West, Inc. During the year, Wall advanced cash to Black and West in the amount of $50,000 and $80,000, respectively. West advanced $70,000 in cash to Black. At December 31, none of the advances was repaid. In the combined December 31 balance sheet of these companies, what amount would be reported as receivables from affiliates?
A
$200,000
B
$130,000
C
$ 60,000
D
$0
A

Explanation:
Like consolidated financial statements, combined financial statements should not include intercompany payables and receivables.

How well did you know this?
1
Not at all
2
3
4
5
Perfectly
40
Q
On October 1 of the current year, a U.S. company sold merchandise on account to a British company for 2,000 pounds (exchange rate, 1 pound = $1.43). At the company's December 31 fiscal year end, the exchange rate was 1 pound = $1.45. The exchange rate was 1 pound = $1.50 on collection in January of the subsequent year. What amount would the company recognize as a gain(loss) from foreign currency translation when the receivable is collected?
A
$0
B
$100
C
$140
D
($140)
A

Explanation:
The exchange rate to be used for translation of foreign currency transactions is as follows. At the date the transaction is recognized, each asset, liability, revenue, expense, gain, or loss arising from the transaction should be measured and recorded in the functional currency of the recording entity by use of the exchange rate in effect at that date. At each balance sheet date, recorded balances that are denominated in a currency other than the functional currency of the recording entity should be adjusted to reflect the current exchange rate. These adjustments should be currently recognized as transaction gains or losses and reported as a component of income from continuing operations. Upon settlement, a transaction gain or loss, measured from the transaction date or the most recent intervening balance sheet date (whichever is later), should be included as a component of income from continuing operations for the period in which the transaction is settled. The U.S. company would recognize a gain of $100 when the receivable is collected (settlement date) based on 2,000 pounds × $0.05, the increase in the exchange rate of $1.45 on the balance sheet date to $1.50 on the settlement date.

How well did you know this?
1
Not at all
2
3
4
5
Perfectly
41
Q
The following information pertains to Flint Co.'s sale of 10,000 foreign currency units under a forward contract dated November 1, of the current year for delivery on January 31 of the following year:
11/1	12/31
Spot-rate	$0.80	$0.83
30-day future rates	0.79	0.82
90-day future rates	0.78	0.81Flint entered into the forward contract in order to speculate in the foreign currency. In Flint's income statement for the current year ended December 31, what amount of loss should be reported from this forward contract?
A
$400
B
$300
C
$200
D
$0
A

Explanation:
A forward exchange contract (forward contract) is an agreement to exchange different currencies at a specified future date and at a specified rate (the forward rate). A forward contract is a foreign currency transaction. Therefore, a gain or loss on a forward contract is included in determining net income in accordance with the requirements for other foreign currency transactions. A gain or loss on a speculative forward contract (that is, a contract that does not hedge an exposure) is computed by multiplying the foreign currency amount of the contract by the difference between the forward rate available for the remaining maturity of the contract and the contracted forward rate (or the forward rate last used to measure a gain or loss on that contract for an earlier period).

Foreign currency units 10,000
Times: excess of forward rate available for the remaining maturity of the contract and the contracted forward rate ($0.82 - $0.78) × $0.04
Loss on forward contract $ 400

How well did you know this?
1
Not at all
2
3
4
5
Perfectly
42
Q

Contributed capital in foreign currency financial statements should be translated by means of which of the following?
A
The current exchange rate at the balance sheet date
B
A weighted-average exchange rate for the period
C
The historical exchange rate
D
The weighted-average rate, less dividends declared during the period, at the exchange rate when declared

A

Explanation:

Contributed capital is translated using the historical exchange rate.

How well did you know this?
1
Not at all
2
3
4
5
Perfectly
43
Q

A U.S. company purchased inventory on account at a cost of 1,000 foreign currency units (FCU) from a non-U.S. company on November 15, to be paid on December 15. The FCU is valued at $0.85 on November 15 and at $0.90 on December 15. The journal entry to record payment on December 15 should include which of the following?

A
Debit inventory and credit cash for $850
B
Debit exchange gains and losses and credit accounts payable for $50
C
Debit accounts payable and credit exchange gains and losses for $50
D
Debit accounts payable and credit cash for $850

A

Explanation:
The correct answer is (B).

Purchased inventory would have been recorded for $850 on November 15th because of $0.85 FCU value x 1,000. On December 15th, with an exchange rate of $0.90, it will require $900 to make the payment. Accounts payable will be increased with a credit of $50 and an exchange loss will be recognized with a debit of $50.

Exchange loss $50
A/P $50
(A) is incorrect because inventory will be debited at the purchase date

(C) is incorrect because there is a loss, the reverse of this entry must be passed.

(D) is incorrect because the amount should include exchange gain or loss

How well did you know this?
1
Not at all
2
3
4
5
Perfectly
44
Q

A six-year finance lease entered into on December 31, 2018, specified equal minimum annual lease payments due on December 31 of each year. The first annual lease payment, paid on December 31, 2018, consists of which of the following?

Interest expense
Lease liability.

A
I only.
B
II only.
C
Both I and II.
D
Neither I nor II.
A

Explanation:
Finance lease liability is amortized by the effective interest rate method, where, the annual lease payment is first applied to the interest due, and the remaining is then applied to the reduction of the lease liability.

The first annual lease payment is made on December 31, 2018, the date the finance lease was entered into, no interest would have yet accrued as there is no passage of time and therefore the entire payment would be applied towards the reduction of the lease liability.

How well did you know this?
1
Not at all
2
3
4
5
Perfectly
45
Q

Oak Co. leased equipment for its entire 9-year estimated life, agreeing to pay $50,000 at the start of the lease term on December 31, year 1, and $50,000 annually on each December 31 for the next eight years. The present value on December 31, year 1, of the nine lease payments over the lease term, using the rate implicit in the lease which Oak knows to be 10%, was $316,500. The December 31, year 1, present value of the lease payments using Oak’s incremental borrowing rate of 12% was $298,500. Oak made a timely second lease payment. What amount should Oak report as finance lease liability in its December 31, year 2 balance sheet?

A
$350,000
B
$243,150
C
$228,320
D
$0
A

Explanation:
The Lease Liability balance as of year 2 is 243,150.

Balance before payment, 12/31, year 1 $ 316,500
Less: lease payment, 12/31, year 1 (50,000)
Balance after Lease Payment, 12/31, year 1 $ 266,500
Lease Payment, 12/31, year 2 $ 50,000
Less: Portion allocable to interest ($266,500 × 10%) (26,650)
Less: principal reduction from 12/31, year 2 lease payment (23,350)
Balance after lease payment, 12/31, year 2 $ 243,150

How well did you know this?
1
Not at all
2
3
4
5
Perfectly
46
Q

Koby Co. entered into a finance lease with a vendor for equipment on January 2 for seven years. The equip­ment has no guaranteed residual value. The lease required Koby to pay $500,000 annually on January 2, beginning with the current year. The present value of an annuity due for seven years was 5.35 at the incep­tion of the lease. What amount should Koby capitalize as leased equipment (ROU Asset)?

A
$ 500,000
B
$ 825,000
C
$2,675,000
D
$3,500,000
A

Explanation:
A lessee must record a finance lease Right of Use Asset in an amount equal to Lease Liability (present value of lease payments using implicit rate of interest) plus Initial direct costs plus prepaid lease payments minus lease incentives. As no initial direct costs, prepaid lease payments or lease incentives exist. Koby should capitalize ROU Asset as PV of Lease Payments i.e. $500,000 × 5.35 = $2,675,000.

How well did you know this?
1
Not at all
2
3
4
5
Perfectly
47
Q

Charm Co. owns a delivery truck with an original cost of $10,000 and accumulated depreciation of $7,000. Charm acquired a new truck by exchanging the old truck and paying $2,000 in cash. The new truck has a fair value of $5,000 at the time of the exchange. What amount of gain or loss should Charm recognize?

A
$0
B
$2,000
C
$2,500
D
$3,000
A

Explanation:
The correct answer is (A).

This is a non-monetary exchange without commercial substance. In an exchange without commercial substance when boot is paid only loss is recognized and gain is not recognized.

Realized gain (or loss) = FV of asset received - CV of asset given up (If FV of asset given up is not known, use FV of asset received) = $5,000 - $3,000 = $2,000. However, a gain is not recognized only loss is recognized.

The journal entry would be as follows:

Asset Received $5,000
Accumulated Depreciation $7,000
Asset Given up $10,000
Cash $2,000

How well did you know this?
1
Not at all
2
3
4
5
Perfectly
48
Q

Markson Co. traded a concrete-mixing truck with a book value of $10,000 to Pro Co. for a cement-mixing machine with a fair value of $11,000. Markson needs to know the answer to which of the following questions in order to determine whether the exchange has commercial substance?
A
Does the book value of the asset given up exceed the fair value of the asset received?
B
Is the gain on the exchange less than the increase in future cash flows?
C
Are the future cash flows expected to change significantly as a result of the exchange?
D
Is the exchange nontaxable?

A

Explanation:
A nonmonetary exchange has commercial substance if the entity’s future cash flows are expected to change significantly as a result of the exchange. The acquisition is recorded at the fair value of the asset surrendered or the fair value of the asset received, whichever is more clearly determinable. Gains or losses should be recognized as the earnings process has culminated for the asset exchanged.

How well did you know this?
1
Not at all
2
3
4
5
Perfectly
49
Q

Which of the following computer software costs should be expensed?

A
Conceptual formulation of alternatives in the preliminary project stage
B
Design of the software configuration during the application development stage
C
Costs of producing product masters after technology feasibility was established
D
Installation to hardware during the application development stage

A

Explanation:
The correct answer is (A).

In the case of computer software developed for sale, costs prior to technological feasibility are expensed as R&D and costs associated with converting a technologically feasible program into final commercial form are capitalized.

Costs incurred after software sales begin are inventories and included in COGS.

Computer software costs that are incurred in the preliminary project stage should be expensed as incurred. Activities in this stage include conceptual formulation and evaluation of alternatives; determination of the existence of needed technology; and final selection of alternatives.

The following should be capitalized:

Design of the software configuration during the application development stage
Costs of producing product masters after technology feasibility was established
Installation to hardware during the application development stage

How well did you know this?
1
Not at all
2
3
4
5
Perfectly
50
Q

Which of the following is a criterion for classifying a lease as a finance lease by a lessee?

A
The lease term is for the major part of the remaining economic life of the underlying asset.
B
The present value of the minimum lease payments is 75% or more of the fair value of the leased property.
C
The lease agreement contains an option to purchase the leased property at its fair value at the end of the lease term.
D
The lease agreement requires that title of the leased property remains with the lessor at the end of the lease term.

A

Explanation:
The correct answer is (A).

Lessee meets the criterion that the lease term is for the major part of the remaining economic life of the underlying asset. Per ASC 842, a lessee must meet just one condition to capitalize:

Ownership transfer - The lease transfers ownership of the underlying asset to the lessee by the end of the lease term.
Written purchase option - The lease grants the lessee an option to purchase the underlying asset that the lessee is reasonably certain to exercise. E.g., Lessee has a bargain purchase option to purchase the asset from the lessor at a price which is estimated to be 50% of the asset’s then fair value
No alternative use - The underlying asset is of a specialized nature such that it is expected to have no alternative use to the lessor at the end of the lease term
Equal or excess PV - The PV of the sum of the lease payments and any residual value guaranteed by the lessee equals or exceeds substantially all of the FV of the underlying asset.
Remaining economic life - The lease term is for the major part of the remaining economic life of the underlying asset
Note: If none of the criteria is met, the lease is an operating lease for the lessee

How well did you know this?
1
Not at all
2
3
4
5
Perfectly
51
Q

Under the acquisition method, which of the following would be considered a bargain purchase?

A
The net of the book value of assets acquired and liabilities assumed exceeds the aggregate of the consideration transferred plus the fair value of any noncontrolling interest
B
The net of the fair value of assets acquired and liabilities assumed exceeds the aggregate of the consideration transferred plus the fair value of any noncontrolling interest
C
The aggregate of the consideration transferred plus the fair value of any noncontrolling interest exceeds the net of the book value of assets acquired and liabilities assumed
D
The aggregate of the consideration transferred plus the fair value of any noncontrolling interest exceeds the net of the fair value of assets acquired and liabilities assumed

A

Explanation:
Under the acquisition method, assets acquired and liabilities assumed by the acquirer are always recorded at their fair values. If the net of the fair value of assets acquired and liabilities assumed exceeds the aggregate of the consideration transferred plus the fair value of any noncontrolling interest, that excess is called a bargain purchase. The acquirer recognizes an ordinary gain in earnings on the acquisition date.

How well did you know this?
1
Not at all
2
3
4
5
Perfectly
52
Q

____________________ in a variable interest entity is a (are) contractual, ownership, or other pecuniary interest(s) in an entity that change(s) with changes in the entity’s net asset value exclusive of variable interests.

A
The primary beneficiary
B
Variable interests
C
Subordinated financial support
D
Expected variability
A

Explanation:
Variable interests in a variable interest entity are contractual, ownership, or other pecuniary interests in an entity that change with changes in the entity’s net asset value exclusive of variable interests. A primary beneficiary refers to an enterprise that consolidates a variable interest entity under the provisions of GAAP. Subordinated financial support refers to variable interests that will absorb some or all of an entity’s expected losses if they occur. Expected variability is the sum of the absolute values of the expected residual return and the expected loss.

How well did you know this?
1
Not at all
2
3
4
5
Perfectly
53
Q

On November 1, year 2, Kir Co. signed a contract to purchase 10,000 British pounds on February 2, year 3. The relevant exchange rates are as follows:

Spot Rate

Forward Rate

November 1, year 2

$1.98

$2.05

December 31, year 2

$2.00

$2.06

Kir accounts for the forward contract as a speculative transaction. What amount of gain, if any, should Kir report from this forward contract in its income statement for the year ended December 31, year 2?

A
$0
B
$100
C
$600
D
$700
A

Explanation:
The correct answer is (B).

This forward contract is owned for speculative purposes. All derivatives held for speculative purposes are recorded at fair value. Gains and losses are recorded directly on the income statement. Kir Co. signed a contract to purchase 10,000£ at a forward price of $20,500 (10,000 × 2.05). This is the amount initially recorded on November 1, year 2. As the value of the forward contract has increased to $20,600 (10,000 × 2.06) by December 31, Year 2, an increase of $100 must be recorded to mark up the derivative to fair value. There will be an unrealized holding gain of $100 reported on the income statement on Dec 31, year 2.

(A) is incorrect because a forward contract resulted in a gain.

(C) is incorrect because instead of taking the difference between the forward rate on November 1 and December 31, the spot rate is used as on December 31 to arrive at the gain [i.e. $600 = ($2.06 - $2.00) x £10,000].

(D) is incorrect because gain from the difference in spot rate and forward rate as on December 31 is added [i.e. $700 = ($2.06 - $2.00) x £10,000 + ($2.06 - $2.05) x £10,000].

How well did you know this?
1
Not at all
2
3
4
5
Perfectly
54
Q

On January 1, year 1, Alpha Co. signed an annual maintenance agreement with a software provider for $15,000 and the maintenance period begins on March 1, year 1. Alpha also incurred $5,000 of costs on January 1, year 1, related to software modification requests that will increase the functionality of the software. Alpha depreciates and amortizes its computer and software assets over five years using the straight-line method. What amount is the total expense that Alpha should recognize related to the maintenance agreement and the software modifications for the year ended December 31, year 1?

A
$5,000
B
$13,500
C
$16,000
D
$20,000
A

B-13500

Explanation:
Internal and external training and maintenance costs should be expensed as incurred.The costs of upgrades are capitalized.Capitalized costs are amortized over the estimated useful life,and adjusted periodically with changes in the estimates of the useful life or when the value of the asset is impaired.Alpha would recognize $13,500 in total expense for year 1, determined as follows:

Software modification ($5,000 /5)	$ 1,000
Annual maintenance ($15,000 * 10/12)	12,500
Total Expense, year 1	$13,500
Option (a) is incorrect because it does not depreciate the software modification costs over 5 years and does not include the annual maintenance agreement expenses for the year.Option (c) is incorrect because it accounts for maintenance agreement cost for full year instead of costs incurred for 10 months [$16,000 = $15,000 + ($5,000/5)]. Option (d) is incorrect because it does not depreciate the software modification costs over 5 years and expensed the annual maintenance agreement expenses for the year instead of expensing 10 months costs ($20,000 = $15,000 + $5,000).
How well did you know this?
1
Not at all
2
3
4
5
Perfectly
55
Q

On June 1, year 1, ABC Co. issued a 200,000 euro purchase order for equipment to be supplied by a German company. ABC’s functional currency is the U.S. dollar. The equipment was delivered to ABC on November 1, year 1, and ABC recorded a payable due to the German company. ABC paid for the equipment on January 31, year 2. The following are the exchange rates in effect:
June 1, year 1 1 euro = 1.40 U.S. dollars
November 1, year 1 1 euro = 1.50 U.S. dollars
December 31, year 1 1 euro = 1.35 U.S. dollars
January 31, year 2 1 euro = 1.30 U.S. dollarsUnder IFRS, what is the foreign currency gain or loss that ABC should record for the year ended December 31, year 1?
A
A loss of $30,000.
B
A loss of $20,000.
C
A gain of $10,000.
D
A gain of $30,000.

A

D- gain 30k because it was less than they were suppose to pay

Explanation:
US GAAP and IFRS are similar in their approach to foreign currency translation. Except for the translation of financial statements in hyperinflationary economies, the method used to translate financial statements from the functional currency to the reporting currency is the same. Both require re measurement into the functional currency before translation into the reporting currency. Assets and liabilities are translated at the period-end rate and income statement amounts generally are translated at the average rate, with the exchange differences reported in equity. The equipment and payable would have been recorded on November 1, year 1 at $300,000 (200,000 euro × 1.50 US dollars). At the December 31, year 1 the exchange rate was 1 euro = 1.35 US dollars so the payable would be adjusted to $270,000 (200,000 euro × 1.35 US dollars) and result in a $30,000 foreign currency gain.

How well did you know this?
1
Not at all
2
3
4
5
Perfectly
56
Q

On January 1, year 1, Peabody Co. purchased an investment for $400,000 that represented 30% of Newman Corp.’s outstanding voting stock. For year 1, Newman reported net income of $60,000 and paid dividends of $20,000. At year end, the fair value of Peabody’s investment in Newman was $410,000. Peabody elected the fair value option for this investment. What amount should Peabody recognize in net income for year 1 attributable to the investment?

A
$6,000
B
$10,000
C
$16,000
D
$18,000
A

C- remeber unrealize loss

Explanation:
Entities may choose to measure eligible items at fair value (the “fair value option”) that are not cur­rently required to be measured at fair value. The decision to elect the fair value option is applied instrument by instrument, is irrevocable, and is applied only to an entire instrument. A business entity shall report unrealized gains and losses on items for which the fair value option has been elected in earnings at each subsequent reporting date. The Investment in Newman would be increased by 30% of the net income and decreased by 30% of the dividends, resulting in a year end carrying amount of $412,000 ($400,000 + 18,000 – 6,000). Since the fair value was $410,000, Peabody had an unrealized loss of $2,000. This loss is netted against the investment income previously recognized of $18,000 for a $16,000 net income impact. Dividends do not affect net income (they reduce the Investment account).

How well did you know this?
1
Not at all
2
3
4
5
Perfectly
57
Q
A(n) \_\_\_\_\_\_\_\_\_\_\_\_\_\_\_\_ is a number of currency units, shares, bushels, pounds, or other units specified in a derivative instrument.
A
Underlying
B
Notional amount
C
Firm commitment
D
Counter amount
A

Explanation:
A notional amount is a number of currency units, shares, bushels, pounds, or other units specified in a derivative instrument.

How well did you know this?
1
Not at all
2
3
4
5
Perfectly
58
Q

In a sale-leaseback transaction, the seller-lessee retains the right to substantially all of the remaining use of the equipment sold. The profit on the sale should be deferred and subsequently amortized by the lessee when the lease is classified as:

Finance Lease	Operating lease
A	No	Yes
B	No	No
C	Yes	Yes
D	Yes	No
A

Explanation:
The correct answer is (B). There is no gain deferral under either a finance lease or an operating lease.

According to ASC 842, the transfer of the asset must meet the requirements for a sale per the Revenue Recognition standards.

If there is no sale for the seller-lessee, the buyer-lessor also does not account for a purchase. Any consideration paid for the asset is accounted for as a financing transaction by both the seller-lessee and the buyer-lessor.

If the leaseback is a finance lease from seller-lessee’s perspective, the transfer of the asset is not a sale and therefore, no gain would be recognized.

However, if the leaseback is an operating lease from seller-lessee’s perspective, transfer of the asset is a sale and gain would be recognized immediately.

How well did you know this?
1
Not at all
2
3
4
5
Perfectly
59
Q

Which of the following criteria must be met for a lease to be classified as a direct financing lease?

A
The lease term is for the major part of the remaining economic life of the underlying asset. However, if the commencement date falls at or near the end of the economic life of the underlying asset, this criterion shall not be used for purposes of classifying the lease.
B
The present value of the sum of the lease payments and any residual value guaranteed by the lessee that is not already reflected in the lease payments equals or exceeds substantially all of the fair value of the underlying asset.
C
The underlying asset is of such a specialized nature that it is expected to have no alternative use to the lessor at the end of the lease term.
D
The present value of the sum of the lease payments and any residual value guaranteed by the lessee that is not already reflected in the lease payments and/ or any other third party unrelated to the lessor equals or exceeds substantially all of the fair value of the underlying asset.

A

D

Explanation:
A lessor shall classify the lease as either a direct financing lease or an operating lease. A lessor shall classify the lease as an operating lease unless both of the following criteria are met, in which case the lessor shall classify the lease as a direct financing lease:

The present value of the sum of the lease payments and any residual value guaranteed by the lessee that is not already reflected in the lease payments and/ or any other third party unrelated to the lessor equals or exceeds substantially all of the fair value of the underlying asset.
It is probable that the lessor will collect the lease payments plus any amount necessary to satisfy a residual value guarantee.
According options (A), (B) and (C) are incorrect as per above explanations.

How well did you know this?
1
Not at all
2
3
4
5
Perfectly
60
Q
In general, retained earnings may increase as a result of which of the following?
A
Prior adjustments
B
Net losses suffered by the firm
C
Stock dividends
D
All of the above
A

A

Explanation:
Retained earnings may increase as a result of a prior period adjustment.

How well did you know this?
1
Not at all
2
3
4
5
Perfectly
61
Q
Which of the following financial instruments is not considered a derivative financial instrument?
A
Interest-rate swaps
B
Currency futures
C
Stock-index options
D
Bank certificates of deposit
A

D

Explanation:
A derivative financial instrument is an instrument or contract that has three characteristics; (1) an underlying and notional amount or payment provision, (2) zero or small investment, and (3) net settlement. Bank certificates of deposit do not contain these features. They are investments that normally require a minimum amount of deposit and can be classified as cash if the original maturity is three months or less. The other items listed are all derivative instruments. An interest rate swap is an arrangement where two companies swap interest payments, but not the principal, to limit interest rate risk. Currency futures are contracts to buy or sell a foreign currency on a specific date in the future at a price set today. Stock-index options are privileges to buy or sell a stock index security to be delivered by the derivative contract.

How well did you know this?
1
Not at all
2
3
4
5
Perfectly
62
Q
During the prior year, Manfred Corp. guaranteed a supplier's $500,000 loan from a bank. On October 1 of the current year, Manfred was notified that the supplier had defaulted on the loan and filed for bankruptcy protection. Counsel believes Manfred will probably have to pay between $250,000 and $450,000 under its guarantee. As a result of the supplier's bankruptcy, Manfred entered into a contract in December to retool its machines so that Manfred could accept parts from other suppliers. Retooling costs are estimated to be $300,000. What amount should Manfred report as a liability in its current year December 31 balance sheet?
A
$250,000
B
$450,000
C
$550,000
D
$750,000
A

A

Explanation:
To accrue a contingent liability, the likelihood of the loss must be probable and the amount reasonably estimable. It is probable that Manfred will have to pay between $250,000 to $450,000 under its guarantee of the supplier’s loan. Since no indication is given that any amount in the range is a better estimate than the others, the lower limit of the range, $250,000, is accrued as a contingent liability. On the other hand, the contract Manfred entered into to retool its machines involves a commitment but not a liability because no performance has been made by the other party to the contract. Thus, there is no asset or liability to be reported for the contract.

How well did you know this?
1
Not at all
2
3
4
5
Perfectly
63
Q
Mellow Co. depreciated a $12,000 asset over five years, using the straight-line method with no salvage value. At the beginning of the fifth year, it was determined that the asset will last another four years. What amount should Mellow report as depreciation expense for Year 5?
A
$ 600
B
$ 900
C
$1,500
D
$2,400
A

A-600

Explanation:
A change in the useful life of a depreciable asset is a change in accounting estimate. It is accounted for in the period of change if the change only affects that period, or in the current and subsequent periods if the change affects both. The straight-line depreciation method is a fixed charge method where an equal amount of depreciable cost is allocated to each period. Mellow would have originally been depreciating the asset $2,400 per year over the five years. At the beginning of the fifth year there would be $9,600 ($2,400 × 4) in accumulated depreciation and the asset would have a net book value of $2,400. This $2,400 would now be depreciated over the new remaining life span of four more years. The depreciation expense for year 5, and each of the following three years, would be $2,400 / 4 = $600.

How well did you know this?
1
Not at all
2
3
4
5
Perfectly
64
Q

At the inception of a finance lease, the guaranteed residual value should be

A
Included as part of the lessee’s lease payments, if the guaranteed residual payment is probable to be owed
B
Included as part of the lessee’s lease payments, if the guaranteed residual payment is is 5% or more of the total asset value
C
Included as part of the lessee’s lease payments, if the guaranteed residual payment is less-than-probable to be owed
D
Excluded from the lessee’s lease payments.

A

A

Explanation:
In its lease payment calculation, the lessee would only include the amount that it is probable that the lessee will owe under the residual value guarantee at the end of the lease term.

How well did you know this?
1
Not at all
2
3
4
5
Perfectly
65
Q
A foreign subsidiary of a U.S. parent company should measure its assets, liabilities and operations using
A
The subsidiary’s local currency
B
The subsidiary’s functional currency
C
The U.S. dollar
D
The best available spot rate
A

B

Explanation:
The assets, liabilities, and operations of a foreign subsidiary of a U.S. parent company should be measured in its functional currency. An entity’s functional currency is the currency of the primary economic environment in which the entity operates; normally, that is the currency of the environment in which an entity primarily generates and expends cash. The functional currency of a foreign subsidiary may be its local currency, the U.S. dollar, or another foreign currency.

How well did you know this?
1
Not at all
2
3
4
5
Perfectly
66
Q

A derivative financial instrument is best described as
A
Evidence of an ownership interest in an entity such as shares of common stock.
B
A contract that has its settlement value tied to an underlying notional amount.
C
A contract that conveys to a second entity a right to receive cash from a first entity.
D
A contract that conveys to a second entity a right to future collections on accounts receivable from a first entity.

A

Explanation:
A derivative instrument is an instrument or other contract that has the following three characteristics: 1) at least one underlying and at least one notational amount or payment provision or both, 2) requires no initial net investment, or one that is smaller than would be required for other types of contracts expected to have a similar response to market factor changes, and 3) requires or permits net settlement, can be readily settled net by a means outside the contract, or provides for delivery of an asset that puts the recipient in a position not substantially different from net settlement.

How well did you know this?
1
Not at all
2
3
4
5
Perfectly
67
Q

Which of the following describes the appropriate reporting treatment for a change in accounting estimate?

A
In the period of change with no future consideration
B
By reporting pro forma amounts for prior periods
C
By restating amounts reported in financial statements of prior periods
D
In the period of change and future periods if the change affects both

A

D- change in estimate, handle prospectively (future)

Explanation:
The changes in the accounting estimate are accounted for prospectively. It is implemented in the current period and continued in future periods with no effect on previously reported retained earnings.

Options (A), (B) and (C) are incorrect as per the above explanation.

How well did you know this?
1
Not at all
2
3
4
5
Perfectly
68
Q
Gordon Ltd., a 100% owned British subsidiary of a U.S. parent company, reports its financial statements in local currency, the British pound. A local newspaper published the following U.S. exchange rates to the British pound at year end:
Current rate	$1.50
Historical rate (acquisition)	1.70
Average rate	1.55
Inventory (FIFO)	1.60Which currency rate should Gordon use to convert its income statement to U.S. dollars at year end?
A
1.50
B
1.55
C
1.60
D
1.70
A

B- average

Explanation:
The foreign currency income statement should conceptually use the exchange rate at the time the revenue or expense was recognized. However, due to the impracticability of this where rates change frequently, a weighted-average exchange rate for the period may be used. The current exchange rate would be used for all assets and liabilities at the balance sheet date. The historical exchange rate is used for contributed capital. There is no inventory (FIFO) exchange rate used in the financial statements.

How well did you know this?
1
Not at all
2
3
4
5
Perfectly
69
Q

Steam Co. acquired equipment under a finance lease for six years. Minimum lease payments were $60,000 payable annually at year-end. The interest rate was 5% with an annuity factor for six years of 5.0757. What amount should Steam report as interest expense at the end of the first year of the lease?

A
$ 0
B
$ 3,000
C
$ 15,227
D
$ 18,000
A

C- 15,227

Explanation:
You must first calculate the present value of the lease payments. The present value of the minimum lease payments is the minimum lease payment amount of $60,000 times the annuity factor of 5.0757 equaling $304,542. Because no payment is made until the end of the year, you would take that amount of $304,542 times the interest rate of 5% and get interest expense of $15,227 at the end of the first year of the lease.

Interest Expense 15,227
Lease Liability 44,773
Cash 60,000

How well did you know this?
1
Not at all
2
3
4
5
Perfectly
70
Q
While preparing its year 3 financial statements, Dek Corp. discovered computational errors in its year 2 and year 1 depreciation expense. These errors resulted in overstatement of each year's income by $25,000, net of income taxes. The following amounts were reported in the previously issued financial statements:
Year 2	Year 1
Retained earnings, 1/1	$700,000	$500,000
Net income	150,000	200,000
Retained earnings, 12/31	$850,000	$700,000Dek's year 3 net income is correctly reported at $180,000. Which of the following amounts should be reported as prior period adjustments and net income in Dek's year 3 and year 2 comparative financial statements?
Year	Prior period
adjustment	Net
income
A	2
3	---
$(50,000)	$150,000
180,000
B	2
3	$(50,000)
---	$150,000
180,000
C	2
3	$(25,000)
---	$125,000
180,000
D	2
3	---
---	$125,000
180,000
A

C- $(25,000) y2 $125,000 y3 180,000

Explanation:
The understatement of year 1 and year 2 depreciation expense discovered in preparing the year 3 financial statements represents the correction of an error in previously issued financial statements (i.e., a prior period adjustment). Prior period adjustments are reported retroactively by (1) correcting all prior period statements presented and (2) restating the beginning balance of retained earnings for the first period presented when the error effects extend to a period prior to that one. Therefore, since year 3 and year 2 comparative financial statements are presented, the understatement of year 1 depreciation expense should be reported as a prior period adjustment in the year 2 financial statements as a $25,000 decrease to the beginning balance of retained earnings. In addition, year 2 net income should be reported at $125,000 (i.e., $150,000 - $25,000) in order to reflect the correct amount of year 2 depreciation expense. Since the understatement of depreciation expense pertains to year 1 and year 2 and is reported retroactively, the reported amount of year 3 net income of $180,000 is not affected by the errors.

How well did you know this?
1
Not at all
2
3
4
5
Perfectly
71
Q

Which of the following is true where a foreign operation is relatively self-contained, integrated within one country and performs independently of the parent company?
A
The entity’s functional currency will be in U.S. dollars.
B
The entity’s functional currency will be in the parent company’s currency.
C
Translation of financial statements from the functional currency into the parent’s reporting currency will be required.
D
None of the above is true.

A

C- translation required

Explanation:
Where the foreign operation is relatively self-contained and integrated within one country, the functional currency will be the local currency; and translation of financial statements from the functional currency into the parent’s reporting currency will be required.

How well did you know this?
1
Not at all
2
3
4
5
Perfectly
72
Q
A company began developing computer software to be sold as a separate product on January 1, year 1. During the planning, coding, and testing phases, the company incurred $1,300,000 of costs. On June 30, year 1, the product was determined to be technologically feasible. The company began producing product masters of the software and incurred an additional $750,000 of costs from July 1, year 1, through September 30, year 1. After the software was available for release on October 1, year 1, the company incurred an additional $275,000 of costs relating to maintenance and customer support. What amount of softwarerelated costs should be capitalized?
A
$275,000
B
$750,000
C
$1,300,000
D
$2,050,000
A

B- 750k capitalize

Explanation:
Costs incurred internally in creating a computer software product are charged to expense when incurred as research and development until technological feasibility has been established for the product. Technological feasibility is established only upon completion of a detailed program design or, in its absence, completion of a working model. The $750,000 of costs for producing product masters incurred subsequent to establishing technological feasibility and thus are capitalized. The $1,300,000 of costs incurred during the planning, designing, coding, and testing activities necessary to establish technological feasibility are expensed as research and development when incurred. Capitalization of computer software costs cease when the product is available for general release to customers. Costs of maintenance and customer support shall be charged to expense when related revenue is recognized or when those costs are incurred, whichever occurs first.

How well did you know this?
1
Not at all
2
3
4
5
Perfectly
73
Q

Milt Co. began operations on January 1, year 1. On January 1, year 3, Milt changed its inventory method from LIFO to FIFO for both financial and income tax reporting. If FIFO had been used in prior years, Milt’s inventories would have been higher by $60,000 and $40,000 at December 31, year 3 and year 2, respec­tively. Milt has a 30% income tax rate. What amount should Milt report as the cumulative effect of this accounting change in its income statement for the year ended December 31, year 3?

A
$0
B
$14,000
C
$28,000
D
$42,000
A

A- $0 because its not reported in the I/S

Explanation:
A change from LIFO to another inventory method is a change in accounting principle that should be reported by retrospectively applying the new method to prior periods and adjusting the beginning balance of retained earnings. The cumulative effect of the accounting change is not reported in the income statement.

How well did you know this?
1
Not at all
2
3
4
5
Perfectly
74
Q

In a direct financing lease, the net investment in the lease is the gross investment in the lease plus which of the following?
A
Any unearned income
B
Any unamortized initial direct costs and unearned income
C
Any unamortized initial direct costs less the unearned income
D
Unearned income less any unamortized initial direct costs

A

C

Explanation:
The net investment in the lease is the gross investment in the lease plus any unamortized initial direct costs less the unearned income.

How well did you know this?
1
Not at all
2
3
4
5
Perfectly
75
Q

Under IFRS, changes in accounting policies are

A
Permitted if the change will result in a more reliable and more relevant presentation of the financial statements.
B
Permitted if the entity encounters new transactions, events, or conditions that are substantively different from existing or previous transactions.
C
Required on material transactions, if the entity previously accounted for similar, though immaterial, transactions under an unacceptable accounting method.
D
Required if an alternate accounting policy gives rise to a material change in assets, liabilities, or the current-year net income.

A

A- presentation and reliabilty

Explanation:
A change in accounting principle results from the adoption of a generally accepted accounting prin­ciple (GAAP) different from the previous GAAP used for reporting purposes. US GAAP and IFRS are similar in that the most common reason for an entity to change an accounting principle is that the change is required by a new or revised accounting standard. Under IFRS, the only other acceptable reason for a change is that a different policy results in financial statements that provide reliable and more relevant information about the effects of transactions and other events or conditions on the entity’s financial position, financial performance, or cash flows.

How well did you know this?
1
Not at all
2
3
4
5
Perfectly
76
Q

During the year, Pitt Corp. incurred costs to develop and produce a routine, low-risk computer software product, as follows:

Completion of detail program design $13,000
Costs incurred for coding and testing to establish technological feasibility 10,000
Other coding costs after the establishment of technological feasibility 24,000
Other testing costs after the establishment of technological feasibility 20,000
Costs of producing product masters for training materials 15,000
Duplication of computer software and training materials from product masters (1,000 units) 25,000
Packaging product (500 units) 9,000
In Pitt’s December 31 balance sheet, what amount should be reported in inventory?

A
$25,000
B
$34,000
C
$40,000
D
$49,000
A

Explanation:
The correct answer is (B).

Computer software developed to sale, lease or market as a product:

RD costs are expensed. These are costs incurred prior to technological feasibility (technological feasibility is established upon completion of a detailed program or design or completion of a working model). The completion of detail program design for $13,000 and costs incurred for coding and testing to establish technological feasibility of $10,000 are expensed.
Costs associated with converting a technologically-feasible program into a final commercial form is capitalized. $24,000 in coding costs after the establishment of technological feasibility, $20,000 in testing costs after the establishment of technological materials, and costs of producing product masters for training materials of $15,000 are all capitalized.
Costs incurred after software sales begin are inventoried - this will include duplication of computer software and training materials from product masters for $25,000 and packaging product for $9,000.
The year-end balance sheet would report inventory of $34,000 (i.e. $25,000 + $9,000).

How well did you know this?
1
Not at all
2
3
4
5
Perfectly
77
Q

Rig Co. sold its factory at a gain, and simultaneously leased it back for 10 years. The factory’s remaining economic life is 20 years. The lease was reported as an operating lease. At the time of sale, Rig should report the gain as:

A
In the income statement
B
An asset valuation allowance.
C
A separate component of stockholders' equity.
D
A deferred credit.
A

Explanation:
The correct answer is (A).

FASB issued ASC 842 to amend accounting & reporting for leases under which for a sale to occur in the context of a sale and leaseback transaction, the transfer of the asset must meet the requirements for a sale per the Revenue Recognition standards. If there is no sale for the seller-lessee, the buyer-lessor also does not account for a purchase.

Any consideration paid for the asset is accounted for as a financing transaction by both the seller-lessee and the buyer-lessor. If the leaseback is a finance lease from seller-lessee’s perspective, transfer of the asset is not a Sale.

However, here the leaseback is an operating lease.

The transaction is recognized as a sale and the entire gain is recognized immediately on the income statement.

How well did you know this?
1
Not at all
2
3
4
5
Perfectly
78
Q

In its financial statements, Pulham Corp. uses the equity method of accounting for its 30% ownership of Angles Corp. At December 31 of the current year, Pulham has a receivable from Angles. How should the receivable be reported in Pulham’s year-end financial statements?
A
None of the receivable should be reported, but the entire receivable should be offset against Angles’ payable to Pulham.
B
Seventy percent of the receivable should be separately reported, with the balance offset against 30% of Angles’ payable to Pulham.
C
The total receivable should be disclosed separately.
D
The total receivable should be included as part of the investment in Angles, without separate disclosure.

A

C

Explanation:
The 30% level of ownership is not sufficient to meet the consolidation requirement of a controlling interest (i.e., > 50%). Therefore, there is no reason to treat the separate entities as one. The total receivable should be reported in the same manner as any receivables from unrelated entities.

How well did you know this?
1
Not at all
2
3
4
5
Perfectly
79
Q

Bale Co. incurred $100,000 of acquisition costs related to the purchase of the net assets of Dixon Co. The $100,000 should be

A
Allocated on a pro rata basis to the nonmonetary assets acquired.
B
Capitalized as part of goodwill and tested annually for impairment.
C
Capitalized as another asset and amortized over five years
D
Expensed as incurred in the current period.

A

D- Acquision costs are expensed

Explanation:
Acquisition costs are those costs the acquirer incurs to effect a business combination, and include: finders’ fees; advisory, legal, accounting, valuation, and other professional or consulting fees; general adminis­trative costs; and costs of registering and issuing debt and equity securities. Acquisition costs are expensed in the period in which the costs are incurred and the services are received.

Option (A), (B) and (C) are incorrect as per above explanatio

How well did you know this?
1
Not at all
2
3
4
5
Perfectly
80
Q
Ahm Corp. owns 90% of Bee Corp's common stock and 80% of Cee Corp.'s common stock. The remaining common shares of Bee and Cee are owned by their respective employees. Bee sells exclusively to Cee, Cee buys exclusively from Bee, and Cee sells exclusively to unrelated companies. Selected information for Bee and Cee follows:
Bee Corp.	Cee Corp.
Sales	$130,000	$91,000
Cost of sales	$100,000	$65,000
Beginning inventory	None	None
Ending inventory	None	65,000What amount should be reported as gross profit in Bee and Cee's combined income statement for the current year ended December 31?
A
$26,000
B
$41,000
C
$47,800
D
$56,000
A

B- 41k

Explanation:
The gross profit to be reported in the combined income statement should be based on the final sale to the outside customer and the original cost to the initial affiliate. Because Bee sells exclusively to Cee, from a combined perspective, the only sales that have occurred are the sales of Cee to unrelated companies, which total $91,000. Of the $130,000 in intercompany’sales’ from Bee to Cee, half (or $65,000) remained in Cee’s ending inventory and the other half (or $65,000) were sold. However, from a combined perspective, the combined cost of the ending inventory and the cost of sales is only $100,000, not $130,000. The additional $30,000 represents intercompany profits, which are eliminated in combined statements. The $100,000, in the combined statements, is allocated half to cost of sales and half to ending inventory; based on the ratio of ending inventory and cost of sales of Cee.

Gross sales $91,000
Cost of sales (50,000)
Gross profit $ 41,000

How well did you know this?
1
Not at all
2
3
4
5
Perfectly
81
Q

On December 31, year 1, Andover Co. acquired Barrelman, Inc. Before the acquisition, a product lawsuit seeking $10 million in damages was filed against Barrelman. As of the acquisition date, Andover believed that it was probable that a liability existed and that the fair value of the liability was $5 million. What amount should Andover record as a liability as of December 31, year 1?

A
$0
B
$5,000,000
C
$7,500,000
D
$10,000,000
A

Explanation:
The correct answer is (B).

A $10,000,000 product liability lawsuit was filed against Barrelman. As of the acquisition date by Andover Co., it was probable that a liability existed and that its fair value was $5,000,000. Andover believed that the loss from the product lawsuit was probable and estimated the loss to be $5 million, Andover Co. will accrue a liability of $5 million in its financial statements in year 1 as well as disclose the liability in the notes to the financial statements

Probability of loss     	Disclose in notes to Financial Statements	Accrue
Remote   	No  	No
Reasonably possible      	Yes     	No
Probable and estimable	Yes  	Yes  
Probable but not estimable	Yes	No
How well did you know this?
1
Not at all
2
3
4
5
Perfectly
82
Q

Neal Corp. entered into a nine-year Finance lease on a warehouse on December 31, year 1. Lease payments of $50,000, which are due annually, beginning on December 31, year 2, and every December 31 thereafter. Neal does not know the interest rate implicit in the lease; Neal’s incremental borrowing rate is 9%. The rounded present value of an ordinary annuity for nine years at 9% is 5.6. What amount should Neal report as Finance lease liability at December 31, year 1?

A
$280,000
B
$291,200
C
$450,000
D
$468,000
A

A- 280k (50k*5.6)

Explanation:
At a Finance Lease inception, the lessee records an ROU Asset and corresponding lease liability at the present value of the lease payments not yet paid.

The lessee’s incremental borrowing rate of 9% is used because the lessee does not know the rate implicit in the lease.

The annual Lease Payments is $50,000. Neal’s first Lease Payment is not due until 12/31, year 2.

Thus, in its 12/31, year 1 balance sheet, Neal should report a Finance Lease liability of $280,000 (i.e., $50,000 × 5.6).

How well did you know this?
1
Not at all
2
3
4
5
Perfectly
83
Q

As an inducement to enter a lease, Graf Co., a lessor, granted Zep, Inc., a lessee, twelve months of free rent under a five-year operating lease. The lease was effective on January 1, year 1, and provides for monthly rental payments to begin January 1, year 2. Zep made the first rental payment on December 30, year 1. In its year 1 income statement, Graf should report rental revenue in an amount equal to

A
Zero.
B
Cash received during year 1.
C
One-fourth of the total cash to be received over the life of the lease.
D
One-fifth of the total cash to be received over the life of the lease.
A

Explanation:
The correct answer is (D).
It 1/5 of the total rent because they are asking for revenue not the actual rent payment, so everything excluded discounts or free rent

Total rent income receivable for the entire lease term is divided evenly over each period in line with matching principle regardless of the pattern of payments. Whether payments increase or decrease during the term of the lease, or whether the lease contains periods that may be rent-free, or involves nonrefundable deposits, the total of the payments received is recognized evenly over the term of the lease.

Over the 5-year term of the lease, Graf Co. will receive monthly rent equal to one-fifth of the total cash to be received over the life of the lease.

How well did you know this?
1
Not at all
2
3
4
5
Perfectly
84
Q

Which of the following risks are inherent in an interest rate swap agreement?

The risk of exchanging a lower interest rate for a higher interest rate
The risk of nonperformance by the counterparty to the agreement
A
I only.
B
II only.
C
Both I and II.
D
Neither I nor II.
A

C- Both

Explanation:
An interest rate swap agreement is an arrangement used to limit interest rate risk. Two companies swap interest payments, but not the principal, in an agreement such as an exchange of a variable interest rate for a fixed rate. The risk of accounting loss from an interest rate swap includes both (1) the risk of exchanging a lower interest rate for a higher rate and (2) the risk of nonperformance by the other party.

How well did you know this?
1
Not at all
2
3
4
5
Perfectly
85
Q

Spring Corp. entered into a five-year lease agreement with Fall Corp. Spring, the lessee, paid an additional $5,000 nonrefundable lease bonus to Fall upon signing the operating lease agreement. When would Fall recognize in income the nonrefundable lease bonus paid by Spring?

A
When received
B
Over the life of the lease
C
At the expiration of the lease
D
At the inception of the lease
A

Explanation:
The correct answer is (B).

The lease bonus is treated as deferred (prepaid) rent and amortized using the Straight-Line Method (SLM) over the lease term.

Option (A), (C) and (D) are incorrect as per the above explanation.

How well did you know this?
1
Not at all
2
3
4
5
Perfectly
86
Q
Sanni Co. had $150,000 in cash-basis pretax income for the year. At the current year end, accounts receivable decreased by $20,000 and accounts payable increased by $16,000 from their previous year-end balances. Compared to the accrual-basis method of accounting, Sanni's cash-basis pretax income is
A
Higher by $4,000
B
Lower by $4,000
C
Higher by $36,000
D
Lower by $36,000
A

D- Lower by 36k

Explanation:
Compared to the accrual basis of accounting, Sanni’s cash-basis pretax income is higher by $36,000. Accounts receivable decreasing by $20,000 would require a reduction of $20,000 to derive the accrual basis. Accounts payable increasing by $16,000 would require a reduction of $16,000 to derive the accrual basis.

How well did you know this?
1
Not at all
2
3
4
5
Perfectly
87
Q
Campbell Corp. exchanged delivery trucks with Highway, Inc. Campbell's truck originally cost $23,000, its accumulated depreciation was $20,000, and its fair value was $5,000. Highway's truck originally cost $23,500, its accumulated depreciation was $19,900, and its fair value was $5,700. Campbell also paid Highway $700 in cash as part of the transaction. The transaction lacks commercial substance. What amount is the new book value for the truck Campbell received?
A
$5,700
B
$5,000
C
$3,700
D
$3,000
A

C- 3700 new book value is based on the CV

Explanation:
In general, accounting for nonmonetary transactions should be based on the fair values (FV) of the assets involved. Nonmonetary exchanges should be based on recorded amounts, rather than FV, of the exchanged assets if any of the following apply: 1) neither the FV of the assets received nor FV of the assets surrendered is reasonably determinable, or 2) the transaction is an exchange of a product or property held for sale in the ordinary course of business for a product or property to be sold in the same line of business to facilitate sales to customers, or 3) the exchange lacks commercial substance. Since the above transaction lacks commercial substance, the new book value for the truck Campbell received ($3,700) is determined as follows:

New Truck (plug) 3,700
Accm Depr Old Truck 20,000
Old Truck (cost) 23,000
Cash 700

How well did you know this?
1
Not at all
2
3
4
5
Perfectly
88
Q

Peg Co. leased equipment from Howe Corp. on July 1 of the current year for an 8-year period. Equal pay­ments under the lease are $600,000 and are due on July 1 of each year. The first payment was made on July 1 of the current year. The rate of interest contemplated by Peg and Howe is 10%. The cash selling price of the equipment is $3,520,000, and the cost of the equipment on Howe’s accounting records is $2,800,000. The lease is appropriately recorded as a sales-type lease. What is the amount of profit on the sale and interest revenue that Howe should record for the current year ended December 31?

Profit on sale	Interest revenue
A	$720,000	$176,000
B	$720,000	$146,000
C	$45,000	$176,000
D	$45,000	$146,000
A

B- 720k & 146k remember for the interest rev its semi, so divide the int rate

Explanation:
At the commencement date, a lessor shall recognize each of the following and derecognize the underlying asset.

A net investment in the lease.
Selling profit or selling loss arising from the lease:
Initial direct costs as an expense if, at the commencement date, the fair value of the underlying asset is different from it carrying amount. If the fair value of the underlying asset equals it carrying amount, initial direct costs are deferred at the commencement date and included in the measurement of the net investment in the lease.
Net Investment in Lease 3,520,000
Carrying Value of the underlying Asset 2,800,000
Selling Profit 720,000
The interest revenue is determined by applying the interest rate implicit in the lease to the lessor’s net receivable. The excess of the fair value of leased property at the lease inception over its cost or carrying amount is dealer’s profit from a sales-type lease and recognized fully at lease inception.

Sales price of equipment	$ 3,520,000
Equipment cost	 (2,800,000)
Profit on sale	$ 720,000
Net Investment in Lease before receipt, 7/1	    $ 3,520,000
Less receipt, 7/1	    (600,000)
Net Investment in Lease after receipt, 7/1	            2,920,000
Interest (10% / 2)	×          5%
Interest revenue	$ 146,000
Cash	600,000	 
Interest Income	 	146,000
Net Investment in Lease	 	454,000
How well did you know this?
1
Not at all
2
3
4
5
Perfectly
89
Q

In which of the following situations should a company report a prior-period adjustment?

A
A change in the estimated useful lives of fixed assets purchased in prior years
B
The correction of a mathematical error in the calculation of prior years’ depreciation
C
A switch from the straight-line to double-declining balance method of depreciation
D
The scrapping of an asset prior to the end of its expected useful life

A

Explanation:
Accounting errors are recorded as prior period adjustments. The correction of a mathematical error in the calculation of prior years’ depreciation would require the company to report a prior period adjustment. If comparative Financial Statements are presented and Financial Statement for the year with error are presented, the error is corrected in those Financial Statements.

Options (A), (C) and (D) are incorrect because these are changes in accounting estimates. These will be accounted for prospectively and Financial Statements for prior periods will not be adjusted.

How well did you know this?
1
Not at all
2
3
4
5
Perfectly
90
Q

On January 1, year 1, Eber Co. leased equipment under a four-year finance lease. The present value of minimum lease payments is $348,680. The equipment had a five-year economic life and a $20,000 guaranteed residual value. The equipment reverted to the lessor at the end of the lease. What amount should Eber report as depreciation expense at December 31, year 1?

A
$87,170
B
$82,170
C
$69,736
D
$65,736
A

Explanation:
The correct answer is (A). 348,680 / 4 = $87,170.

At commencement, the initial measurement of the ROU asset (regardless of lease classification) is calculated as the lease liability (present value of minimum lease payments, increased by any initial direct costs and prepaid lease payments, reduced by any lease incentives received before commencement.

The present value of minimum lease payments is $348,680.

Depreciation expense on December 31st, year 1 will be $348,680 / 4 = $87,170.

There is no title transfer or bargain purchase option in this financial lease.

The asset will be depreciated over the shorter of either the lease term or useful life, which in this case is the 4-year lease term compared to the 5-year useful life.

How well did you know this?
1
Not at all
2
3
4
5
Perfectly
91
Q

A company incurred the following costs to complete a business combination in the current year:

Issuing debt securities	$30,000
Registering debt securities	25,000
Legal fees	10,000
Due diligence costs	1,000
What amount should be reported as current-year expenses, not subject to amortization?
A
$1,000
B
$11,000
C
$36,000
D
$66,000
A

Explanation:
The correct answer is (B).

Acquisition-related costs are costs the acquirer incurs to effect a business combination.

Those costs include finder’s fees; advisory, legal, accounting, valuation, and other professional or consulting fees; general administrative costs; and costs of registering and issuing debt and equity securities.

The acquirer should account for acquisition-related costs as expenses in the periods in which the costs are incurred and the services are received, with one exception—the costs to register and issue debt or equity securities shall be recognized in accordance with other applicable GAAP.

Legal fees and due diligence Costs ($11,000 = $10,000 + $1,000) are expensed. Direct, indirect or general costs are all expenses e.g., legal, accounting, consulting, finder’s fees, G&A.

Costs associated with the issuance and registration of debt or equity securities, are treated as Bond Issue Costs and netted against the proceeds.

How well did you know this?
1
Not at all
2
3
4
5
Perfectly
92
Q

Which of the following is a criterion for classifying a lease as a Finance lease by a lessee?

A
The term lease covers the rest of the economic life of the underlying asset.
B
The lease transfers ownership of the underlying asset to the lessor by the end of the lease term.
C
The lease grants the lessee an option to purchase the underlying asset that the lessee is not expected to exercise.
D
The underlying asset is of such a generalized nature that it is expected to have no alternative use to the lessor at the end of the lease term.

A

Explanation:
The correct answer is (A).

A Finance Lease must meet one of the following criteria:

Present Value equals or exceeds substantially all (90%) of the Fair Value
Option to Purchase (exercise is reasonably certain)
Economic Life - Major part (75%) of asset’s economic life is used
Transfer of Ownership at lease termination
Specialized Nature - No alternative use to the lessor at lease termination
Note: the implementation guidance for ASC 842 uses the 75/90 thresholds, even though the standard is principles-based.

(B) is incorrect. The lease would transfer ownership to the lessee, not the lessor.

(C) is incorrect. The lessee would reasonably certain to exercise the purchase option.

(D) is incorrect. The asset is of a specialized nature, not generalized.

How well did you know this?
1
Not at all
2
3
4
5
Perfectly
93
Q

A company leases trucks and properly classifies the leases as finance leases. The leases have a 10-year term, and the lease calculations were done three years ago when interest rates were lower. Which of the following is the appropriate accounting treatment, if any, for the application of the fair value option to lease transactions?

A
Leases are not eligible for the fair value option
B
Recognize the change to fair value accounting with accumulative adjustment to beginning retained earnings
C
Recognize the change to fair value accounting with an unrealized loss in the income statement
D
Recognize the change to fair value accounting with an unrealized loss in accumulated other comprehensive income

A

Explanation:
The correct answer is (A).

Generally, the fair value option does not apply to financial assets and liabilities under leases. Therefore, the finance leases in the question are not eligible for the fair value option.

How well did you know this?
1
Not at all
2
3
4
5
Perfectly
94
Q
Bell Co. is a defendant in a lawsuit that could result in a large payment to the plaintiff. Bell's attorney believes that there is a 90% chance that Bell will lose the suit, and estimates that the loss will be anywhere from $5,000,000 to $20,000,000 and possibly as much as $30,000,000. None of the estimates are better than the others. What amount of liability should Bell report on its balance sheet related to the lawsuit?
A
$0
B
$5,000,000
C
$20,000,000
D
$30,000,000
A

Explanation:
B is corrent.- 5M

The requirement is to determine the amount of liability that Bell should report on its balance sheet related to the lawsuit. In this case, ASC Topic 450 requires accrual of the lower limit of the estimate of probable loss, and disclosure of the possible amounts. Therefore, this answer is correct; Bell should accrue $5,000,000.

How well did you know this?
1
Not at all
2
3
4
5
Perfectly
95
Q
Yola Co. and Zaro Co. are fuel oil distributors. To facilitate the delivery of oil to their customers, Yola and Zaro exchanged ownership of 1,200 barrels of oil without physically moving the oil. Yola paid Zaro $30,000 to compensate for a difference in the grade of oil. On the date of the exchange, cost and market values of the oil were as follows:
Yola Co.	Zaro Co.
Cost	$100,000	$126,000
Market values	$120,000	$150,000In Zaro's income statement, what amount of gain should be reported from the exchange of the oil?
A
$0
B
$4,800
C
$24,000
D
$30,000
A

B- 4800
remember to recognize the cash gain which is 20%

Explanation:
The transaction is an exchange of a product held for sale in the ordinary course of business for a product to be sold in the same line of business to facilitate sales to customers other than parties to the exchange and does not result in the culmination of an earnings process. The nonmonetary exchange shall be measured based on the recorded amount of the nonmonetary assets relinquished. Only to the extent cash (boot) has been received has a portion of the asset exchanged been sold and a gain can be recognized. The fair value of the asset acquired is reduced by the portion of the gain realized which is not recognized. The gain recognized on the exchange is determined by the ratio of the cash to the total consideration received (asset received and cash).

Cash received $ 30,000
Fair value of inventory received 120,000
Total fair value received 150,000
Carrying amount of inventory exchanged (126,000)
Gain realized on exchange 24,000
Cash $ 30,000
Total fair value received / 150,000
Extent earnings process culminated x 20%
Gain recognized on exchange $ 4,800

How well did you know this?
1
Not at all
2
3
4
5
Perfectly
96
Q

Which of the following examples would require restatement of prior years’ financial statements?

A
A calculation change of warranty obligations based on updated claim information for the prior year.
B
A change from the income tax basis of accounting to the accrual basis.
C
An insurance premium that was due in the prior year but that lapsed because the policy was not paid.
D
An intangible asset with a remaining estimated amortization period of two years, which is determined to be obsolete.

A

Explanation:
The correct answer is (B).

A change from the income tax basis of accounting to the accrual basis will require a prior-period adjustment (restatement) of prior years’ financial statements. The other answers would not lead to a restatement but handled in another way. For example, obsolete inventory leads to a write-down and a change in accounting estimate, which is handled prospectively.

(A), (C) and (D) are incorrect because they are changes in the accounting estimates, which are accounted for prospectively and do not affect previously reported retained earnings.

How well did you know this?
1
Not at all
2
3
4
5
Perfectly
97
Q

Althouse Co. discovered that equipment purchased on January 2 for $150,000 was incorrectly expensed at the time. The equipment should have been depreciated over five years with no salvage value. What amount, if any, should be adjusted to Althouse’s depreciation expense at January 2, the beginning of the third year, when the error was discovered?

A
$0
B
$30,000
C
$60,000
D
$150,000
A

A- 0

Explanation:
The company should reverse the impact of expensing the equipment purchased and account for the depreciation of that equipment for 2 years. This correction will be recognized by adjusting the beginning retained earnings in the beginning of third year. There will be no depreciation expense reported in the beginning of the third year.

Options (B), (C) and (D) are incorrect as per the above explanation.

How well did you know this?
1
Not at all
2
3
4
5
Perfectly
98
Q

Crane Mfg. leases a machine from Frank Leasing. Ownership of the machine returns to Frank after the 15-year lease expires. The machine is expected to have an economic life of 17 years. At this time, Frank is unable to predict the collectability of the lease payments to be received from Crane. The present value of the minimum lease payments exceeds 90% of the fair value of the machine. What is the appropriate classification of this lease for Crane?

A
Operating
B
Leveraged
C
Finance
D
Installment
A

Explanation:
The correct answer is (C).

A Finance Lease must meet one of the following criteria:

Present Value equals or exceeds substantially all (90%) of the Fair Value
Option to Purchase (exercise is reasonably certain)
Economic Life - Major part (75%) of asset’s economic life is used
Transfer of Ownership at lease termination
Specialized Nature - No alternative use to the lessor at lease termination
Note: the implementation guidance for ASC 842 uses the 75/90 thresholds, even though the standard is principles-based.

If none of the above criteria is met, the lease is an operating lease for the lessee

The lease qualifies as a finance lease since the lease term is for the majority of the useful life of the property and also since the PV of the sum of the lease payments equals or exceeds substantially all of the FV of the underlying asset.

Option (A), (B) and (D) are incorrect as per above explanation.

How well did you know this?
1
Not at all
2
3
4
5
Perfectly
99
Q

Wind Co. incurred organization costs of $6,000 at the beginning of its first year of operations. How should Wind treat the organization costs in its financial statements in accordance with GAAP?

A
Never amortized
B
Amortized over 60 months
C
Amortized over 40 years
D
Expensed immediately
A

Explanation:
The correct answer is (D).

Generally accepted accounting principles that apply to established operating enterprises govern the recognition of revenue by a development stage enterprise and determine whether a cost incurred by a development stage enterprise is to be charged to expense when incurred or is to be capitalized or deferred.

Accordingly, capitalization or deferral of costs shall be subject to the same assessment of recoverability that would be applicable in an established operating enterprise.

Organization costs are written off over 60 months for tax purposes, but for financial accounting purposes. Start-up activities, including organization costs, should be expensed as incurred.

How well did you know this?
1
Not at all
2
3
4
5
Perfectly
100
Q

Compared to the accrual basis of accounting, the cash basis of accounting understates income by the net decrease during the accounting period of

Accounts receivable	Accrued expenses
A	Yes	Yes
B	Yes	No
C	No	No
D	No	Yes
A

D- No, Yes

Explanation:
Let’s say:
Net Income = 100,000. Decrease in AR = 50,000 and Decrease in Accrued Expense = 40,000.

To convert Accural to Cash, a Decrease in AR would be an added to net income and a decrease in accrued expense would be subtracted to net income.

Accounts Receivable

100,000 NI + 50000 = 150,000 cash basis

Accrued Expense

100,000 NI - 40,000 = 60,000 cash basis

So to answer the question, cash basis of 60k is understated compared to accrual basis for accrued expense. For Account receivable, cash basis of 150k is overstated.

How well did you know this?
1
Not at all
2
3
4
5
Perfectly
101
Q

Under IFRS, each of the following is a disclosure requirement related to the correction of a material prior-period error, except

A
A description of the internal controls put in place to prevent the occurrence of the error in future periods.
B
The impact of the correction on basic and diluted earnings per share for each period presented.
C
The nature of the error.
D
The amount of the correction at the beginning of the earliest period presented.

A

Explanation:
The correct answer is (A).

Under International Financial Reporting Standards (IFRS), the disclosure requirement related to the correction of material prior period error includes describing the nature of the error. The cumulative effect adjustment is made to the beginning retained earnings in the Balance Sheet. The impact of correction on earnings per share for each period is also disclosed. There is no such requirement of describing the internal controls to prevent occurrence of the errors in future periods.

Options (B), (C) and (D) are incorrect as per the above explanation

How well did you know this?
1
Not at all
2
3
4
5
Perfectly
102
Q

Zest Co. owns 100% of Cinn, Inc. On January 2, Zest sold equipment with an original cost of $80,000 and a carrying amount of $48,000 to Cinn for $72,000. Zest had been depreciating the equipment over a five-year period using straight-line depreciation with no residual value. Cinn is using straight-line depreciation over three years with no residual value. In Zest’s December 31, consolidating worksheet, by what amount should depreciation expense be decreased?

A
$ 0
B
$ 8,000
C
$16,000
D
$24,000
A

Explanation:
Sales of fixed assets between members of an affiliated group will result in the recognition of gain or loss by the seller, if the selling price differs from the carrying amount of the asset. However, no gain or loss has taken place for the consolidated entity; assets merely have been transferred from one set of books to another. The buyer of the asset will record it in its books at the purchase price; subsequent depreciation charges will be based upon this purchase price, thus requiring adjustment if the price is different from the book value. The equip­ment was being depreciated by the seller at a rate of $16,000 per year ($80,000 original cost / 5 years) and had been depreciated $32,000 ($80,000 original cost – $48,000 book value with no residual value) before being sold. The buyer will record depreciation expense of $24,000 ($72,000 cost / 3 years). With intercompany sales of fixed assets, in the year of the sale the carrying amount of the asset is restored to its original book value and the gain (loss) recorded by the seller is eliminated. For each period depreciation expense and accumulated depreciation are adjusted to reflect the original book value of the asset. The consolidating worksheet in this problem should decrease depreciation expense $8,000, the difference between the $24,000 recorded by the buyer based on the purchase price and the $16,000 determined based on the initial carrying amount.

How well did you know this?
1
Not at all
2
3
4
5
Perfectly
103
Q

On January 1 of the current year, Mollat Co. signed a 7-year lease for equipment having a 10-year economic life. The present value of the monthly lease payments equaled 80% of the equipment’s fair value. The lease agreement provides for neither a transfer of title to Mollat nor a bargain purchase option. In its current year income statement, Mollat should report

A
Rent expense equal to the current year lease payments.
B
Rent expense equal to the current year lease payments less interest expense.
C
Lease amortization equal to one-tenth of the equipment’s fair value.
D
Lease amortization equal to one-seventh of 80% of the equipment’s fair value.

A

Explanation:
The correct answer is (A)

A Finance Lease must meet one of the following criteria:

Present Value equals or exceeds substantially all (90%) of the Fair Value
Option to Purchase (exercise is reasonably certain)
Economic Life - Major part (75%) of asset’s economic life is used
Transfer of Ownership at lease termination
Specialized Nature - No alternative use to the lessor at lease termination
Note: the implementation guidance for ASC 842 uses the 75/90 thresholds, even though the standard is principles-based.

None of the above criteria are met, so the lease is an operating lease and each lease payment is treated as rent expense.

How well did you know this?
1
Not at all
2
3
4
5
Perfectly
104
Q

Brite Corp. had the following liabilities at December 31 of the current year:
Accounts payable $ 55,000
Unsecured notes, 8%, due 07/01 next year 400,000
Accrued expenses 35,000
Contingent liability 450,000
Deferred income tax liability 25,000
Senior bonds, 7%, due 03/31 next year 1,000,000
The contingent liability is an accrual for possible losses on a $1,000,000 lawsuit filed against Brite. Brite’s legal counsel expects the suit to be settled in 2 years, and has estimated that Brite will be liable for damages in the range of $450,000 to $750,000.

The deferred income tax liability is not related to an asset for financial reporting and is expected to reverse in 2 years.

What amount should Brite report in its current year December 31 balance sheet for current liabilities?

A
$515,000
B
$940,000
C
$1,490,000
D
$1,515,000
A

Explanation:
The accounts payable and accrued expenses are current. Since the senior bonds are due within one year, they should be reported in the balance sheet as a current liability. Since the contingent liability is possible, not probable, it should not be accrued as a liability but should be disclosed in a footnote. The deferred income tax liability is not related to an asset for financial reporting and is expected to reverse more than one year after the balance sheet date, so it should be reported as a noncurrent liability

Accounts payable	$ 55,000
Unsecured notes, due July 1	400,000
Accrued expenses	35,000
Senior bonds, due March 31	  1,000,000
Current liabilities	$1,490,000
How well did you know this?
1
Not at all
2
3
4
5
Perfectly
105
Q
On December 31, Year 1, Bit Co. had capitalized costs for a new computer software product with an economic life of five years. Sales for year 2 were 30 percent of expected total sales of the software. At December 31, Year 2, the software had a net realizable value equal to 90 percent of the capitalized cost. What percentage of the original capitalized cost should be reported as the net amount on Bit's December 31, Year 2, balance sheet?
A
70%
B
72%
C
80%
D
90%
A

Explanation:
The annual amortization of the capitalized software cost is the greater of: (1) the ratio of current revenues to current and future revenues (e.g., 30%) or (2) the straight-line method over the remaining useful life of the software including the period to be reported upon (e.g., 1 / 5 = 20%). Because the software has a net realizable value of 90% of the capitalized cost, it can be reported on the balance sheet at 70% (i.e., 1 - 30%) of its capitalized cost.

How well did you know this?
1
Not at all
2
3
4
5
Perfectly
106
Q

Which of the following should be reported in accumulated other comprehensive income?
A
Discount on convertible bonds that are common stock equivalents
B
Premium on convertible bonds that are common stock equivalents
C
Cumulative foreign exchange translation loss
D
Organization costs

A

Explanation:
If an entity’s functional currency is a foreign currency, which has not experienced significant inflation, translation adjustments result from the process of translating that entity’s financial statements into the reporting currency. Translation adjustment should not be included in determining net income but should be reported in OCI. A cumulative foreign exchange translation loss would be reported in accumulated OCI as a stockholders’ equity contra account.

How well did you know this?
1
Not at all
2
3
4
5
Perfectly
107
Q

Slate Co. and Talse Co. exchanged similar plots of land with fair values in excess of carrying amounts. In addition, Slate received cash from Talse to compensate for the difference in land values. As a result of this exchange with commercial substance, Slate should recognize
A
A gain equal to the difference between the fair value and the carrying amount of the land given up.
B
A gain in an amount determined by the ratio of cash received to total consideration.
C
A loss in an amount determined by the ratio of cash received to total consideration.
D
Neither a gain nor a loss.

A

Explanation:
In general, accounting for nonmonetary transactions should be based on the fair values of the assets involved. The acquisition is recorded at the fair value of the asset surrenedred of the FV of the asset received, whichever is more clearly determinable, and gains or losses should be recognized. The amount would be the difference between the fair value received and the carrying value of the consideration given up.

How well did you know this?
1
Not at all
2
3
4
5
Perfectly
108
Q

During the year, Pitt Corp. incurred costs to develop and produce a routine, low-risk computer software product, as follows:
Completion of detail program design $13,000
Costs incurred for coding and testing to establish technological feasibility 10,000
Other coding costs after establishment of technological feasibility 24,000
Other testing costs after establishment of technological feasibility 20,000
Costs of producing product masters for training materials 15,000
Duplication of computer software and training materials from product masters (1,000 units) 25,000
Packaging product (500 units) 9,000In Pitt’s December 31, balance sheet, what amount should be capitalized as software cost, subject to amortization?
A
$54,000
B
$57,000
C
$59,000
D
$69,000

A

Explanation:
Costs of producing product masters incurred subsequent to establishing technological feasibility should be capitalized. These costs include coding and testing performed subsequent to establishing technological feasibility. Thus, the costs of producing product masters for training materials, the coding costs, and testing costs incurred after establishment of technological feasibility should be capitalized ($15,000 + $24,000 + $20,000 = $59,000).

How well did you know this?
1
Not at all
2
3
4
5
Perfectly
109
Q

The primary criteria for determining a fair value hedge includes the fact that the hedged item does which of the following?

A
The hedged item is specifically identified as either all or a specific portion of a recognized asset or liability or of an unrecognized firm commitment.
B
All answer choices are correct.
C
The hedged item is a single asset or liability (or a specific portion thereof) or is a portfolio of similar assets or a portfolio of similar liabilities.
D
The hedged item presents an exposure to changes in fair value attributable to the hedged risk that could affect reported earnings.

A

Explanation:
The correct answer is (B).

An asset or a liability is eligible for designation as a hedged item in a fair value hedge if all of the following additional criteria are met:

The hedged item is specifically identified as either all or a specific portion of a recognized asset or liability or of an unrecognized firm commitment.
The hedged item is a single asset or liability (or a specific portion thereof) or is a portfolio of similar assets or a portfolio of similar liabilities.
The hedged item presents an exposure to changes in fair value attributable to the hedged risk that could affect reported earnings.
For determining a fair value hedge includes the fact that the hedged item does all of the above.

How well did you know this?
1
Not at all
2
3
4
5
Perfectly
110
Q

On January 1 of the current year, Wren Co. leased a building to Brill under an operating lease for ten years at $50,000 per year, payable the first day of each lease year. Wren paid $15,000 to a real estate broker as a finder’s fee. The building is depreciated $12,000 per year. For the year, Wren incurred insurance and property tax expense totaling $9,000. Wren’s net rental income for the year should be

A
$27,500
B
$29,000
C
$35,000
D
$36,500
A

Explanation:
The correct answer is (A).

Annual rental payment $50,000
Less: Depreciation $12,000
Executory costs (insurance and property taxes) 9,000
Amortization of initial direct costs ($15,000 / 10 years) 1,500 (22,500)
Net rental income $27,500
Option (B) is incorrect because $1,500 amortized finders’ fee is not deducted.

Option (C) is incorrect because the entire finders’ fee is wrongly deducted and no deduction is made to depreciation and executory costs.

Option (D) is incorrect because executory costs are not deducted.

How well did you know this?
1
Not at all
2
3
4
5
Perfectly
111
Q

Thyme, Inc. owns 16,000 of Sage Co.’s 20,000 outstanding common shares. The carrying value of Sage Co’s equity is $500,000. Sage subsequently issues an additional 5,000 previously unissued shares for $200,000 to an outside party that is unrelated to either Thyme or Sage. What is the total non-controlling interest after the additional shares are issued?

A
$140,000
B
$172,000
C
$252,000
D
$300,000
A

Explanation:
The correct answer is (C).

Non-controlling Interest = 9,000 / 25,000 = 36%
Total equity of Sage = $700,000 (500,000 + 200,000).
Value of Non-Controlling Interest = Non-Controlling Interest x Total equity of Sage = $700,000 x 36% = $252,000.
Note: Non-controlling interest is the portion of the subsidiary’s equity held by minority shareholders who do not have control over the company. Thyme owns 16,000 of the 20,000 outstanding shares. The remaining 4,000 shareholders form the minority. Out of the new issue, all the 5,000 shares are issued to an outside party unrelated to Thyme or Sage. They also form the minority. In Sage Co. total 9,000 shareholders (i.e. 4,000 shareholders of previously outstanding shares plus the 5,000 new shareholders) constitute the non-controlling interest in the company.

How well did you know this?
1
Not at all
2
3
4
5
Perfectly
112
Q
Gains from remeasuring a foreign subsidiary's financial statements from the local currency, which is not the functional currency, into the parent company's currency should be reported as a(an)
A
Deferred foreign exchange gain.
B
Item of other comprehensive income.
C
Extraordinary item, net of income taxes.
D
Part of continuing operations.
A

Explanation:
If an entity does not maintain its books in its functional currency, remeasuring into the functional currency is required prior to translation into the reporting currency (i.e., the parent company’s currency). Exchange gains and losses that result for the remeasuring process are recognized in income from continuing operations.

How well did you know this?
1
Not at all
2
3
4
5
Perfectly
113
Q
A(n) \_\_\_\_\_\_\_\_\_\_\_\_\_\_\_\_\_ is a risk management strategy to protect against the possibility of loss, such as from price fluctuations.
A
Hedge
B
Underlying
C
Forecasted transaction
D
Firm commitment
A

Explanation:

Hedging is a risk management strategy to protect against the possibility of loss, such as from price fluctuations.

How well did you know this?
1
Not at all
2
3
4
5
Perfectly
114
Q

On January 1, year 1, a company with a calendar year end began developing a software program that it intends to market and sell to its customers. The software coding was completed on March 31, year 1, at a cost of $200,000, and the software testing was completed on June 30, year 1, at a cost of $100,000. The company achieved technological feasibility on July 31, year 1, at which time the company began producing product masters at a cost of $125,000. What amount should the company report for the total research and development expense for the year ended December 31, year 1?

A
$100,000
B
$200,000
C
$300,000
D
$425,000
A

Explanation:
The company was developing a software program for sale. Following is how expenses incurred at various stages of developing such a computer software,whether to sell, lease or market, should be treated:Expense all the costs incurred until the point where technological feasibility is established as research and development cost.Capitalize all costs incurred to convert a technologically feasible program into a final commercial product.

Expense as cost of goods sold (COGS) costs incurred after software sale begins.The company achieved technological feasibility on July 31, year 1. All costs incurred before this date will be expensed as research and development cost. Total research and development cost = $200,000 (software coding cost incurred till March 31, year1) + $100,000 (software testing costs incurred till June 30, year 1) = $300,000.

Option (a) is incorrect because it treats only $100,000 software testing costs incurred till June 30, year 1 as research and development costs and fails to include$200,000 software coding cost incurred till March 31, year 1 as a part of research and development cost. All costs incurred until technological feasibility has been established has to be expensed as R&D.

Option (b) is incorrect because it treats only $200,000 software coding cost incurred till March 31, year 1 as research and development costs and fails to include$100,000 software testing costs incurred till June 30, year 1 as a part of research and development cost.

Option (d) is incorrect because besides $300,000 (i.e. $200,000 + 100,000) it treat seven the cost of $125,000 incurred after the company achieved technological feasibility as research and development cost. This is not a R&D expense. This has to be capitalized.

How well did you know this?
1
Not at all
2
3
4
5
Perfectly
115
Q

Glade Co. leases computer equipment to customers under direct-financing leases. The equipment has no residual value at the end of the lease, and the leases do not contain purchase options. Glade wishes to earn 8% interest on a 5-year lease of equipment with a fair value of $323,400. The present value of an annuity due of $1 at 8% for five years is 4.312. What is the total amount of interest revenue that Glade will earn over the life of the lease?

A
$ 51,600
B
$ 75,000
C
$129,360
D
$139,450
A

Explanation:
The correct answer is (A).

The fair value of the equipment is also used as the present value.

The present value factor of an annuity due of $1 at 8% for 5 years is given as 4.312, the formula would be 4.312 x annual payment = $323,400 or annual payment = $323,400 / 4.312 = $75,000.

Glade will receive a total of 5 payments at $75,000 each or a total of $375,000.

The present value of the lease payments is $323,400, the difference of $51,600 (i.e. $375,000 - $323,400) is the interest that will be earned over the life of the lease.

No gain or loss is recognized under a direct-financing lease.

Option (B) is incorrect because this represents the annual payment over the life of the lease.

Options (C) and (D) are incorrect due to inaccurate calculations.

How well did you know this?
1
Not at all
2
3
4
5
Perfectly
116
Q

According to ASC 842, lease payments include all of the following except:

A
Guarantee of the Lessor’s debt
B
Residual Value Guarantees
C
Fees Paid to Owners of Special-Purpose Entities
D
Variable Lease Payments
A

Explanation:
The correct answer is (A).

Per ASC 842, Lease Payments include:

Fixed Lease Payments
Variable Lease Payments
Renewal, Purchase, and Termination Option Payments
Fees Paid to Owners of Special-Purpose Entities
Residual Value Guarantees
Lease Payments don’t include:

Payments for variable leases not dependent on an index or rate
Guarantee of the Lessor’s debt

How well did you know this?
1
Not at all
2
3
4
5
Perfectly
117
Q

For a Finance Lease, the amount recorded initially by the lessee as a liability should normally

A
Exceed the total of the minimum lease payments.
B
Exceed the present value of the minimum lease payments at the beginning of the lease.
C
Equal the present value of the minimum lease payments + salvage value
D
Equal the present value of the lease payments not yet paid at the beginning of the lease.

A

Explanation:
The lessee should normally record a Finance Lease as an ROU (Asset) and Lease Liability at an amount equal to the present value of the lease payments not yet paid at the beginning of the lease term.

How well did you know this?
1
Not at all
2
3
4
5
Perfectly
118
Q

Which of the following methods should a company use to account for a contingent liability when the loss is probable but not reasonably estimated?

A
The liability should not be reported
B
The liability should be reported as a short-term liability
C
The liability should be reported as a long-term liability
D
The liability should only be disclosed in the notes to the financial statements

A

Explanation:
The correct answer is (D).

Probability of loss Disclose in notes to Financial Statements Accrue
Remote No No
Reasonably possible Yes No
Probable and estimable Yes Yes
Probable but not estimable Yes No
A contingent liability, when a loss is probable but cannot be reasonably estimated, should only be disclosed in the notes to financial statements.

How well did you know this?
1
Not at all
2
3
4
5
Perfectly
119
Q
Kent Co. filed a voluntary bankruptcy petition, and the statement of affairs reflects the following amounts:
Book value	Estimated current value
Assets:		
Assets pledged with fully secured creditors	$ 300,000	$370,000
Assets pledged with partially secured creditors	180,000	120,000
Free assets	420,000	320,000
$ 900,000	$810,000
Liabilities (book value):		
Liabilities with priority	70,000	
Fully secured creditors	260,000	
Partially secured creditors	200,000	
Unsecured creditors	540,000	
$1,070,000	Assume that the assets are converted to cash at the estimated current values and the business is liquidated. What amount of cash will be available to pay unsecured nonpriority claims?
A
$240,000
B
$280,000
C
$320,000
D
$360,000
A

Explanation:
Secured creditors are paid first with the proceeds from the sale of specific assets upon which they have liens. Any excess proceeds from such sales are first applied against the liabilities with priority, and then to the unsecured creditors. If the claims of partially secured creditors exceed the proceeds from the sale of the assets pledged with such creditors, such excess constitutes an unsecured claim.

Total cash available $810,000
Payments to fully secured creditors $260,000
Payments to partially secured creditors 120,000
Payments to creditors with priority 70,000 (450,000)
Cash to pay unsecured nonpriority claims $360,000

How well did you know this?
1
Not at all
2
3
4
5
Perfectly
120
Q

In sales-type leases, the lessor must disclose which of the following?

A
The cost and carrying amount, if different, of property leased or held for leasing, by major class and total accumulated depreciation
B
A general description of leasing arrangements
C
Net investment components
D
All of the above
A
Explanation:
In sales-type and direct financing leases, the lessor must disclose the net investment components, including: future MLP; unguaranteed residual value; unearned income; and the future MLP to be received in each of the succeeding 5 years. For operating leases, the lessor must disclose: the cost and carrying amount, if different, of property leased or held for leasing, by major class and total accumulated depreciation; the minimum future rentals on noncancelable leases, in aggregate, for each of the next 5 years; and a general description of leasing arrangements.
How well did you know this?
1
Not at all
2
3
4
5
Perfectly
121
Q
A corporation is in the final stages of developing a computer software program that will be sold to the general public. The company’s costs related to the software are as follows:
Development of a working model of the software	$4 million
Customer support and training	2 million
Product master production	1 millionThe costs associated with the product master production were incurred after the establishment of technological feasibility. What amount, if any, should the corporation expense against earnings?
A
$6 million
B
$5 million
C
$4 million
D
$0
A

Explanation:
Costs incurred internally in creating a computer software product to be sold, leased, or otherwise marketed as a separate product, or as a part of a product or process, are charged to expense when incurred as research and development until technological feasibility has been established for the product. The costs of producing product masters incurred subsequent to establishing technological feasibility are capitalized. The corporation should expense $6 million; the $4 million development of a working model of the software and the $2 million customer support and training that was incurred. Only the $1 million for product master production would be capitalized.

How well did you know this?
1
Not at all
2
3
4
5
Perfectly
122
Q

In Year 7, Spirit, Inc. determined that the 12-year estimated useful life of a machine purchased for $48,000 in January Year 2 should be extended by three years. The machine is being depreciated using the straight-line method and has no salvage value. What amount of depreciation expense should Spirit report in its financial statements for the year ending December 31, Year 7?

A
$2,800
B
$3,200
C
$4,200
D
$4,800
A

Explanation:
Changing the estimated useful life of a machine is considered a change in accounting estimate and accounted for in the current and subsequent periods. The machine had been depreciated a total of $20,000 thus far, 5 years at $4,000 per year ($48,000 / 12 years straight-line = $4,000 per year). The original amount of $48,000 less the $20,000 depreciated so far equals $28,000 worth of useful life. Extending the useful life three years means it now has 10 years of useful life left (12 years originally less the 5 years gone by plus 3 more years). So the depreciation expense is $2,800 ($28,000 / 10) per year.

How well did you know this?
1
Not at all
2
3
4
5
Perfectly
123
Q

Which of the following is not a part of Lease Payments?

A
Variable lease payments dependent on a rate or index
B
Payments associated with renewal or termination options or the exercise of a purchase option
C
Variable lease payments not dependent on an index or a rate
D
Fees Paid by the Lessee to Owners of Special-Purpose Entities

A

Explanation:
Lease payments include:

Fixed Lease Payments and in substance fixed payments
Variable lease payments dependent on a rate or index
Payments associated with renewal or termination options or the exercise of a purchase option
Fees Paid by the Lessee to Owners of Special-Purpose Entities
Residual Value Guarantee
Lease payments exclude:

Variable lease payments not dependent on an index or a rate
Guarantee of lessors debt

How well did you know this?
1
Not at all
2
3
4
5
Perfectly
124
Q

A collection agency spent $50,000 in staff payroll costs investigating the feasibility of developing its own software program for tracking customer contacts. After committing to funding the project, software developers were paid $200,000 to write the code, and the company incurred $70,000 in general and administrative costs related to training and software maintenance. What amount should be capitalized?

A
$200,000
B
$250,000
C
$270,000
D
$320,000
A

Explanation:
The collection agency wanted to create a computer software for tracking its customer contacts for its internal use. Following is how expenses incurred at various stages of developing such a computer software for internal use should be treated:

Expense all the costs incurred in the preliminary project stage like those relatedto coding and testing, designing, investing feasibility, vendor selection etc. as research and development cost.
Capitalize all costs incurred in the development stage until the software is substantially complete and ready for its intended use.
Capitalize any upgrades and enhancements.
Expense costs incurred in training, data conversion and maintenance as period costs.
Amortize capitalized costs by using straight line method.
$50,000 staff payroll costs incurred for investigating the feasibility of developing its own software program is a cost incurred in the preliminary project stage and will be expensed as research and development cost. $70,000 general and administrative costs related to training and software maintenance will be expensed as period cost.Only $200,000 paid for writing the code will be capitalized as cost for developing the computer software. Options (b), (c) and (d) are incorrect because both $50,000 staff payroll costs and$70,000 general and administrative costs should be expensed and not capitalized.

How well did you know this?
1
Not at all
2
3
4
5
Perfectly
125
Q
Madden Company owns a tract of land which it purchased four years ago for $100,000. The land is held as a future plant site and has a fair market value of $140,000 on July 1 of the current year. Hall Company also owns a tract of land held as a future plant site. On this date, Madden exchanged its land and paid $50,000 cash for the land owned by Hall. Assuming there was commercial substance, at what amount should Madden record the land acquired in the exchange?
A
$150,000
B
$160,000
C
$190,000
D
$200,000
A

Explanation:
In an exchange with commercial substance, the transaction is accounted for at the fair value of the asset received or the asset given up, whichever is more clearly evident, and a gain or loss is recognized on the exchange. Fair value of consideration given up was $140,000 for the land and $50,000 cash = $190,000.

How well did you know this?
1
Not at all
2
3
4
5
Perfectly
126
Q

Compared to its year 2 cash basis net income, Potoma Co.’s year 2 accrual basis net income increased when it
A
Declared a cash dividend in year 1 that it paid in year 2.
B
Wrote off more accounts receivable balances than it reported as uncollectible accounts expense in year 2.
C
Had lower accrued expenses on December 31, year 2, than on January 1, year 2.
D
Sold used equipment for cash at a gain in year 2

A

Explanation:
Potoma’s accrued expenses decreased during year 2. Hence, Potoma’s year 2 payments for expenses exceeded the amount of expense recognized on the accrual basis in year 2. The increased amount of expenses recognized in year 2 under the cash basis increases Potoma’s year 2 accrual basis net income as compared to its year 2 cash basis net income. The declaration or payment of a cash dividend does not affect net income computed under either the cash or accrual basis. Compared to its year 2 cash basis net income, Potoma’s year 2 accrual basis net income decreased when it recognized uncollectible accounts expense in year 2. Potoma’s year 2 cash basis net income is not affected by either the accounts receivable balances written off in year 2 or the uncollectible account expense recognized in year 2. The sale of the used equipment at a gain increases net income under both the cash and accrual basis by equal amounts.

How well did you know this?
1
Not at all
2
3
4
5
Perfectly
127
Q
Combined statements may be used to present the results of operations of
Unconsolidated
subsidiaries	Companies under
common management
A	Yes	Yes
B	Yes	No
C	No	Yes
D	No	No
A

Explanation:
Combined statements would be used to present the financial position and results of operations of a group of unconsolidated subsidiaries. They might also be used to combine the financial statements of companies under common management.

How well did you know this?
1
Not at all
2
3
4
5
Perfectly
128
Q

On January 1 of the current year, Nori Mining Co. (lessee) entered into a 5-year lease for drilling equip­ment. Nori accounted for the acquisition as a finance lease for $240,000, which includes a $10,000 option. At the end of the lease, Nori expects to exercise the purchase option. Nori esti­mates that the equipment’s fair value will be $20,000 at the end of its 8-year life. Nori regularly uses straight-line depreciation on similar equipment. For the current year ended December 31, what amount should Nori recognize as depreciation expense on the leased ROU asset?

A
$48,000
B
$46,000
C
$30,000
D
$27,500
A

Explanation:
The correct answer is (D).

A Finance Lease must meet one of the following criteria:

Present Value equals or exceeds substantially all (90%) of the Fair Value
Option to Purchase (exercise is reasonably certain)
Economic Life - Major part (75%) of the asset’s economic life is used
Transfer of Ownership at lease termination
Specialized Nature - No alternative use to the lessor at lease termination
Note: the implementation guidance for ASC 842 uses the 75/90 thresholds, even though the standard is principles-based.

If there is a transfer of ownership or a bargain purchase option at the end of a finance lease, the lessee would depreciate the leased equipment over the useful life of the asset. Since the lessee had an option that was reasonably certain of being exercised, the asset should be depreciated over its estimated useful life, instead of the lease term, because the lessee will obtain ownership of the asset.

The drilling equipment has a useful life of 8 years.

Depreciation on straight-line basis = (Asset value - Salvage value) / Useful life = ($240,000 - $20,000) / 8 = $27,500.

How well did you know this?
1
Not at all
2
3
4
5
Perfectly
129
Q

Yellow Co. spent $12,000,000 during the current year developing its new software package. Of this amount, $4,000,000 was spent before it was at the application development stage and the package was only to be used internally. The package was completed during the year and is expected to have a four year useful life. Yellow has a policy of taking a full-year’s amortization in the first year. After the development stage, $50,000 was spent on training employees to use the program. What amount should Yellow report as an expense for the current year?

A
$1,600,000
B
$2,000,000
C
$6,012,500
D
$6,050,000
A

Explanation:
Internal use computer software costs that are incurred in the preliminary project stage should be expensed as incurred. Most costs incurred in the application development stage are capitalized and should not cease until the software project is substantially complete and ready for its intended use. Training costs and data conversion costs are generally expensed in the application development stage. In the post-implementation/operation stage, training and maintenance costs should be expensed as incurred while the costs of upgrades or enhancements are capitalized. The annual amortization cost is calculated by taking the remaining $8,000,000 (12,000,000 less the $4,000,000 spent) and dividing by the four-year useful life.

Preliminary project costs	$4,000,000
Post development training costs	50,000
Year 1 amortization expense	2,000,000
Current year expense	6,050,000
Option (a) is incorrect as per the above explanation.Option (b) is incorrect because it excludes preliminary project stage costs expensed as R&D and training costs as period expense. $2,000,000 are amortization expense for the year ($8,000,000/4). Option (c) is incorrect because it capitalized the training costs instead of expensing it. [$6,012,500 = $4,000,000 + ($8,050,000/4)](ID: 8104)
How well did you know this?
1
Not at all
2
3
4
5
Perfectly
130
Q

Which of the following is the market with the greatest volume or level of activity for an orderly transac­tion to occur for an asset or liability?

A
Perfect market
B
Principal market
C
Financial market
D
Most advantageous market
A

Explanation:
The principle market is the market with the greatest volume or level of activity for an orderly trans­action to occur for an asset or liability. The price in the principle market will be the fair value measurement, even if there is a more advantageous price in a different market. A perfect market would have perfect information and there is perfect competition. A financial market is where entities can easily buy and sell financial securities. Accounting standards make no reference to perfect or financial markets. A most advantageous market is the market with the price that maximizes the amount that would be received for an asset or minimizes the amount that would be paid to transfer a liability.

How well did you know this?
1
Not at all
2
3
4
5
Perfectly
131
Q

An entity may designate, as a type of hedge of foreign currency exposure, a derivative instrument or a non derivative financial instrument that may give rise to a foreign currency transaction gain or loss as which of the following?

A
A fair value hedge
B
A cash flow hedge
C
A hedge of a net investment in a foreign operation
D
None of the above
A

Explanation:
An entity may designate a derivative instrument or a non derivative financial instrument that may give rise to a foreign currency transaction gain or loss as a hedge of the foreign currency exposure of a net invest­ment in a foreign operation.

How well did you know this?
1
Not at all
2
3
4
5
Perfectly
132
Q

On January 1, year 5, Poe Construction Inc., changed to an adjusted version of the percentage-of-completion method in accordance with the input method prescribed in ASC 606 and recognized revenue “over time” instead of at a “point in time” as before. Poe can justify this change in accounting principle.

As of December 31, year 4, Poe compiled data showing that income under the previous revenue recognition method (recognized at a point in time), aggregated $700,000. If the new method had been used, the accumulated income through December 31, year 4, would have been $880,000. Assuming an income tax rate of 40% for all years, the cumulative effect of this accounting change should be reported by Poe in the year 5

A
Retained earnings statement as a $180,000 credit adjustment to the beginning balance.
B
Income statement as a $180,000 credit.
C
Retained earnings statement as a $108,000 credit adjustment to the beginning balance.
D
Income statement as a $108,000 credit.
A

Explanation:
This type of change in accounting principle is accounted for as an adjustment to the beginning balance of retained earnings, net of its income tax effect.

Revenue recognized over time $ 880,000
Revenue recognized at a point in time (700,000)
Increase in income under new method 180,000
Less: Income tax effect ($180,000 × 40%) (72,000)
Credit to 1/1, year 5, retained earnings $ 108,000

How well did you know this?
1
Not at all
2
3
4
5
Perfectly
133
Q

Jay’s lease payments are made at the end of each period. Jay’s liability for a Finance Lease would be reduced periodically by the

A
lease payment less the portion of the Lease payment allocable to interest.
B
lease payment plus the amortization of the related asset.
C
lease payment less the amortization of the related asset.
D
lease payment.

A

Explanation:
In a Finance Lease, each lease payment should be allocated between a reduction of the Finance Lease liability and Interest expense. Therefore, the lessee’s balance sheet liability for a Finance Lease should be periodically reduced by the Lease payment less the portion of the Lease payment allocable to interest.

How well did you know this?
1
Not at all
2
3
4
5
Perfectly
134
Q

Cody Corp. incurred the following costs during the year:
Design of tools, jigs, molds, and dies involving new technology $125,000
Modification of the formulation of a process 160,000
Trouble-shooting in connection with breakdowns during commercial production 100,000
Adaptation of an existing capability to a particular customer’s need as part of a continuing commercial activity 110,000In its year-end income statement, Cody should report research and development expense of
A
$125,000.
B
$160,000.
C
$235,000.
D
$285,000.

A

Explanation:
The design of tools, jigs, molds, and dies involving new technology ($125,000) and the modification of the formulation of a process ($160,000) are activities that typically would be included in research and development expense. Troubleshooting in connection with breakdowns during commercial production and the adaptation of an existing capability to a particular customer’s need as part of a continuing commercial activity typically are excluded from research and development.

How well did you know this?
1
Not at all
2
3
4
5
Perfectly
135
Q

Under what circumstances are disclosures required for subsequent events?

A
Only when the financial statements include notes
B
Only when financial statements are compiled
C
Only when financial statements are reviewed
D
Always

A

Explanation:
The disclosure requirements associated with subsequent events, along with other issues, only need to be included in notes to financial statements when the
financial statements include notes.

How well did you know this?
1
Not at all
2
3
4
5
Perfectly
136
Q

Grey Co. purchased stock in Cherry Co. Grey purchased a put option on the stock. The strike price is the current market price. What is the most likely reason Grey purchased the put option?

A
Cherry stock has remained flat, and Grey believes the stock is going to remain at its original purchase price.
B
Cherry stock has increased in price, and Grey believes the stock is going to continue to increase in price.
C
Cherry stock has decreased in price, but Grey believes the stock is going to increase in price.
D
Cherry stock has increased in price, but Grey is concerned that the price might decrease.

A

Explanation:
The correct answer is (D)

Buying a put option confers the right on the holder of the option to sell the stock at a price, regardless of the trading price of the underlying asset/stock. In essence, the put option buyer buys the right to sell the underlying stock to the put option holder at a predetermined rate.

Hence, put options are purchased mainly when the buyer is bearish and aims at hedging itself of price falls in the future.Since Grey Co. has purchased a put option on the Cherry stocks, it means that the company is concerned that the price might decrease.

How well did you know this?
1
Not at all
2
3
4
5
Perfectly
137
Q

During the year, Pitt Corp. incurred costs to develop and produce a routine, low-risk computer software product, as follows:

Design of tools, jigs, molds, and dies involving new technology $125,000
Completion of detail program design 13,000
Costs incurred for coding and testing to establish technological feasibility 10,000
Other coding costs after establishment of technological feasibility 24,000
Other testing costs after establishment of technological feasibility 20,000
Costs of producing product masters for training materials 15,000
Duplication of computer software and training materials
From product masters (1,000 units) 25,000
Packaging product (500 units) 9,000
In Pitt’s December 31 Balance Sheet, what amount should be capitalized as software costs, subject to amor­tization?

A
$54,000
B
$57,000
C
$59,000
D
$69,000
A

Explanation:
The correct answer is (C).

For computer software developed to sell, lease or market as a product:

Costs associated with converting a technologically feasible program into a final commercial product are capitalized. Other coding costs after the establishment of technological feasibility of $24,000, other testing costs after the establishment of technological materials of $20,000, and costs of producing product masters for training materials of $15,000 are all Capitalized.

($20,000+24,000+15,000 = $ 59,000)

Research and Development (R&D) costs are expensed. These are costs incurred prior to technological feasibility, which is established upon completion of a detailed program or design or completion of a working model). The completion of a detailed program design for $13,000 and costs incurred for coding and testing to establish technological feasibility of $10,000 are expensed.

Costs incurred after software sales begin are Inventoried – this will include the duplication of computer software and training materials from product masters for $25,000 and packaging products for $9,000. The year-end balance sheet would report an inventory of $34,000 (i.e. $25,000 + $9,000).

How well did you know this?
1
Not at all
2
3
4
5
Perfectly
138
Q

Troop Co. frequently borrows from the bank to maintain sufficient operating cash. The following loans were at a 12% interest rate, with interest payable at maturity. Troop repaid each loan on its scheduled maturity date.

Date of Loan Amount Maturity date Term of loan
11/1, Yr 5 $10,000 10/31, Yr 6 1 year
2/1, Yr 6 30,000 7/31, Yr 6 6 months
5/1, Yr 6 16,000 1/31, Yr 7 9 months
Troop records interest expense when the loans are repaid, thus interest expense of $3,000 was recorded in year 6. If no correction is made, by what amount would year 6 interest expense be understated?

A
$1,080
B
$1,240
C
$1,280
D
$1,440
A

Explanation:
Interest expense should be accrued in the period in which it is earned, rather than the period in which it is paid. In year 6, the total interest expense of $4,080 should be reported, as shown below. If only $3,000 interest expense is recorded, the understatement is $1,080.

Loan date Amount Monthly Interest Months in Year 6 Year 6 Interest
11/01, year 5 to 10/31, year 6 $10,000 $100* 10 $1,000
02/01, year 6 to 07/31, year 6 30,000 300 6 1,800
05/01, year 6 to 01/31, year 7 16,000 160 8 1,280
Total interest $4,080
Less interest expense recorded (3,000)
Understated interest $1,080
Note:

*Loan was taken on November 1 Year 5 and repaid on Year October 31 Year 6. That implies this loan was outstanding in Y6 for 10 Months. Interest Accrued for the same should be $10,000 x 12% x 10/12 = $1,000. Since Monthly Interest is calculated, it might be causing the confusion. As for the entire year interest on $10,000 at 12% is 1200. So, for each month, it is $100

How well did you know this?
1
Not at all
2
3
4
5
Perfectly
139
Q

Which of the following items would be classified as a research and development cost?

A
Periodic design changes to an existing product.
B
Engineering follow-up in an early phase of commercial production.
C
Testing in search of product or process alternatives.
D
Legal work in connection with a patent application.

A

Explanation:
Research activities are those aimed at the discovery of knowledge that will be useful in developing or significantly improving products or processes. Development activities are those concerned with translating research findings and other knowledge into plans or designs for new or significantly improved products or pro­cesses. Testing in search of product or process alternatives is classified as a research and development cost. Periodic design changes to an existing product, engineering follow-up in an early phase of commercial produc­tion, and legal work in connection with a patter application are not classified as research and development costs.Option (a) and (b) are incorrect because commercial production/activity or seasonal,routine or troubleshooting activities in production stage are not treated as R&D costs.Option (d) is incorrect because legal work related to patent is not a R&D cost.

How well did you know this?
1
Not at all
2
3
4
5
Perfectly
140
Q

Which of the following describes the lessee’s incremental borrowing rate?

A
The discount rate the lessee would pay to borrow on a collateralized basis over a similar term an amount equal to the lease payments in a similar economic environment
B
The rate of change for the fair value of the leased property from the start of the lease to the end of the lease term
C
The interest rate that will discount the minimum lease payments plus unguaranteed residual value to the fair value of the leased property at the lease inception date
D
None of the above

A

Explanation:
The lessee incremental borrowing rate is the rate of interest that lessee would have to pay to borrow on a collateralized basis over a similar term an amount equal to the lease payments in a similar economic environment.

Note: If the lessor’s implicit rate is known, always use that rate instead.

How well did you know this?
1
Not at all
2
3
4
5
Perfectly
141
Q

Which of the following is not included in a “lease term”?

A
All periods, representing renewal periods “reasonably certain” of exercise
B
All periods representing time covered by a termination option reasonably certain not to be exercised
C
All periods representing renewal periods (non-termination periods) controlled by lessor
D
All periods beyond the date at which a bargain purchase option becomes exercisable

A

Explanation:
The lease term includes all periods, if any, covered by renewal periods “reasonably certain” of exercise , periods covered by a termination option reasonably certain not to be exercised and renewal periods (non-termination periods) controlled by lessor. In no case should the lease term extend beyond the date at which a bargain purchase option becomes exercisable.

How well did you know this?
1
Not at all
2
3
4
5
Perfectly
142
Q

A company’s research department incurred $1,000,000 in material, labor, and overhead costs to construct a prototype of a new product and $100,000 to test and modify the prototype. Which of the following statements correctly describes the accounting treatment of prototype costs incurred by the company?

A
Capitalize $1,100,000 and amortize it over the expected sales life of the new product
B
Capitalize $1,100,000 and amortize it over the life of the prototype
C
Capitalize $1,000,000 and amortize it over the life of the prototype and expense $100,000 as incurred
D
Expense $1,100,000 as incurred

A

Explanation:
The correct answer is (D).

Costs incurred for research and development (R&D) are expensed as incurred. The R&D costs include:

New knowledge or new technology.
Model or prototype.
Application of new research findings.
In the current year, the company will recognize the R&D expenses of $1,100,000 which would include material, labor, overhead, modification and testing costs.

How well did you know this?
1
Not at all
2
3
4
5
Perfectly
143
Q

Pear Co.’s income statement for the current year ended December 31, as prepared by Pear’s controller, reported income before taxes of $125,000. The auditor questioned the following amounts that had been included in income before taxes:

Equity in earnings of Cinn Co. $40,000
Dividends received from Cinn 8,000
Adjustments to profits of prior years for arithmetical errors in depreciation (35,000)
Pear owns 40% of Cinn’s common stock. Pear’s December 31 income statement should report income before taxes of

A
$ 85,000
B
$117,000
C
$120,000
D
$152,000
A

Explanation:
Since Pear owns 40% of Cinn’s common stock, Pear has the ability to exercise significant influence over Cinn by virtue of its investment and should account for its investment in Cinn by the equity method. There­fore, Pear’s $40,000 equity in Cinn’s earnings is properly included in Pear’s current year income before taxes. Under the equity method, the dividends received from Cinn reduce the carrying amount of the investment; they do not affect the amount of investment income that Pear recognizes. So, the $8,000 of dividends received from Cinn is erroneously included in current year income, before taxes are subtracted to correct that figure. The arith­metical errors in depreciation of prior years represents a correction of errors of prior periods. The correction of the errors should be reported as a prior-period adjustment by restating the prior-period financial statements. So, the $35,000 of arithmetical errors in depreciation of prior years that Pear had inadvertently subtracted from cur­rent year income before taxes are added back to correct that figure.

Income before taxes, before adjustment $125,000
Less: Dividends received from equity method investee (8,000)
Add: Arithmetical errors in depreciation of prior years _ 35,000
Corrected income before taxes $152,000

How well did you know this?
1
Not at all
2
3
4
5
Perfectly
144
Q

Which of the following is not considered in evaluating the highest and best use of an asset by market participants at the measurement date?

A
Physically possible
B
Legally permissable
C
Readily accessible
D
Financially feasible
A

Explanation:
An asset being readily accessible is not considered in evaluating an asset for fair value measure­ment. The highest and best use of an asset establishes the valuation premise used to measure the fair value of an asset. The highest and best use of the asset is applied considering the use of the asset that is physically possible, legally permissible, and financially feasible at the measurement date.

How well did you know this?
1
Not at all
2
3
4
5
Perfectly
145
Q

An entity is required to account for each business combination by applying the acquisition method. In applying the acquisition method, which of the following is not required?

A
Determining the acquisition date
B
Recognizing and measuring identifiable assets acquired, liabilities assumed, and any noncontrolling interest in the acquiree
C
Allocating the excess of fair value over purchase price as a pro rata reduction of acquired assets
D
Recognizing and measuring goodwill or a gain from a bargain purchase

A

Explanation:
Allocating the excess of fair value over purchase price as a pro rata reduction of acquired assets is not a part of applying the acquisition method. Applying the acquisition method requires (1) identifying the acquirer, (2) determining the acquisition date, (3) recognizing and measuring identifiable assets acquired, liabil­ities assumed, and (4) recognizing and measuring goodwill or a gain from a bargain purchase.

146
Q

During the current year, Orca Corp. decided to change from the FIFO method of inventory valuation to the weighted-average method. Inventory balances under each method were as follows:
FIFO Weighted-average
January 1 $71,000 $77,000
December 31 79,000 83,000Orca’s income tax rate is 30%. Orca should report the cumulative effect of this accounting change as a(an)
A
Prior period adjustment to the beginning balance of retained earnings
B
Component of income from continuing operations.
C
Extraordinary item.
D
Component of income after extraordinary items.

A

Explanation:
A change in inventory valuation method is a change in accounting principle, and the cumulative effect of the change in accounting principle should be reported as an adjustment to the beginning balance of retained earnings in the period of change. It is not reported in the income statement.

147
Q

In preparing consolidated financial statements of a U.S. parent company with a foreign subsidiary, the foreign subsidiary’s functional currency is the currency

A
In which the subsidiary maintains its accounting records.
B
Of the country in which the subsidiary is located.
C
Of the country in which the parent is located.
D
Of the environment in which the subsidiary primarily generates and expends cash.

A

Explanation:
The correct answer is (D).

Functional currency is the currency of the principal economy or country in which an entity carries on its business. The functional currency remains the same irrespective of the location of the parent or method of accounting.

148
Q

Isle Co. owned a copy machine that cost $5,000 and had accumulated depreciation of $2,000. Isle exchanged the copy machine for a computer that cost $4,000. Isle’s future cash flows are not expected to change significantly as a result of the exchange. What amount of gain or loss should Isle report and at what amount should it record the asset?
A
No gain or loss in the income statement; $3,000 asset in the balance sheet.
B
No gain or loss in the income statement; $4,000 asset in the balance sheet.
C
$1,000 gain in the income statement; $3,000 asset in the balance sheet.
D
$1,000 gain in the income statement; $4,000 asset in the balance sheet.

A

Explanation:
Both exchanges and nonreciprocal transfers that involve little or no monetary assets or liabilities are referred to as nonmonetary transactions. In general, accounting for nonmonetary transactions should be based on the fair values of the assets involved. Nonmonetary exchanges should be based on recorded amounts (rather than fair values) of the exchanged assets if any of the following conditions apply: (a) neither the fair value of the assets received nor the fair value of the assets surrendered is determinable within reasonable limits; (b) the transaction is an exchange of a product or property held for sale in the ordinary course of business for a product or property to be sold in the same line of business to facilitate sales to customers (other than the parties to the exchange); or (c) the transaction lacks commercial substance. In a nonmonetary exchange that has no commercial substance, the assets exchanged are accounted for at book value (after reduction, if appropriate, for an indicated impairment of value) of the nonmonetary asset(s) given up. Isle’s future cash flows not being expected to change significantly as a result of the exchange indicates the transaction lacks commercial substance. In a nonmonetary exchange that has no commercial substance, the assets exchanged are accounted for at book value of the nonmonetary asset(s) given up; therefore, no gains or losses are normally recognized on the exchange itself. Isle would not recognize any gain or loss in the income statement and would record the computer as a $3,000 asset in the balance sheet.

149
Q

Which of the following items would best enable Driver Co. to determine whether the fair value of its investment in Favre Corp. is properly stated in the balance sheet?
A
Discounted cash flow of Favre’s operations.
B
Quoted market prices available from a business broker for a similar asset.
C
Quoted market prices on a stock exchange for an identical asset.
D
Historical performance and return on Driver’s investment in Favre.

A

Explanation:
Fair value is a market-based (not an entity-based) measurement. Fair value is the amount at which an asset (liability) could be bought (incurred) or sold (settled) in a current transaction between willing parties other than in a forced or liquidation sale. Generally, quoted market prices, if available, are the best evidence of fair value. Quoted market prices on a stock exchange for an identical asset are Level 1 inputs to valuation (highest priority) and would be better than quoted market prices available from a business broker for a similar asset (Level 2 inputs). If quoted market prices are unavailable, the estimate of fair value should be based on the best information available in the circumstances.

150
Q

Winn Co. manufactures equipment that is sold or leased. On December 31, year 1, Winn leased equip­ment to Bart for a 5-year period ending December 31, year 6, at which date ownership of the leased asset will be transferred to Bart. Equal payments under the lease are $22,000 and are due on December 31 of each year. The first payment was made on December 31, year 1. Collectability of the remaining lease payments is reasonably assured, and Winn has no material cost uncertainties. The normal sales price of the equipment is $77,000, and cost is $60,000. On December 31, year 1, what amount of income should Winn realize from the lease transaction?

A
$17,000
B
$22,000
C
$23,000
D
$33,000
A

Explanation:
A Finance Lease must meet one of the following criteria:

Present Value equals or exceeds substantially all (90%) of the Fair Value
Option to Purchase (exercise is reasonably certain)
Economic Life - Major part (75%) of asset’s economic life is used
Transfer of Ownership at lease termination
Specialized Nature - No alternative use to the lessor at lease termination
Note: the implementation guidance for ASC 842 uses the 75/90 thresholds, even though the standard is principles-based.

At the commencement date, a lessor shall recognize. At the lease commencement date, the lessor is required to calculate the selling profit or loss as (1) the fair value of the underlying asset (or the sum of lease receivable and any prepaid lease payments by lessee, if lower); minus (2) the carrying amount of the underlying asset net of any unguaranteed residual asset; minus (3) any deferred initial direct costs of the lessor

The lease qualifies as a sales-type lease because (1) the lease transfers ownership of the property to the lessee by the end of the lease term. In year 1, the lessor should recognize a manufacturer’s profit calculated as FV off the equipment over its CV ($77,000 – $60,000 = $17,000). The lessor does not recognize any interest income from this lease in year 1 because the lease term began on December 31.

151
Q

Ball Corp. had the following foreign currency transactions during the current year:
Merchandise was purchased from a foreign supplier on January 20 for the U.S. dollar equivalent of $90,000. The invoice was paid on March 20 at the U.S. dollar equivalent of $96,000.
On July 1, Ball borrowed the U.S. dollar equivalent of $500,000 evidenced by a note that was payable in the lender’s local currency on July 1 in two years. On December 31, the U.S. dollar equivalents of the principal amount and accrued interest were $520,000 and $26,000, respectively. Interest on the note is 10% per annum.
In Ball’s year-end income statement, what amount should be included as foreign exchange loss?
A
$0
B
$ 6,000
C
$21,000
D
$27,000

A

Explanation:
The payables resulting from the merchandise purchased from the foreign supplier and the borrowing are transactions denominated in a foreign currency. The payable from the merchandise purchased was recorded at $90,000 on 1/20. It was paid on 3/20 at the U.S. dollar equivalent of $96,000, resulting in a $6,000 foreign exchange loss. The payable from the borrowing was recorded at $500,000 at 7/1. Accrued interest on the borrowing was $25,000 ($500,000 × 10% × 6/12) at 12/31. At 12/31, the U.S. dollar equivalents of principal and accrued interest were $520,000 and $26,000, respectively, resulting in an additional $21,000 [($520,000 + $26,000) - ($500,000 + $25,000)] foreign exchange loss. Thus, the total amount of foreign exchange loss to be recognized in the yearend income statement is $27,000 ($6,000 + $21,000).

152
Q
On July 1, year 1, Clark Company borrowed 1,680,000 local currency units (LCUs) from a foreign lender, evidenced by an interest bearing note due on July 1, year 2, which is denominated in the currency of the lender. The U.S. dollar equivalent of the note principal was as follows:
Date	Amount
7/1, year 1 (date borrowed)	$210,000
12/31, year 1 (Clark's year end)	240,000
7/1, year 2 (date repaid)	280,000In its income statement for year 2, what amount should Clark include as a foreign exchange gain or loss?
A
$70,000 gain
B
$70,000 loss
C
$40,000 gain
D
$40,000 loss
A

Explanation:
A transaction gain or loss (measured from the transaction date or most recent balance sheet date, whichever is later) is realized upon settlement of a foreign currency transaction. In year 2, a loss of $40,000 (i.e., $240,000 – $280,000) is recorded due to the additional amount that must be repaid since the last balance sheet date.

153
Q

Which of the following is a level three input to valuation techniques used to measure the fair value of an asset?

A
Quoted prices in active markets for identical assets
B
Quoted prices for similar assets in active markets
C
Unobservable inputs for the asset
D
Inputs other than quoted prices that are observable for the asset

A

Explanation:
The correct answer is (C).

Level One: The preferred inputs to valuation efforts are current quoted prices in active markets for identical assets or liabilities.

Level Two: This is a valuation based on market observables. The second level arises when assets and liabilities are similar to, but not the same as, those traded in an active market. In this case, the reporting company makes some assumptions about what the fair value of the reported items could be in a market.

Level Three: The FASB describes Level 3 inputs as unobservable. If inputs from levels 1 and 2 are not available, FASB acknowledges that fair value measurements could be based on unobservable inputs. Level 3 allows for situations in which there is little if any, market activity for the asset or liability at the measurement date.

Unobservable inputs for an asset is a level 3 input to valuation techniques used to measure the fair value of an asset.

154
Q

A company owns a financial asset that is actively traded on two different exchanges (market A and market B). There is no principal market for the financial asset. The information on the two exchanges is as follows:

Quoted price of asset Transaction costs

Market A $ 1,000 $ 75
Market B 1,050 150
What is the fair value of the financial asset?

A
$ 900
B
$ 925
C
$1,000
D
$1,050
A

Explanation:
Fair value is the price that would be received to sell an asset or paid to transfer a liability in an orderly transaction between market participants at the measurement date. It is assumed that the transaction would occur in the principle market, or in the absence of a principle market, the most advantageous market would be used. The most advantageous market is the market with the price that maximizes the amount that would be received for the asset or minimizes the amount that would be paid to transfer the liability. In determining the most advantageous market, transaction costs are netted against the price of the asset. The fair value price is not adjusted for transaction costs. Market A would yield a net $925 ($1,000 – $75) while Market B would yield $900 ($1,050-$150), so Market A is the most advantageous, and the fair value of the asset would be $1,000.

155
Q
Eagle Co. has cosigned the mortgage note on the home of its president, guaranteeing the indebtedness in the event that the president should default. Eagle considers the likelihood of default to be remote. How should the guarantee be treated in Eagle's financial statements?
A
Disclosed only
B
Accrued only
C
Accrued and disclose
D
Neither accrued nor disclosed
A

Explanation:
Contingent liabilities that are considered to have a remote possibility of loss normally do not require disclosure. Exceptions include the guarantees of indebtedness of others, in which case disclosure, but not accrual, is required.

156
Q

Crossroads Co. chooses to report a financial asset at its fair value. The asset trades in two different markets; however, neither market is the principal market for the financial asset. In the first market, sales proceeds are $76, which is net of transaction costs of $6. In the secondary market, the sales proceeds are $80, which is net of transaction costs of $1. What amount should Crossroads report as the fair value of the asset?

A
$76
B
$80
C
$81
D
$82
A

Explanation:
The correct answer is (C).

According to the facts of the question, the asset trades in two different markets; however, neither market is the asset’s principal market, which is the market with the greatest volume and level of activity.

If the asset’s principal market cannot be identified, the most advantageous market should be used when determining the fair value of a financial asset.

The most advantageous market would be the one which generates the highest net price, after subtracting transaction costs.

The secondary market in the question generates the highest net price of $80 after subtracting the transaction costs; therefore, this market should be used for fair value purposes.

The fair value amount will exclude any transaction costs, which results in a fair value of $81 for the asset.

Note: Transaction costs will not be incorporated into the fair value.

(A) is incorrect because $76 is the net proceeds in the first market which is not the most advantageous market when compared to the secondary market.

(B) is incorrect because $80 is the fair value after adjusting for transaction costs in the secondary market.

(D) is incorrect because first market is not the most advantageous market after considering the transaction costs (i.e. $82 = $76 + $6).

157
Q

Which of the following is the proper treatment of the cost of equipment used in research and development activities that will have alternative future uses?

A
Expensed in the year in which the research and development project started
B
Capitalized and depreciated over the term of the research and development project
C
Capitalized and depreciated over its estimated useful life
D
Either capitalized or expensed, but not both, depending on the term of the research and development project

A

Explanation:
Materials, equipment, facilities, or intangibles purchased from others that are acquired for a particu­lar R&D project and have no alternative use should be expensed in the period in which acquired.However,these items should be recorded as assets and amortized over their useful lives to R&D expense if alternative future uses are expected, whether in other R&D activities or in normal operations.If these assets are no longer deemed to have alternative future uses, the remaining unamortized cost is charged to R&D expense for the period.Option (a) is incorrect because the equipment is capitalized, not expensed.Option (b) is incorrect because the equipment is depreciated over its useful life, not the project life.Option (d) is incorrect because the equipment cost must be capitalized and depreciated over its useful life, irrespective of the term of the project.

158
Q
What is the priority level given to unobservable inputs in the fair value hierarchy for inputs to valuation techniques used to measure fair value?
A
Level 1
B
Level 2
C
Level 3
D
Level 4
A

Explanation:
The lowest priority, Level 3, is given to unobservable inputs. The highest level, Level 1, is given to quoted prices in active markets for identical assets or liabilities. Level 2 inputs are other than quoted prices included within Level 1 that are observable for the asset or liability, either directly or indirectly. There is no Level 4 in the fair value hierarchy.

159
Q

Under IFRS (IAS17, Leases), which of the following is not a criterion for a finance lease?

A
The present value of the lease payments is equal to substantially all of the fair value of the asset
B
There is a bargain purchase option
C
Cancellation losses are borne by the lessor
D
The asset is of a specialized nature, useful only to the lessee without modification

A

Explanation:
The correct answer is (C).

Cancellation losses are borne by the lessee, not the lessor, for financing leases under IFRS.

The remaining answer choices are all criteria for a finance lease under IFRS.

160
Q
A foreign subsidiary's functional currency is its local currency, which has not experienced significant inflation. The weighted average exchange rate for the current year would be the appropriate exchange rate for translating
Salaries
expense	Sales to
external customers
A	Yes	Yes
B	Yes	No
C	No	Yes
D	No	No
A

Explanation:
Foreign currency financial statements should be translated by means of the following rates: all assets and liabilities at the current exchange rate at the balance sheet date; revenues and expenses at the exchange rate at the time the revenue or expense was recognized, however, due to the impracticability of this where rates change frequently, a weighted-average exchange rate for the period may be used; contributed capital at the historical exchange rate; and retained earnings at the translated amount of retained earnings for the prior period, plus (less) net income(loss) at the weighted-average rate, less dividends declared during the period at the exchange rate when declared. The weighted average exchange rate would be an appropriate exchange rate for salaries expense and sales to external customers.

161
Q

During the current year, Beta Motor Co. incurred the following costs related to a new solar-powered car:

Salaries of laboratory employees researching how to build the new car $250,000
Legal fees for the patent application for the new car 20,000
Engineering follow-up during the early stages of commercial production (the follow-up occurred during the current year) 50,000
Marketing research to promote the new car 30,000
Design, testing, and construction of a prototype 400,000
What amount should Beta Motor report as research and development expense in its income statement for the current year?

A
$250,000
B
$650,000
C
$720,000
D
$750,000
A

Explanation:
Research activities are those aimed at the discovery of knowledge that will be useful in developing or significantly improving products or processes.Development activities are those concerned with translating research findings and other knowledge into plans or designs for new or significantly improved products or pro­cesses.Future economic benefits deriving from research and development (R&D) activities, if any, are uncertain in their amount and timing.Due to these uncertainties, most R&D costs are required to be charged to expense the year in which incurred.The major exception is that assets recorded for R&D costs with alternative future uses should be amortized over their useful lives by periodic charges to R&D expense.Another exception is that R&D costs conducted for others under contract are not expensed as R&D costs.The only items that qualify as R&D are the salaries of laboratory employees researching how to build the new car and the design, testing, and construction of a prototype.($250,000 + $400,000 = $650,000). Legal fees to obtain the patent, engineering costs in early stage production and marketing research costs are not considered part of R&D expense.Option (a) is incorrect because it fails to include design, testing and construction of a prototype expense.Option (c) is incorrect because it includes legal fees and engineering follow up expenses. It fails to include design, testing and construction of a prototype expense. ($720,000 = $250,000 + $20,000 + $50,000 + $400,000). Option (d) is incorrect because it includes legal fees, engineering follow up and market research costs in expenses. ($750,000 = $250,000 + $20,000 + $50,000 +$400,000).

162
Q

A company leases a machine from Leasing, Inc. on January 1, year 1. The lease terms include a $100,000 annual payment beginning January 1, year 1. The machine’s fair value is $500,000 and the residual value is estimated at $20,000. The company guarantees the residual value. The useful life of the machine is six years, and the lease term is five years. The implicit rate of interest is 6% and is known by the company. The following present value factors are provided:

Five years Six years

Present value of $1 at 6% 0.7473 0.7050
Present value of an annuity due at 6% 4.4651 5.2124
Present value of an ordinary annuity at 6% 4.2124 4.9173
What is the value of the machine in the company’s balance sheet at lease inception?

A
$446,510
B
$461,456
C
$520,000
D
$535,340
A

Explanation:
The correct option is (B).

A Finance Lease must meet one of the following criteria:

Present Value equals or exceeds substantially all (90%) of the Fair Value
Option to Purchase (exercise is reasonably certain)
Economic Life - Major part (75%) of asset’s economic life is used
Transfer of Ownership at lease termination
Specialized Nature - No alternative use to the lessor at lease termination
Note: the implementation guidance for ASC 842 uses the 75/90 thresholds, even though the standard is principles-based.

With a lease term equal to 5 years of the asset’s 6 years useful life, as the lease term is for the major part of the remaining economic life of the underlying asset, the lease is a finance lease and will result in the recognition of an asset and a liability equal to the present value of the minimum lease payments. Guaranteed residual value on the machine is a one-time payment at the end of the lease term calculated at the PV of $1 at 6% for 5 years.

Ref

Particulars

Amount

A Annual payments for leasing the machine $100,000
B PV of an annuity due at 6% for 5 Years 4.4651
C PV of an annual payments (A x B) $446,510
D Residual value of machine $20,000
E PV of $1 at 6% 0.7473
F PV of residual value of the machine (D x E) $14,946
G Value of machine at lease inception (C + F) $461,456
Option (A) is incorrect because it does not consider PV of the residual value at the end of the lease term of $14,946.

Option (C) is incorrect because it does not consider the PV of the annual payments and residual value.

Option (D) is incorrect because it considers the PV factors for annuity and PV of the $1 for 6 years instead of 5 years factors

163
Q

A 20-year property lease, classified as an operating lease, provides for a 10% increase in annual payments every five years. In the sixth year compared to the fifth year, the lease will cause the following expenses to increase Rent Interest

Rent	Interest
A	No	Yes
B	Yes	No
C	Yes	Yes
D	No	No
A

Explanation:
The correct answer is (D).

The effects of scheduled rent increases, which are included in minimum lease payments, should be recognized by lessors and lessees on a straight-line basis over the lease term.

The annual rent expense for this lease is constant over the 20-year term of the lease. Since the lease is an operating lease, the lessee does not report any interest expense as a result of the lease.

164
Q

ASU 2016-02 covers leases of all property, plant, and equipment, as well as

A
Assets under construction
B
Biological assets
C
Intangible assets
D
None of the above
A

Explanation:
The correct answer is (D)

According to ASU 2016-02, a Lease is a contract that gives a customer the right to control the use of an identified asset (PP&E) for a period of time in exchange for some form of consideration. Control exists if the customer has:

  1. The right to obtain substantially all of the economic benefits from the asset.
  2. The right to direct (control) the use of the asset.

It includes leases of all PP&E, but excludes leases for:

Intangible assets
Exploring for minerals, gas, oil, etc.
Biological assets
Inventory
Assets under construction
165
Q

Which of the following errors could result in an overstatement of both current assets and stockholders’ equity?
A
An understatement of accrued sales expenses.
B
Noncurrent note receivable principal is misclassified as a current asset.
C
Annual depreciation on manufacturing machinery is understated.
D
Holiday pay expense for administrative employees is misclassified as manufacturing overhead.

A

Explanation:
Since holiday pay for administrative employees is a period cost, it should have been expensed when incurred. Instead, it was misclassified and inventoried as manufacturing overhead. This error overstates both inventory and net income, thus overstating both current assets and stockholder’s equity. The understatement of an accrued expense overstates net income and stockholders’ equity and understates current liabilities. Misclassifying noncurrent note receivable principal as a current asset overstates current assets and understates noncurrent assets. The understatement of depreciation expense overstates net income, noncurrent assets, and stockholders’ equity.

166
Q

Lease M does not contain a bargain purchase option, but the lease term is equal to 90% of the estimated economic life of the leased property. Lease P does not transfer ownership of the property to the lessee at the end of the lease term, but the lease term is equal to 75% of the estimated economic life of the leased property. How should the lessee classify these leases?

Lease M	Lease P
A	Finance lease	Operating lease
B	Finance lease	Finance lease
C	Operating lease	Finance lease
D	Operating lease	Operating lease
A

Explanation:
The correct answer is (B).

A Finance Lease must meet one of the following criteria:

Present Value equals or exceeds substantially all (90%) of the Fair Value
Option to Purchase (exercise is reasonably certain)
Economic Life - Major part (75%) of asset’s economic life is used
Transfer of Ownership at lease termination
Specialized Nature - No alternative use to the lessor at lease termination
Note: the implementation guidance for ASC 842 uses the 75/90 thresholds, even though the standard is principles-based.

Since both the leases have terms that are for the major part of the remaining economic life of the underlying asset, both are finance leases.

167
Q
On June 19, Don Co., a U.S. company, sold and delivered merchandise on a 30-day account to Cologne GmbH, a German corporation, for 200,000 euros. On July 19, Cologne paid Don in full. Relevant currency exchange rates were:
June 19	July 19
Spot rate	$ .988	$ .995
30-day forward rate	.990	1.000What amount should Don record on June 19 as an account receivable for its sale to Cologne?
A
$197,600
B
$198,000
C
$199,000
D
$200,000
A

Explanation:
At the date a transaction is recognized, each asset, liability, revenue, expense, gain, or loss arising from a foreign currency transaction should be measured and recorded in the functional currency of the recording entity by use of the exchange rate (i.e. spot rate) in effect at that date. Don Co should record an account receivable of $197,600 on June 19 (200,000 x .988)

168
Q

Cott, Inc. prepared an interest amortization table for a five-year lease payable with a purchase option of $2,000, exercisable at the end of the lease. At the end of the five years, the balance in the lease liability column of the spreadsheet was zero. Cott asked Grant, CPA, to review the spreadsheet to deter­mine the error. Only one error was made on the spreadsheet. Which of the following statements represents the best explanation for this error?

A
The beginning present value of the lease did not include the present value of the purchase option.
B
Cott subtracted the annual interest amount from the lease liability balance instead of adding it.
C
The present value of the purchase option was subtracted from the present value of the annual payments.
D
Cott discounted the annual payments as an ordinary annuity, when the payments actually occurred at the beginning of each period.

A

Explanation:
The correct answer is (A).

The error made by Cott Inc. would have been that the beginning present value of the lease did not include the present value of the bargain purchase option. Thus, once all the minimum lease payments were made, at the end of the lease term, the payable column would have shown zero.

Options (B) and (C) are incorrect because both these errors would have resulted in a negative payable amount, not zero.

Option (D) is incorrect because discounting the annual payments as an ordinary annuity when the payments actually occurred at the beginning of each period, would have resulted in a higher present value, leading to a positive balance of payable and not zero at the end of the lease term.

169
Q

Which of the following information about threatened litigation should not be considered to determine whether an accrual is appropriate prior to issuance of a company’s financial statements?

A
The period in which the underlying cause of the threatened litigation occurred
B
The degree of probability of an unfavorable outcome
C
The ability to make a reasonable estimate of the amount of loss
D
The period in which the threatened litigation became known to management

A

Explanation:
The correct answer is (D).

The following information about threatened litigation should be considered to determine whether an accrual is appropriate prior to issuance of a company’s financial statements:

The period in which the underlying cause of the threatened litigation occurred.
The degree of probability of an unfavorable outcome.
The ability to make a reasonable estimate of the amount of loss. This would be quantitative but a range is permissible.
The disclosure and accrual of contingency loss in the financial statements depend on the probability of occurrence of the loss and the possibility of reasonably estimating the amount of loss and the period in which the underlying cause of threatened litigation occurred. If the contingency loss is probable (likely to occur) and the amount of loss can be reasonably estimated and underlying cause of the threatened litigation occurred in last period then it should be accrued for prior to issuance of the company’s financial statements. Hence, the period in which the threatened litigation became known to management is irrelevant.

170
Q

Which of the following is not an indicator that a buyer-lessor has obtained control of the property?

A
Customer has legal title
B
Customer has physical possession
C
Customer has the significant risks and rewards of ownership
D
Customer has purchased the asset at fair value.
A

Explanation:
The seller-lessee satisfies its performance obligation by transferring control of assets to the buyer-lessor. ASC Topic 606 identifies five indicators that a customer (in this case a buyer-lessor) has obtained control of an asset:

Customer has legal title
Customer has physical possession
Customer has the significant risks and rewards of ownership
Customer has accepted the asset
Seller has a present right to payment.
Option (A), (B) and (C) are incorrect as per above explanation.

171
Q

On December 31, year 3, Byte Co. had capitalized software costs of $600,000 with an economic life of four years. Sales for year 4 were 10% of the expected total sales of the software. At December 31, year 4, the software had a net realizable value of $480,000. In its December 31, year 4 balance sheet, what amount should Byte report as net capitalized cost of computer software?

A
$432,000
B
$450,000
C
$480,000
D
$540,000
A

Explanation:
The correct answer is (B).

The net capitalized cost of computer software = $450,000.

In the case of computer software that is developed for sale, lease or to market as a product, the costs incurred to convert a technologically-feasible program into a final commercial product are capitalized. The capitalized costs are then amortized annually to arrive at the carrying value which is then compared with the net realizable value for reporting purposes.

Annual amortization = Greater of

SLM (Capitalized Amount / Estimate of economic life)
Units of sales [Capitalized Amount x (Revenues during the year / Total revenues expected)].
The software capitalized cost of Byte Co. on December 31, year 3 = $600,000.

Annual amortized cost for year 4 = $600,000 / 4 = $150,000. (Since SLM > Units of Sales)

Carrying value on December 31, year 4 = $600,000 - $150,000 = $450,000.

Net realizable value on December 31, year 4 = $480,000.

As the computer software has to be reported at the lower of carrying value (CV) or Net realizable value (NRV), Byte will report net capitalized cost of computer software at the carrying value of $450,000 because it is lower than the net realizable value (NRV) of $480,000.

Option (A) is incorrect because of incorrect calculation [i.e. $432,000 = $480,000 (NRV) – 10% of $480,000].

Option (C) is incorrect because the computer software has to be reported at the lower of carrying value (CV) $450,000 or Net realizable value (NRV) $480,000.

Option (D) is incorrect because this is based on unit of sales method (i.e. $540,000 = $600,000 – 10% of $600,000).

172
Q

At December 31 of the current year, Off-Line Co. changed its method of accounting for demo costs from writing off the costs over two years to expensing the costs immediately. Off-Line made the change in recognition of an increasing number of demos placed with customers that did not result in sales. Off-Line had deferred demo costs of $500,000 at December 31 of the previous year, $300,000 of which were to be written off in the current year and the remainder in the following year. Off-Line’s income tax rate is 30%. In its current year financial statements, what should Off-Line report?
A
$140,000 as cumulative effect of change in accounting principle
B
$200,000 as current period transaction with no prior period adjustment
C
$350,000 as cumulative effect of change in accounting principle
D
$500,000 as current period transaction with no prior period adjustment

A

Explanation:
A change in accounting estimate affected by a change in accounting principle is accounted for as a change in estimate. A change in accounting estimate shall be accounted for in (a) the period of change if the change affects that period only or (b) the period of change and future periods if the change affects both. A change in accounting estimate shall not be accounted for by restating or retrospectively adjusting amounts reported in financial statements of prior periods or by reporting pro forma amounts for prior periods. Because the change was made, the total amount of $500,000 would be recorded.

173
Q

A transaction was reported as a nonmonetary exchange of assets. Under which of the following circumstances should the exchange be measured based on the reported amount of the nonmonetary asset surrendered?
A
When the entity’s future cash flows are expected to change as a result of the exchange.
B
When the timing of future cash flows of the asset received differs significantly from the configuration of the future cash flows of the asset transferred.
C
When the transaction lacks commercial substance.
D
When the transaction has commercial substance.

A

Explanation:
Both exchanges and nonreciprocal transfers that involve little or no monetary assets or liabilities are referred to as nonmonetary transactions. Nonmonetary exchanges should be based on recorded amounts (rather than fair values) of the exchanged assets if any of the following conditions apply: (a) neither the fair value of the assets received nor the fair value of the assets surrendered is determinable within reasonable limits; (b) the transaction is an exchange of a product or property held for sale in the ordinary course of business for a product or property to be sold in the same line of business to facilitate sales to customers (other than the parties to the exchange); or (c) the transaction lacks commercial substance. In a nonmonetary exchange that has no commercial substance, the assets exchanged are accounted for at book value (after reduction, if appropriate, for an indicated impairment of value) of the nonmonetary asset(s) given up.

174
Q

On January 17 of the current year, an explosion occurred at a Sims Co. plant causing extensive property damage to area buildings. Although no claims had yet been asserted against Sims by March 10, Sims’ management and counsel concluded that it is likely that claims will be asserted and that it is reasonably possible Sims will be responsible for damages. Sims’ management believed that $1,250,000 would be a reasonable estimate of its liability. Sims’ $5,000,000 comprehensive public liability policy has a $250,000 deductible clause. In Sims’ prior year December 31 financial statements, which were issued on March 25 of the current year, how should this item be reported?
A
As an accrued liability of $250,000
B
As a footnote disclosure indicating the possible loss of $250,000
C
As a footnote disclosure indicating the possible loss of $1,250,000
D
No footnote disclosure or accrual is necessary

A

Explanation:
Accruing a contingent liability requires that the likelihood of the loss be probable and the amount be reasonably subject to estimation. Since the likelihood of the loss contingency in question is only reasonably possible, the loss contingency should not be accrued; it should only be disclosed. The disclosure should indicate the nature of the contingency and the minimum amount of the possible loss, the $250,000 deductible clause of the policy.

175
Q
Frey Corp. failed to accrue warranty costs of $50,000 in its previous year December 31 financial statements. In addition, a change from straight-line to accelerated depreciation made at the beginning of the current year resulted in a cumulative effect of $30,000 on Frey's retained earnings. Both the $50,000 and the $30,000 are net of related income taxes. What amount should Frey report as prior period adjustments in the current year?
A
$0
B
$30,000
C
$50,000
D
$80,000
A

Explanation:
The recognition of the effect of the failure to accrue $50,000 of warranty costs in the previous year is the correction of an error of a prior period. The correction of the error should be reported as a prior-period adjustment by restating the prior-period financial statements. The change from straight-line to accelerated depreciation represents a change in accounting estimate effected by a change in accounting principle. A change in accounting estimate should only be accounted for in the period of change if the change only effects that period, or in the current and subsequent periods, if the change effects both, as a component of income from continuing operations.

176
Q
During January of the current year, Haze Corp. won a litigation award for $15,000 which was tripled to $45,000 to include punitive damages. The defendant, who is financially stable, has appealed only the $30,000 punitive damages. Haze was awarded $50,000 in an unrelated suit it filed, which is being appealed by the defendant. Counsel is unable to estimate the outcome of these appeals. In its current year financial statements, Haze should report what amount of pretax gain?
A
$15,000
B
$45,000
C
$50,000
D
$95,000
A

Explanation:
Haze should report a pretax gain of $15,000 (i.e., $45,000 - $30,000) for the amount of litigation award from the financially stable defendant that is not being appealed. The portion of the litigation awards that are being appealed (i.e., $30,000 and $50,000) represent gain contingencies and, thus, should not be accrued before realization; however, they should be disclosed in the financial statement notes.

177
Q

Which of the following statements is correct regarding accounting changes that result in financial statements that are, in effect, the statements of a different reporting entity?

A
Cumulative-effect adjustments should be reported as separate items on the financial statements pertaining to the year of change.
B
No restatements or adjustments are required if the changes involve consolidated methods of accounting for subsidiaries.
C
No restatements or adjustments are required if the changes involve the cost or equity methods of accounting for investments.
D
The financial statements of all prior periods presented should be reported by retrospective application.

A

Explanation:
Retrospective application based on the earliest period presented is provided for an accounting change that is change in reporting entity. Here, the term “restatement” refers only to correction of errors in previously issued financial statements.

Options (A), (B) and (C) are incorrect based on the above explanation.

178
Q

Bensol Co. and Sable Co. exchanged similar trucks with fair values in excess of carrying amounts. In addition, Bensol paid Sable to compensate for the difference in truck values. As a consequence of the exchange, which lacked commercial substance, Sable recognizes
A
A gain equal to the difference between the fair value and carrying amount of the truck given up.
B
A gain determined by the proportion of cash received to the total consideration.
C
A loss determined by the proportion of cash received to the total consideration.
D
Neither a gain nor a loss.

A

Explanation:
Sable exchanged a truck with a fair value in excess of its carrying amount for a similar truck and received monetary consideration. Hence, Sable experiences a gain on the exchange. The recipient of monetary consideration in an exchange measured based on the recorded amount recognizes recognizes a portion of the gain experienced. The amount of the gain recognized is based on the ratio of the monetary consideration to the total consideration received.

179
Q

Arena Corp. leased equipment from Bolton Corp. and correctly classified the lease as a finance lease. The present value of the minimum lease payments at the lease inception was $1,000,000. The initial direct costs paid by Arena were $50,000, and the fair value of the equipment at lease inception was $900,000. What amount should Arena report as the lease liability and ROU Asset at the lease inception?

Lease Liability	ROU Asset
A	$900,000	$950,000
B	$950,000	$950,000
C	$900,000	$1,000,000
D	$1,050,000	$900,000
A

Explanation:
The correct answer is (A).

At commencement, the initial measurement of the lease liability (regardless of lease classification) is calculated as the lower of cost (present value of minimum lease payments) or fair market value of the leased asset at the time of inception of the lease with a corresponding lease liability.

The present value of minimum lease payments for the equipment was $1,000,000.
The fair value of the equipment at lease inception was $900,000.
The lower of the two values $900,000 would be recorded as the lease liability whereas the initial measurement of the ROU asset (regardless of lease classification) is calculated as the lease liability, increased by any initial direct costs and prepaid lease payments, reduced by any lease incentives received before commencement i.e. $900,000 (lease liability) + $50,000 (Initial Direct costs) = $950,000.

180
Q

In an operating lease, the lessor’s net rental income includes which of the following?

A
Amortization of any lease bonus
B
Amortization of any initial direct costs incurred
C
Executory costs incurred
D
All of the above
A

Explanation:
Although the lessor’s net rental income from an operating lease includes the amortization of any lease bonus, the net rental income also includes the amortization of any initial direct costs incurred and the recording of any executory costs incurred.

Since the answers for A., B., and C. are all correct, the best answer for this question is D., all of the above.

181
Q

On March 15, year 2, a calendar-year company issued its year 1 financial statements. On March 1, year 2, a fire destroyed the company’s only manufacturing plant. Which of the following statements is correct regarding the treatment of the loss in the December 31, year 1, financial statements?

A
The loss should not be recognized or disclosed in the year 1 financial statements
B
The loss should be recognized in the year 1 financial statements
C
The loss should be disclosed and not recognized in the year 1 financial statements
D
Any probable insurance recoveries should be recognized in the year 1 financial statements

A

Explanation:
The correct answer is (C).

There are two types of subsequent events:

Recognized subsequent events are those for which condition existed as on Balance Sheet date and recognized in Financial Statements.
Unrecognized subsequent events are those for which conditions did not exist as on Balance Sheet date and they arose later and are recognized in Financial Statements. However, footnote disclosure should be made if Financial Statements would become misleading otherwise.
A fire or a natural disaster are unpredictable and conditions for it did not exist on the Balance Sheet date. Because the fire did not occur until after the balance sheet date, it is a Type II unrecognized subsequent event. Type II subsequent events are not recognized in the financial statements, but disclosure of the event is required.

182
Q

A company owns land and a building that houses its manufacturing operations. When the company purchased the manufacturing facility 10 years ago, the purchase price allocated to the land account was $120,000. The manufacturing facility is located in an area that was once the site of many factories. The owners of many of the neighboring factories have recently sold their facilities to residential real estate developers. The company’s land is also suitable for residential development. The estimated current value of the land as part of the manufacturing facility is $150,000. The estimated current value of the land as an undeveloped investment is $130,000, and the current value of the land as part of a residential development would be $180,000. What is the fair value of the land?

A
$120,000
B
$130,000
C
$150,000
D
$180,000
A

Explanation:
The correct answer is (D).

The fair value of an asset is determined in the most advantageous market, in which the amount realized for the asset is the maximum or in which the costs to sell are the least.

In the given problem, there are three advantageous markets for the land:

As a manufacturing facility
As an undeveloped investment
As part of a residential development
However, the amount realized for the land as part of a residential development is the maximum value vs all other markets. The fair value of the said land is $180,000.

183
Q
On January 1, year 3, Pell Corp. purchased a machine having an estimated useful life of 10 years and no salvage. The machine was depreciated by the double declining balance method for both financial statement and income tax reporting. On January 1, year 8, Pell changed to the straight-line method for financial statement reporting but not for income tax reporting. Accumulated depreciation at December 31, year 7, was $560,000. If the straight-line method had been used, the accumulated depreciation at December 31, year 7, would have been $420,000. Pell's enacted income tax rate for year 8 and thereafter is 30%. The amount shown in the year 8 income statement for depreciation expense after changing to the straight-line method should be a
A
$56,000 debit.
B
$84,000 debit.
C
$84,000 credit.
D
$98,000 credit.
A

Explanation:
A change in depreciation method, such as from the double declining balance to the straight-line method is a change in accounting principle that is inseparable from a change in accounting estimate. When the effects of the two changes cannot be separated, the change should be treated as a change in estimate. A change in accounting estimate should be accounted for in the period of change if the change only effects that period, or in the current and subsequent periods, if the change effects both, as a component of income from continuing operations. If accumulated depreciation using the straight-line method at the end of year 7, 5 years worth, would have been $420,000 the machine must have cost $840,000. The $840,000 less accumulated depreciation of $560,000 gave the machine a $280,000 carrying value as of January 1, year 8. Dividing that $280,000 by 5 more remaining straight-line years provides for $56,000 of depreciation expense in each of the remaining years. Depreciation expense would be debited for $56,000. No prior period adjustment is made.

184
Q
Park Co.'s wholly-owned subsidiary, Schnell Corp., maintains its accounting records in euros. Because all of Schnell's branch offices are in Switzerland, its functional currency is the Swiss franc. Remeasurement of Schnell's current year financial statements resulted in a $7,600 gain, and translation of its financial statements resulted in an $8,100 gain. What amount should Park report as a foreign exchange gain in its income statement for the current year ended December 31?
A
$0
B
$ 7,600
C
$ 8,100
D
$15,700
A

Explanation:
Park’s foreign subsidiary does not maintain its accounting records in its functional currency (the Swiss Franc). Therefore, the financial statements of the foreign subsidiary must be remeasured. The $7,600 remeasurement gain is reported in income from continuing operations. The $8,100 translation gain is reported in other comprehensive income. Translation gains and losses are not reported in income.

185
Q
When a company changes from the straight-line method of depreciation for previously recorded assets to the double-declining-balance method, which of the following should be reported?
Cumulative effect of
changes in accounting principle	Pro forma effects of
retroactive application
A	No	No
B	No	Yes
C	Yes	Yes
D	Yes	No
A

Explanation:
A change in depreciation methods is a change in estimate effected by a change in principle. It is treated as change in estimate, prospective only. No cumulative or pro forma effects are reported.

186
Q

On January 2, year 3, to better reflect the variable use of its only machine, Holly, Inc. elected to change its method of depreciation from the straight-line method to the units of production method. The original cost of the machine on January 2, year 1, was $50,000, and its estimated life was 10 years. Holly estimates that the machine’s total life is 50,000 machine hours. Machine hours usage was 8,500 during year 2 and 3,500 during year 1. Holly’s income tax rate is 30%. Holly should report the accounting change in its year 3 financial statements as a (an)
A
Cumulative effect of a change in accounting principle of $2,000 in its income statement.
B
Adjustment to beginning retained earnings of $2,000.
C
Current period transaction with no prior period adjustment.
D
Adjustment to beginning retained earnings of $1,400.

A

Explanation:
A change in depreciation method is a change in accounting principle that is inseparable from a change in accounting estimate. When the effects of the two changes cannot be separated, the change should be treated as a change in estimate. A change in accounting estimate shall be accounted for in (a) the period of change if the change affects that period only or (b) the period of change and future periods if the change affects both. A change in accounting estimate shall not be accounted for by restating or retrospectively adjusting amounts reported in financial statements of prior periods or by reporting pro forma amounts for prior periods.

187
Q

On December 12, year 1, Imp Co. entered into three forward exchange contracts, each to purchase 100,000 euros in 90 days. The relevant exchange rates are as follows:

Spot rate Forward rate (for 3/12, year 2)

December 12, year 1 $1.86 $1.80
December 31, year 1 $1.96 $1.83
Imp entered into the first forward contract to hedge a purchase of inventory in November of year 1, payable in March of year 2. At December 31, year 1, what amount of foreign currency transaction gain should Imp include in income from this forward contract?

A
$0
B
$3,000
C
$ 10,000
D
$ 5,000
A

Explanation:
The correct answer is (B).

Since the forward contract was entered into to hedge a liability that is payable in March, it would be accounted for as a cash flow hedge. At December 31, Imp has a contract to purchase 100,000 Euros on March 12 for $1.80 per Euro when the expected exchange rate is $1.83 per Euro. As a result, Imp will gain $.03 per Euro and the derivative would be worth (100,000 x $.03) $3,000.

The derivative would be adjusted to fair value but any gain or loss would be recognized in comprehensive income until such time as the effect on the hedged item. In this case, the hedged item is a payable related to the purchase of inventory, which would have been originally recorded at the spot rate in November, which is not given. At December 31, year 1, the spot rate has increased to $1.96, requiring that the liability is increased to $196,000 resulting in a loss. As a result, the gain on the cash flow derivative would be recognized in income in the same period.

Journal Entries

12/12/Year 1:

J/E @90-day forward rate for the forward exchange contract (entered to hedge against a possible increase in the value of Euros to be paid when the invoice is due for inventory)

Dr. Euro receivable $180,000
Cr. US$ payable $180,000

12/31/20X0:

J/E for gain or loss on a forward exchange contract at the forward rate.

Dr. Euro receivable $3,000
Cr. Gain on Forward Exchange Contract $3,000

Note: Foreign currency transactions can have operating transactions in the foreign currency such as buying inventory or a purchase commitment for equipment. Fluctuations in foreign exchanges (spot rate) will result in a gain or loss in inventory or receivable/payable. Such fluctuations always use the spot rate.

To hedge such fluctuations, a forward contract is used, which is an agreement to exchange two different currencies at a future date, at a specific rate.

Gains or losses in forward contracts use the forward rate.

In this particular question when spot rates increase, the payable for inventory will increase resulting in a loss.

However, the forward exchange contract will have a gain of $1.83 - $1.80 i.e. $0.03 resulting in a $3,000 gain on the forward contract.

188
Q

A company owns a financial asset that has no principal market. The financial asset is actively traded in four markets and the company has the ability to transact in all four of these markets. The following are the quoted prices for the financial asset in each of the four markets:

Market 	Quoted Price 
A 	$20,000 
B 	25,000 
C 	30,000 
D 	35,000 
What is the fair value of the financial asset?
A
$20,000
B
$25,000
C
$27,500
D
$35,000
A

Explanation:
The correct answer is (D).

In order to determine the fair value of an asset, the principal market is considered. In the absence of a principal market, the most advantageous market which maximizes the sales proceeds for the said asset or has the least costs to sell is used.

In the given case, the financial asset has no principal market. The most advantageous market shall be used. Since Market D has the maximum quoted price of $35,000, it should be used as the fair value of the asset.

189
Q

In the long-term liabilities section of its balance sheet at December 31, year 1, Mene Co. reported a Finance Lease obligation of $75,000, net of current portion of $1,364. Payments of $9,000 were made on both January 2, year 2 and January 2, year 3. Mene’s incremental borrowing rate on the date of the lease was 11% and the lessor’s implicit rate, which was known to Mene, was 10%. In its December 31, year 2, balance sheet, what amount should Mene report as Finance Lease obligation, net of current portion?

A
$66,000
B
$73,500
C
$73,636
D
$74,250
A

Explanation:
The Finance Lease obligation must be reduced by only the principal portion of the $9,000 payment. The $75,000 already is net of the $1,364 payment made on January 2, year 2, and must be further reduced by the amount of amortization of the lease obligation for year 2. The lessor’s implicit rate of interest is known to the lessee and it thus the lessee must use the lessor’s implicit rate.

Noncurrent Finance Lease obligation, 12/31, year 1 $75,000
Less: Current portion [$9,000 - ($75,000 × 10%)] (1,500)
Noncurrent Finance Lease obligation, 12/31, year 2 $73,500

190
Q

On January 1, year 1, JCK Co. signed a contract for an eight-year lease of its equipment with a 10-year life. The present value of the 16 equal semiannual payments in advance equaled 85% of the equipment’s fair value. The contract had no provision for JCK, the lessor, to give up legal ownership of the equipment. Should JCK recognize rent or interest revenue in year 3, and should the revenue recognized in year 3 be the same or smaller than the revenue recognized in year 2?

Year 3 revenues recognized	Year 3 amount recognized compared to year 2
A	Rent	The same
B	Rent	Smaller
C	Interest	The same
D	Interest	Smaller
A

Explanation:
The correct answer is (D).

The 8-year lease term is for the majority of the equipment’s 10-year useful life. JCK will record the lease as a finance lease. JCK will recognize interest income, but no rent under a finance lease.

Each payment received will be allocated to interest first with the remainder allocated to reducing the principal balance of the receivable. The principal balance is reduced each period, the amount allocated to interest will decrease each period, as interest will be calculated on the decreasing principal balance each year. Thus, JCK will recognize interest revenue and the amount will decrease each year.

Options (A), (B) and (C) are incorrect based on the above explanation.

191
Q
During December of the previous year, Nile Co. incurred special insurance costs but did not record these costs until payment was made during the current year. These insurance costs related to inventory that had been sold by December 31 of the previous year. What is the effect of the omission on Nile's accrued liabilities and retained earnings at December 31 of the previous year?
Accrued liabilities	Retained earnings
A	No effect	No effect
B	No effect	Overstated
C	Understated	Overstated
D	Understated	No effect
A

Explanation:
Accrual accounting recognized and reports the effects of transactions and other events on the assets and liabilities of a business enterprise in the time periods to which they relate rather than only when cash is received or paid. Accrual accounting attempts to match revenues and the expenses associated with those revenues in order to determine net income for an accounting period. The insurance costs were not recorded during the period to which they relate, so the accrued liabilities is understated. Because the expense was not included in the inventory that was sold, the COGS was less than it should have been, and as a result, the retained earnings were overstated.

192
Q

Tulip Co. owns 100% of Daisy Co.’s outstanding common stock. Tulip’s cost of goods sold for the year totals $600,000 and Daisy’s cost of goods sold totals $400,000. During the year, Tulip sold inventory costing $60,000 to Daisy for $100,000. By the end of the year, all transferred inventory was sold to third parties. What amount should be reported as cost of goods sold in the consolidated statement of income?

A
$900,000
B
$940,000
C
$960,000
D
$1,000,000
A

Explanation:
The correct answer is (A).

In the consolidated statement of income, inter-company transactions have to be eliminated. In an inter-company sale of inventory, the sale and purchase of the inventory between the holding and the subsidiary have to be eliminated. Tulip Co. sold inventory to Daisy for $100,000. The eliminating entry for this transaction, in the consolidated statements, would be

Sales $100,000
COGS $100,000
The elimination entry reduces the cost of goods sold (COGS) by $100,000. The total cost of goods sold in the consolidated statements would be the COGS of Tulip of $600,000 + the COGS of Daisy of $400,000 - decrease in COGS due to inter-company elimination entry of $100,000 = $900,000.

193
Q

Which of the following statements is correct concerning start-up costs?
A
Costs of start-up activities, including organization costs, should be expensed as incurred.
B
Costs of start-up activities, including organization costs, should be capitalized and expensed only if an impairment exists.
C
Costs of start-up activities, including organization costs, should be capitalized and amortized on a straight-line basis over the lesser of the estimated economic life of the company, or 60 months.
D
Costs of start-up activities should be capitalized and amortized on a straight-line basis over the lesser of the estimated economic life of the company, or 60 months, while organization cost should be expensed as incurred.

A

Explanation:
Generally accepted accounting principles that apply to established operating enterprises govern the recognition of revenue by a development stage enterprise and determine whether a cost incurred by a development stage enterprise is to be charged to expense when incurred or is to be capitalized or deferred. Accordingly, capitalization or deferral of costs shall be subject to the same assessment of recoverability that would be applicable in an established operating enterprise. Amortization is a component of the tax treatment for organization costs; it is not an option for financial accounting purposes. Organization costs are written off over 60 months for tax purposes, not for financial accounting purposes. Start-up activities, including organization costs, should be expensed as incurred.

194
Q

Which of the following statements regarding changes in accounting estimates is false?
A
Changes in accounting estimates differ from changes in accounting principles in that changes in estimates are simply necessary consequences of periodic financial reporting.
B
Examples of changes in estimates reflected in the financial statements include changes in estimates related to uncollectible receivables.
C
Examples of changes in estimates reflected in the financial statements do not include warranty costs.
D
Accounting estimates change as new events occur, as more experience is acquired, and as additional information is obtained.

A

Explanation:

False. Examples of changes in estimates reflected in the financial statements do include warranty costs.

195
Q
Nolan owns 100% of the capital stock of both Twill Corp. and Webb Corp. Twill purchases merchandise inventory from Webb at 140% of Webb's cost. During the current year, merchandise that cost Webb $40,000 was sold to Twill. Twill sold all of this merchandise to unrelated customers for $81,200 during this year. In preparing combined financial statements for the year, Nolan's bookkeeper disregarded the common ownership of Twill and Webb. By what amount was unadjusted revenue overstated in the combined income statement for the year?
A
$16,000
B
$40,000
C
$56,000
D
$81,200
A

Explanation:
Any sale of inventory made between two commonly controlled companies will trigger the individual accounting systems for both companies. Revenue is recorded by the seller while the purchase is simultaneously entered into the buyer’s accounts. However, from a combined perspective, neither sale nor purchase has occurred. Thus, all intercompany sales are eliminated from the combined financial statements. Since the intercompany sale was not eliminated in preparing the combined financial statements, unadjusted revenue is overstated by $56,000 ($40,000 × 140%).

196
Q

Which of the following methods should be used to account for research and development costs with no alternative future use?

A
Charging all costs to expense when incurred.
B
Capitalizing all costs when incurred.
C
Capitalizing costs specified by management and charging all other costs to expense.
D
Accumulating all costs in a separate component of stockholders’ equity until the existence of future benefits can be determined.

A

Explanation:
The correct answer is (A).

Under US GAAP, Research and Development costs are almost always expensed as and when they are incurred. These expenses are capitalized only when alternate future uses are expected from such research and development carried out. Thus, research and development costs with no alternative future use should be charged to expense when incurred.

197
Q

For which of the following reporting units is the preparation of combined financial statements most appropriate?
A
A corporation and a majority-owned subsidiary with nonhomogeneous operations.
B
A corporation and a foreign subsidiary with nonintegrated homogeneous operations.
C
Several corporations with related operations with some common individual owners.
D
Several corporations with related operations owned by one individual.

A

Explanation:
There are circumstances where combined financial statements (as distinguished from consolidated financial statements) of commonly controlled companies are likely to be more meaningful than their separate statements. Examples of such circumstances are: (1) where one individual owns a controlling interest in several corporations which are related in their operations, (2) to present the financial position and the results of operations of a group of unconsolidated subsidiaries and (3) to combine the financial statements of companies under common management.

198
Q
During 2004, a former employee of Dane Co. began a suit against Dane for wrongful termination in November 2003. After considering all of the facts, Dane's legal counsel believes that the former employee will prevail and will probably receive damages of between $1,000,000 and $1,500,000, with $1,300,000 being the most likely amount. Dane's financial statements for the year ended December 31, 2003, will not be issued until February 2004. In its December 31, 2003, balance sheet, what amount should Dane report as a liability with respect to the suit?
A
$0
B
$1,000,000
C
$1,300,000
D
$1,500,000
A

Explanation:
Contingent liabilities arise from events or circumstances occurring before the balance sheet date, the resolution of which is contingent on a future event or circumstance. Pending or threatened litigation is one example of a contingent liability. The accounting treatment depends on the likelihood that future events will confirm the contingent loss and whether the amount can be reasonably estimated. Where the likelihood of confirmation of a loss is considered probable and the loss can be reasonable estimated, the estimated loss should be accrued by a charge to income and the nature of the contingency should be disclosed. If, however, only a range of possible loss can be estimated, with no amount in the range better than any other, the minimum amount in the range should be accrued. The wrongful termination event took place in 2003 and because a loss from it is considered probable it should be reported in the December 31, 2003, balance sheet. The most likely amount (reasonably estimated) would be reported.

199
Q

In a transaction accounted for as a business combination, the appraised values of the identifiable net assets acquired exceeded the acquisition price. How should the excess appraised value be reported?

A
As negative goodwill
B
In Income from Discontinued Operations
C
In Other Comprehensive Income
D
In Income from Continuing Operations
A

Explanation:
The correct answer is (D).

In a business combination, if the appraised values of the identifiable assets acquired exceeded the acquisition price, it would lead to a credit balance (a negative goodwill) which is recorded as a gain on the income statement from Continuing Operations.

(A) is incorrect because negative goodwill will be recorded as a gain in the income statement.

(B) and (C) are incorrect because these are inaccurate explanations.

200
Q

Each of the following would be considered a Level 2 observable input that could be used to determine an asset or liability’s fair value, except

A
Quoted prices for identical assets and liabilities in markets that are not active.
B
Quoted prices for similar assets and liabilities in markets that are active.
C
Internally generated cash flow projections for a related asset or liability.
D
Interest rates that are observable at commonly quoted intervals.

A

Explanation:
Level 2 inputs are directly or indirectly observable inputs; it includes:

Quoted prices from active markets for similar assets/liabilities
Quoted prices from limited activity markets for similar/identical assets
Other observable inputs like yield curves, bank prime rates, interest rates, credit risks, default rates on loans.
Options (A), (B) and (D) are incorrect because all of the above fall in the level 2 input category.

201
Q

On December 31 of the current year, Park Corp. sold Edlow Corp. an airplane with an estimated remaining life of ten years. At the same time, Park leased back the airplane for 3 years. Additional information is as follows:

Sales price $600,000
Carrying amount of airplane at sale date $100,000
Monthly rental under lease $ 6,330
Interest rate implicit in the lease as computed by Edlow and known by Park 12%
Present value of operating lease rentals ($6,330 for 36 months @ 12%) $190,581
The leaseback is considered an operating lease. In Park’s December 31 current year balance sheet, what amount should be included as deferred revenue on this transaction?

A
$0
B
$190,581
C
$309,419
D
$500,000
A

Explanation:
The correct answer is Option (A).

FASB issued ASC 842 to amend accounting & reporting for leases under which for a sale to occur in the context of a sale and leaseback transaction, the transfer of the asset must meet the requirements for a sale per the Revenue Recognition standards. If there is no sale for the seller-lessee, the buyer-lessor also does not account for a purchase.

Any consideration paid for the asset is accounted for as a financing transaction by both the seller-lessee and the buyer-lessor. If the leaseback is a finance lease from seller-lessee’s perspective, transfer of the asset is not a Sale.

However, here the leaseback is an operating lease.

The transaction is recognized as a sale and the entire gain is recognized immediately.

Option (B), (C) and (D) are incorrect because the gain is not deferred in a sale-leaseback transaction as per ASC 842.

202
Q

When there is a change in the reporting entity, how should the change be reported in the financial statements?

A
Prospectively, including note disclosures
B
Retrospectively, including note disclosures, and application to all prior period financial statements presented
C
Currently, including note disclosures
D
Note disclosures only
A

Explanation:
A change in reporting entity occurs when an accounting change results in financial statements that are, in effect, the statements of a different reporting entity. The change is reported by retrospectively applying the change to the financial statements of all prior periods presented to include the financial information for the new reporting entity for the periods. A description of both the nature of the change and the reason for it shall be disclosed in the note disclosures for the period of the change.

Options (A), (C) and (D) are incorrect as per the above explanation.

203
Q

For the previous year, Pac Co. estimated its two-year equipment warranty costs based on $100 per unit sold in the previous year. Experience during the current year indicated that the estimate should have been based on $110 per unit. The effect of this $10 difference from the estimate is reported
A
In current year income from continuing operations.
B
As an accounting change, net of tax, below current year income from continuing operations.
C
As an accounting change requiring previous year financial statements to be restated.
D
As a correction of an error requiring previous year financial statements to be restated.

A

Explanation:
A change in estimated warranty costs is an example of a change in accounting estimate. The effect of a change in accounting estimate should be accounted for as a component of income from continuing operations entirely in the period of change if it affects that period only. Therefore, the effect of the additional $10 of estimated warranty costs should be reported in current year income from continuing operations since the change in accounting estimate affects only the current year.

204
Q

At the inception of a finance lease, the guaranteed residual value should be:

A
Included as part of minimum lease payments at present value.
B
Included as part of minimum lease payments at future value.
C
Included as part of minimum lease payments only to the extent that guaranteed residual value is expected to exceed estimated residual value.
D
Excluded from minimum lease payments

A

Explanation:
The correct answer is (C).

In a finance lease, at the inception, the lessee would capitalize the present value of future minimum lease payments.

Minimum lease payments include:

Periodic lease payments

+ Required buyout of guaranteed residual value (Guaranteed Residual Value – Expected Residual Value)
+ Bargain purchase option, if any.
The guaranteed residual value would be included as part of minimum lease payments at the present value of the difference between Guaranteed Residual Value and Expected Residual Value

Options (A), (B) and (D) are incorrect based on the above explanation.

205
Q

Zero Corp. suffered a material loss on an uncollectible trade account receivable after a customer suddenly went out of business due to a natural disaster ten days after Zero’s balance sheet date, but one month before the issuance of the financial statements. Under these circumstances, the financial statements:
A
Should be adjusted, but should not disclose the event
B
Should be adjusted and should disclose the event
C
Should not be adjusted and should not disclose the event
D
Should not be adjusted, but should disclose the event

A

Explanation:
There are two types of subsequent events. The first type requires adjustment of the financial statements because it provides evidence about conditions that existed at the balance sheet date. The second type of subsequent event provides evidence about conditions that did not exist at the balance sheet date and does not require adjustment of the financial statements; however, if material in nature, the event(s) may require disclosure to keep the financial statements from being misleading.

This is an example of the second type. The event requires disclosure only.

206
Q

A material overstatement in ending inventory was discovered after the year-end financial statements of a company were issued to the public. What effect did this error have on the year-end financial statements?

Current assets	Gross profit
A	Understated	Overstated
B	Overstated	Overstated
C	Understated	Understated
D	Overstated	Understated
A

Explanation:
The current assets portion of the balance sheet would be overstated because the inventory account is part of current assets. Gross profit would be overstated because an overstatement in inventory results in an understatement of cost of goods sold. The ending inventory is subtracted from goods available for sale to get cost of goods sold. If costs of goods sold is understated, then gross profit is overstated.

207
Q

Stam Co. incurred the following research and development project costs during the current year:
Equipment purchased for current and future projects $100,000
Equipment purchased for current projects only 200,000
Research and development salaries for current projects 400,000
Legal fees to obtain patent 50,000
Material and labor costs for prototype product 600,000The equipment has a five-year useful life and is depreciated using the straight-line method. What amount should Stam recognize as research and development expense at year end?
A
$ 450,000
B
$ 1,000,000
C
$ 1,220,000
D
$ 1,350,000

A

Explanation:
Future economic benefits deriving from research and development (R &D) activities, if any, are uncertain in their amount and timing. Due to these uncertainties, most R&D costs are required to be charged to expense the year in which incurred. However, any materials, equipment, facilities, or intangibles purchased that have alternative future uses should be recorded as assets. Assets recorded for R&D costs with alternative future uses should be amortized over their useful lives by periodic charges to R&D expense. Stam should recognize only the amortization amount of $20,000 as R&D expense for the $100,000 worth of equipment purchased for current and future projects. Stam should also recognize the $200,000 of equipment purchased for current projects only, the $400,000 of R&D salaries for current projects, and the $600,000 of material and labor costs for a prototype product as R& D expense at year end for a total of $1,220,000. The legal fees to obtain a patent would not be included in R&D expense. Only R&D costs incurred to internally develop a patent would be expensed as incurred.

208
Q

If Tron Inc. a US-based entity has accounts payable valued in foreign currency, it should be adjusted for
changes in the:

A
Spot rate and the foreign exchange gains or losses are reported as operating gains or losses on the income statement.
B
Spot rate and the foreign exchange gains or losses are reported as non-operating gains or losses on the income statement.
C
Average rate and the foreign exchange gains or losses are reported as operating gains or losses on the income statement.
D
Average rate and the foreign exchange gains or losses are reported as non-operating gains or losses on the income statement.

A

Explanation:
The correct answer is (B).

When an entity with the US Dollar as a functional currency engages in a transaction in a foreign currency, it remeasures the transaction using the spot rate as of the transaction date.

If the entity has any monetary assets or liabilities (e.g., cash, accounts receivable, accounts payable) valued in the foreign currency, these need to be adjusted for changes in the spot rate and the foreign exchange gains or losses are reported as non-operating gains or losses on the Income Statement.

209
Q

On June 1 of the current year, Ichor Company entered into a ten-year non-cancellable lease with Gillie, Inc., for a machine owned by Gillie. The machine had a fair value of $180,000 at the inception of the lease. Ownership of the machine is transferred to Ichor upon the expiration of the lease. The present value of the ten $30,000 annual lease payments, based on Ichor’s incremental borrowing rate of 12%, is $180,000. The lease agreement specifies that all executory costs are assumed by Ichor. How much should Ichor record as an asset and corresponding lease liability at the inception of the lease?

A
$0
B
$150,000
C
$180,000
D
$300,000
A

Explanation:
The correct answer is (C).

A Finance Lease must meet one of the following criteria:

Present Value equals or exceeds substantially all (90%) of the Fair Value
Option to Purchase (exercise is reasonably certain)
Economic Life - Major part (75%) of the asset’s economic life is used
Transfer of Ownership at lease termination
Specialized Nature - No alternative use to the lessor at lease termination
Note: the implementation guidance for ASC 842 uses the 75/90 thresholds, even though the standard is principles-based.

Since at the end of the lease term the ownership of the machine is transferred to Ichor, the lease is classified as a finance lease.

Executory costs are recurring expenses, which if incurred by the lessee, is expensed as incurred.

At commencement, the initial measurement of the asset (regardless of lease classification) is calculated as the lease liability, increased by any initial direct costs and prepaid lease payments, reduced by any lease incentives received before commencement.

Since there is no initial direct cost, prepaid lease payments or lease incentives, the leased asset would be equal to lease liability which is the present value of minimum lease payments of $180,000 which will be capitalized as a leased asset and lease liability at the inception of the lease.

210
Q

On August 1 of the current year, Kern Company leased a machine to Day Company for a 6-year period requiring payments of $10,000 at the beginning of each year. The machine cost $48,000, which is the fair value at the lease date, and has an estimated life of eight years with no residual value. Kern’s implicit interest rate is 10% and present value factors are as follows:

Present value of an annuity due of $1 at 10% for 6 periods 4.791
Present value of an annuity due of $1 at 10% for 8 periods 5.868
Kern appropriately recorded the lease as a sales-type lease. At the inception of the lease, the Net Investment in lease account balance should be

A
$60,000
B
$58,680
C
$48,000
D
$47,910
A

Explanation:
The net investment in the lease is measured as the present value of the sum of the (1) lease payments not yet received by the lessor, (2) residual value guaranteed by the lessee or a third party unrelated to the lessor, and (3) residual value that is not guaranteed by the lessee or a third party unrelated to the lessor. The lessor determines the present value of the lease payments not yet received as 47,910 ( 10,000 x 4.791)

Option (A), (B) and (C) are incorrect as per above explanation.

211
Q

Which of the following would be accounted for as a change in accounting principle and treated retrospectively?

A
A change in the method of inventory pricing, such as from LIFO to FIFO
B
A change in depreciation method, such as from the double-declining-balance method to the straight-line method
C
A change in the amortization method for an intangible asset
D
All of the above

A

Explanation:
The correct answer is (A).

A change in the method of inventory pricing is accounted for as a change in accounting principle and treated retrospectively.

Per SFAS 15 changes in depreciation, amortization, and depletion methods are treated as changes in accounting estimates affected by a change in accounting principle and treated prospectively.

212
Q
Fogg Co., a U.S. company, contracted to purchase foreign goods. Payment in foreign currency was due one month after the goods were received at Fogg's warehouse. Between the receipt of goods and the time of payment, the exchange rates changed in Fogg's favor. The resulting gain should be included in Fogg's financial statements as a(an)
A
Component of income from continuing operations.
B
Extraordinary item.
C
Deferred credit.
D
Item of other comprehensive income.
A

Explanation:
A change in exchange rates between the functional currency and the currency in which the transaction is denominated increases or decreases the expected amount of functional currency cash flows upon a settlement of the transaction. That increase or decrease in expected functional currency cash flows is a foreign currency transaction gain or loss that generally should be included as a component of income from continuing operations for the period in which the transaction is settled.

213
Q

The following information pertains to a sale and leaseback of equipment by Mega Co. on December 31 of the current year:

Sales Price	$400,000
Carrying amount	300,000
Monthly lease payment	3,250
Present value of lease payments	36,900
Estimated remaining life	25 years
Lease term	1 year
Implicit rate	12%
What amount of gain on the sale should Mega report at December 31 of the current year?
A
$0
B
$36,900
C
$63,100
D
$100,000
A

Explanation:
Mega concludes that the transfer of the tower assets is a sale. Mega’s sale price to be $400,000 and CV is $300,000. The Journal entry to record sale is:

Cash 400,000
Asset 300,000
Gain on Sale 100.000
Option (A), (B) and (C) is incorrect as per above explanation.

214
Q
The following information pertains to shipments of merchandise from Home Office to Branch during the current year:
Home Office's cost of merchandise	$160,000
Intracompany billing	200,000
Sales by Branch	250,000
Unsold merchandise at Branch on December 31	20,000In the combined income statement of Home Office and Branch for the year ended December 31, what amount of the above transactions should be included in sales
A
$250,000
B
$230,000
C
$200,000
D
$180,000
A

Explanation:
Any sale of inventory between a home office and a branch will trigger the individual accounting systems of both units. Revenue is recorded by the seller while the purchase is simultaneously entered into the acquirer’s accounts. However, from a combined perspective, neither sale nor purchase has occurred. Thus, only the $250,000 of sales by Branch should be included in the combined income statement; the intracompany billing of $200,000 should be eliminated in preparing the combined statement.

215
Q

The disclosure requirement associated with the date through which management has evaluated subsequent events, along with the basis for that date being the appropriate date, needs to be implemented in financial statements without regard to which of the following?

A
Whether the statements have been compiled.
B
Whether the statements have been prepared using an Other Comprehensive Basis of Accounting.
C
Whether the statements are full disclosure.
D
Both A. and B.

A

Explanation:
The correct answer is (D).

The disclosure requirement associated with the date through which management has evaluated subsequent events, along with the basis for that date being the appropriate date, needs to be implemented in full-disclosure financial statements without regard to whether the statements have been audited, reviewed, or compiled.

Further, the disclosure needs to be made without regard to whether the statements are prepared using U.S. GAAP, or an Other Comprehensive Basis of Accounting (OCBOA).

Essentially, the disclosure is required regardless of the level of service performed on the financial statements, and regardless of the basis of accounting used in preparing the statements.

216
Q

GAAP provides sample situations where the 20-percent ownership presumption may be overcome. Which of the following is an (are) example(s) of this?

A
Opposition by the investee
B
The investor and investee sign an agreement under which the investor surrenders significant rights as a shareholder
C
The investor tries and fails to obtain representation on the investee's board of directors
D
All of the above
A

Explanation:
Sample situations where the 20 percent ownership presumption may be overcome include: opposition by the investee, such as litigation or complaints to governmental regulatory authorities, challenges the investor’s ability to exercise significant influence; the investor and investee sign an agreement under which the investor surrenders significant rights as a shareholder; majority ownership of the investee is concentrated among a small group of shareholders who operate the investee without regard to the views of the investor; or the investor needs or wants more financial information to apply the equity method than is available to the investee’s other shareholders, but fails in her/his attempts to obtain that information. Since answers A., B., and C. are incorrect, answer D. is the
best choice.

217
Q

Giaconda, Inc. acquires an asset for which it will measure the fair value by discounting future cash flows of the asset. Which of the following terms best describes this fair value measurement approach?

A
Market
B
Income
C
Cost
D
Observable inputs
A

Explanation:
Using present value techniques to discount the cash flows or earnings is called the income approach.
Option (A) is incorrect because market approach uses prices and relevant information from market transactions for identical or comparable assets/liabilities.
Option (C) is incorrect because cost approach uses current replacement cost.
Option (D) is incorrect because observable inputs are the inputs for FV valuation and not a valuation technique.

218
Q

On September 1, year 1, Phillips, Inc. issued common stock in exchange for 20% of Sago, Inc.’s outstanding common stock. On July 1, year 3, Phillips issued common stock for an additional 75% of Sago’s outstanding common stock. Sago continues in existence as Phillips’ subsidiary. How much of Sago’s year 3 net income should be reported as accruing to Phillips?
A
20% of Sago’s net income to June 30 and all of Sago’s net income from July 1 to December 31.
B
20% of Sago’s net income to June 30 and 95% of Sago’s net income from July 1 to December 31.
C
95% of Sago’s net income.
D
All of Sago’s net income.

A

Explanation:
When a transaction is accounted for as a business combination under the acquisition method, the parent company accrues its equity in the subsidiary’s earnings that occur subsequent to the date of the investment. Therefore, in year 3, Phillips should accrue 20 percent of Sago’s net income from January 1 to June 30, and 95 percent of Sago’s net income from July 1 to December 31.

219
Q

Cuthbert Industrials, Inc. prepares three-year comparative financial statements. In year 3, Cuthbert dis­covered an error in the previously issued financial statements for year 1. The error affects the financial statements that were issued in years 1 and 2. How should the company report the error?

A
The financial statements for years 1 and 2 should be restated; an offsetting adjustment to the cumulative effect of the error should be made to the comprehensive income in the year 3 financial statements.
B
The financial statements for years 1 and 2 should not be restated; financial statements for year 3 should disclose the fact that the error was made in prior years.
C
The financial statements for years 1 and 2 should not be restated; the cumulative effect of the error on years 1 and 2 should be reflected in the carrying amounts of assets and liabilities as of the beginning of year 3.
D
The financial statements for years 1 and 2 should be restated; the cumulative effect of the error on years 1 and 2 should be reflected in the carrying amounts of assets and liabilities as of the beginning of year 3.

A

Explanation:
Any correction of error in the accounting statements of prior years’ calls for a retroactive restatement. If comparative Financial Statements are presented and Financial Statement for the year with the error is presented, then corrections of errors should be made in those prior Financial Statements. Therefore, the Financial Statement of Cuthbert Industrials for years 1 & 2 should be restated, the cumulative effect of the error on years 1 and 2 should be reflected in the carrying amounts of assets and liabilities as of the beginning of year 3.

Options (A), (B) and (C) are incorrect as per the above explanation .

220
Q

Which of the following items would most likely require a subsequent event adjustment to the financial statements for the year ended December 31, year 1?
A
Uninsured loss of inventories purchased in year 1 as a result of a flood in year 2
B
Settlement of litigation in year 2 over an event that occurred in year 2
C
Loss on an uncollectible trade receivable recorded in year 1 from a customer that declared bankruptcy in year 2 before the financial statements were issued
D
Proceeds from a capital stock issuance in year 2 which was being approved by the board of directors in year 1

A

Explanation:
There are two types of subsequent events: recognized and nonrecognized. The first type requires adjustment of the financial statements because it provides evidence about conditions that existed at the balance sheet date. The second type of subsequent event provides evidence about conditions that did not exist at the balance sheet date and does not require adjustment of the financial statements; however, if material in nature, the event(s) may require disclosure to keep the financial statements from being misleading.

Loss on an uncollectible trade receivable recorded in year 1 from a customer that declared bankruptcy in year 2 is a classic example of providing evidence about conditions that existed at the balance sheet date and thus it requires a financial statement adjustment.

221
Q

On December 15, a U.S. company bought inventory from a European supplier. Payment is required in euros in 30 days. What exchange rate should be used to value the payable for this transaction at year-end?

A
Exchange rate at settlement date.
B
Exchange rate at purchase date.
C
Exchange rate at year-end.
D
Weighted-average exchange rate for the year.
A

Explanation:
The correct answer is (C)

An organization which purchases or sells goods in currencies other than in its functional currency is required to recognize foreign currency gain/loss on them at the settlement date using the spot rate. Where the settlement date is after the year-end, the foreign currency gain/loss is recognized at such year-end using the exchange rate at year-end.

222
Q
A company’s foreign subsidiary operation maintains its financial statements in the local currency. The foreign operation’s capital accounts would be translated to the functional currency of the reporting entity using which of the following rates?
A
Historical exchange rate
B
Functional exchange rate
C
Weighted-average exchange rate
D
Current exchange rate at the balance sheet date
A

Explanation:
Prior to translation, the foreign currency statements must be conformed to US GAAP and be measured in the functional currency of the foreign entity (otherwise, remeasurement into the functional currency is required). The remeasuring process should achieve the same result as if the books had been initially recorded in the functional currency. This requires the remeasuring of certain accounts (nonmonetary items) at historical exchange rates, including the foreign operations’ capital accounts. All other accounts are remeasured at current rates.

223
Q
Mr. and Mrs. Gasson own 100% of the common stock of Able Corp. and 90% of the common stock of Baker Corp. Able previously paid $4,000 for the remaining 10% interest in Baker. The condensed December 31 balance sheets of Able and Baker are as follows:
Able	Baker
Assets	$600,000	$60,000
Liabilities	200,000	30,000
Common stock	100,000	20,000
Retained earnings	300,000	10,000
$600,000	$60,000In a combined balance sheet of the two corporations at December 31, what amount should be reported as total stockholders' equity?
A
$430,000
B
$426,000
C
$403,000
D
$400,000
A

Explanation:
The amount to be reported as total stockholders’ equity in a combined balance sheet at December 31, is determined as follows:

Common stock ($100,000 + $20,000) $120,000
Retained earnings ($300,000 + $10,000) 310,000
Subtotal 430,000
Less Able’s cost of 10% interest in Baker 4,000
Stockholders’ equity in combined balance sheet $426,000

224
Q

Developing and publishing which of the following is not one of the stages in the due process the IASB uses is developing standards?

A
A Discussion Paper
B
A Comment Letter
C
An Exposure Draft
D
An IFRS
A

Explanation:
Developing and publishing a Comment Letter is not one of the stages in the due process the IASB uses in developing standards. The development of comment letters is done by users in response to the IASB. The due process comprises the following six stages, performing research; developing and publishing the discus­sion paper; presenting a proposal; developing and publishing the Exposure Draft; developing and publishing the IFRS; and post-implementation review after the standard is issued.

225
Q

The fair value for an asset or liability is measured as

A
The appraised value of the asset or liability.
B
The price that would be paid to acquire the asset or received to assume the liability in any type of transaction between two parties.
C
The price that would be received when selling an asset or paid when transferring a liability in an orderly transaction between market participants.
D
The cost of the asset less any accumulated depreciation or the carrying value of the liability on the date of the sale.

A

Explanation:
The fair value is defined as the price that would be received to sell an asset or paid to transfer a liability in an orderly arm’s length transaction between market participants at the measurement date (at exit price).
Option (A) is incorrect because the appraised value is not the Fair Value (FV).
Option (B) is incorrect because FV is the exit price and not the entry price.
Option (D) is incorrect because the carrying value is the historical cost and not the FV.

226
Q

If a company that is not a public business entity wants to apply the simplified hedge accounting approach to a cash flow hedge of a variable rate borrowing with a receive-variable, pay-fixed interest rate swap, which of the following is a condition that must be met?

A
The notional value of the swap is greater than the principal of the hedged borrowing
B
The fair value of the interest rate swap executed has a value equivalent to the hedged borrowing
C
The variable interest rate on the interest rate swap is capped at 250 basis points above the cap on the hedged borrowing
D
The variable interest rate on the interest rate swap and the variable interest rate on the hedged borrowing are linked to the same index

A

Explanation:
The correct answer is (D).

For a non-issuer (i.e., not a public business entity) to apply the simplified hedge accounting approach to such a cash flow hedge, the variable interest rate on the interest swap and the variable interest rate on the hedged borrowing must be linked to the same index.

227
Q

Which of the following is a characteristic of a Finance lease?

A
Leased asset is non-specialized in nature
B
Lessee has an option to purchase the underlying asset that the lessee is reasonably certain to exercise
C
The lease transfers ownership of the underlying asset to the lessor at the end of the lease term
D
The present value of the lease payments, plus the residual value guaranteed equals or exceeds 35% the fair value of the asset.

A

Explanation:
The correct answer is (B):

Lessee has an option to purchase the underlying asset that the lessee is reasonably certain to exercise

A Finance Lease must meet one of the following criteria:

Present Value equals or exceeds substantially all (90%) of the Fair Value
Option to Purchase (exercise is reasonably certain)
Economic Life - Major part (75%) of asset’s economic life is used
Transfer of Ownership at lease termination
Specialized Nature - No alternative use to the lessor at lease termination
Note: the implementation guidance for ASC 842 uses the 75/90 thresholds, even though the standard is principles-based.

228
Q

During the current year ended December 31, Metal, Inc. incurred the following costs:

Laboratory research aimed at discovery of new knowledge $ 75,000
Design of tools, jigs, molds, and dies involving new technology 22,000
Quality control during commercial production, including routine testing 35,000
Equipment acquired two years ago, having an estimated useful life of
five years with no salvage value, used in various R & D projects 150,000
Research and development services performed by Stone Co. for Metal, Inc. 23,000
Research and development services performed by Metal, Inc. for Clay Co. 32,000
What amount of research and development expenses should Metal report in its current-year income statement?

A
$120,000
B
$150,000
C
$187,000
D
$217,000
A

Explanation:
The R&D costs include:

New Knowledge or new technology.
Model or prototype.
Application of new research findings.
In the current year, Metal Inc., will recognize the R&D Expenses of $150,000

Ref

Summary

Amount

a

Equipment purchased 2 years ago

$150,000

b

Useful life of the equipment

5

c

Depreciation of equipment (a/b)

$30,000

d

Laboratory research aimed at discovery of new knowledge

$75,000

e

Design of tools, jigs, molds and dies involving new technology

$22,000

f

R&D services performed by Stone Co. for Metal,Inc.

$23,000

g

Expense for the current year (c+d+e+f)

$150,000

        R&D performed for Clay Co., and quality control costs during commercial production are not treated as R&D costs.Option (a) is incorrect because it does not include depreciation expense on equipment purchased 2 years ago. $120,000 = ($75,000 + $22,000 + $35,000).Option (c) is incorrect because it does not include depreciation expense on equipment. It incorrectly includes the quality control costs during commercial production and R&D performed for Clay co., which are not treated as R&D costs $187,000 = ($75,000 + $22,000 +$35,000 + $23,000 + 32,000). Option (d) is incorrect because it includes the quality control costs during commercial production and R&D performed for Clay co., which are not treated as R&D costs [$217,000 = $75,000 + $22,000 +$35,000 + $23,000 + 32,000 +(150,000/5)].
229
Q

Robbins Inc. leased a machine from Ready Leasing Co. The lease qualifies as a finance lease and requires 10 annual payments of $10,000 beginning immediately. The lease specifies an interest rate of 12% and a purchase option of $10,000 at the end of the tenth year, even though the machine’s estimated value on that date is $20,000. Robbins’ incremental borrowing rate is 14%.

The present value of an annuity due of $1 at: The present value of $1 at:
12% for 10 years is 6.328 12% for 10 years is .322
14% for 10 years is 5.946 14% for 10 years is .270
What amount should Robbins record as lease liability at the beginning of the lease term?

A
$62,160
B
$64,860
C
$66,500
D
$69,720
A

Explanation:
The correct answer is (C).

A Finance Lease must meet one of the following criteria:

Present Value equals or exceeds substantially all (90%) of the Fair Value
Option to Purchase (exercise is reasonably certain)
Economic Life - Major part (75%) of asset’s economic life is used
Transfer of Ownership at lease termination
Specialized Nature - No alternative use to the lessor at lease termination
Note: the implementation guidance for ASC 842 uses the 75/90 thresholds, even though the standard is principles-based.

The lessee records a finance lease as ROU asset and as lease liability calculated as the present value of the lease payments not yet paid by using the lease term and discount rate determined at lease commencement. The lease payments include the fixed payments called for by the lease over the lease term and payment called for by the purchase option reasonably certain to be exercised. The purchase option is reasonably certain to be exercised because the lessee is permitted to purchase the leased property for a price which is significantly lower than the expected fair value of the property at the option’s exercise date (i.e., $10,000 < $20,000).

Since the annual lease payment is payable at the beginning of each lease year, the PV factor for an annuity due is used. The PV of the lease payments is computed using the lessor’s implicit interest rate of 12%.

Present value of annual payments $63,280
PV of purchase option ($10,000 × 0.322) 3,220
Finance lease liability at beginning of lease term $66,500
Options (A) is incorrect because the incremental borrowing rate of 14% is used instead of the rate implicit in the lease of 12%.

Options (B) and (D) are incorrect because of inaccurate calculations.

230
Q

Historically, where separate incorporation is maintained, the subsidiary’s financial records are which of the following?
A
Not affected by either the acquisition or the consolidation
B
Affected by the acquisition, but not the consolidation
C
Not affected by the acquisition, but affected by the consolidation
D
Affected by the acquisition and the consolidation

A

Explanation:
Historically, where separate incorporation is maintained, the subsidiary’s financial records are not affected by either the acquisition or the consolidation.

231
Q

Which of the following phrases best describes a Level 1 input for measuring the fair value of an asset or liability?

A
Inputs for the asset or liability based on the reporting entity’s internal data.
B
Quoted prices for similar assets or liabilities in active markets.
C
Inputs that are principally derived from or corroborated by observable market data.
D
Unadjusted quoted prices for identical assets or liabilities in active markets.

A

Explanation:
The correct answer is (D).

There are 3 levels of input for measuring the fair value of an asset or liability.

Level 1- Unadjusted quoted prices for identical assets or liabilities in active markets.

Level 2: Inputs other than quoted prices included within Level 1 that are observable for similar assets or liabilities, either directly or indirectly.

Level 3: Unobservable inputs for the asset or liability.

(A), (B) and (C) are incorrect because they are Level 2 inputs.

232
Q
Martin Pharmaceutical Co. is currently involved in two lawsuits. One is a class-action suit in which consumers claim that one of Martin's best selling drugs caused severe health problems. It is reasonably possible that Martin will lose the suit and have to pay $20 million in damages. Martin is suing another company for false advertising and false claims against Martin. It is probable that Martin will win the suit and be awarded $5 million in damages. What amount should Martin report on its financial statements as a result of these two lawsuits?
A
$0
B
$5 million income.
C
$15 million expense.
D
$20 million expense.
A

Explanation:
Reasonably possible means more than remote, but less than probable. Where the loss is considered reasonably possible, no charge should be made to income but the nature of the contingency should be disclosed. This treatment also applies to probable losses that cannot be reasonably estimated. Gain contingencies should be disclosed but not recognized as income. Care should be taken to avoid misleading implications as to the likelihood of realization. Therefore, Martin should report $0 on its financial statements, but disclose both events in the notes.

233
Q

Able Co. leased equipment to Baker under a noncancellable lease with a transfer of title. Will Able record depreciation expense on the leased asset and interest revenue related to the lease?

Depreciation expense	Interest revenue
A	Yes	Yes
B	Yes	No
C	No	No
D	No	Yes
A

Explanation:
A non-cancelable lease with a transfer of title is considered a finance lease.

A Finance Lease must meet one of the following criteria:

Present Value equals or exceeds substantially all (90%) of the Fair Value
Option to Purchase (exercise is reasonably certain)
Economic Life - Major part (75%) of asset’s economic life is used
Transfer of Ownership at lease termination
Specialized Nature - No alternative use to the lessor at lease termination
Note: the implementation guidance for ASC 842 uses the 75/90 thresholds, even though the standard is principles-based.

The lessor, Able, does not record depreciation expense. Instead, the lessee, Baker, would amortize the equipment. Able would record interest revenue related to the lease whether it was treated as a sales-type lease or direct financing-type lease.

234
Q
The following information relates to two projects performed by Miley Co. during the year for laboratory research aimed at discovering new knowledge:
Project Cost	Likelihood that effort will result in future benefits
I. $100,000	Probable
II. $50,000	Reasonably possibleWhat should Miley report as research and development expenses in its income statement for the year?
A
$0
B
$50,000
C
$100,000
D
$150,000
A

Explanation:
Research activities are those aimed at the discovery of knowledge that will be useful in developing or significantly improving products or processes. Future economic benefits from research and development (R&D) activities, if any, are uncertain in their amount and timing. Due to these uncertainties, most R&D costs are required to be charged to expense the year in which incurred. The likelihood the research projects performed by Miley will result in future benefits is insignificant, all the costs are expensed.

235
Q

How should the effect of a change in accounting principle that is inseparable from the effect of a change in accounting estimate be reported?
A
As a component of income from continuing operations
B
By restating the financial statements of all prior periods presented
C
As a correction of an error
D
By footnote disclosure only

A

Explanation:
When a change in accounting estimate and a change in accounting principle are inseparable, the change should be accounted for as a change in estimate, which is a component of income from continuing operations. A change in estimate does not require the restatement of prior period financial statements. Corrections of errors are considered prior period adjustments and prior-period financial statements are restated. Material effects of a change in estimate on income, and related per share amounts should be disclosed. Footnote disclosure only is not proper accounting treatment of a change in estimate.

236
Q

On January 2 of the current year, LTTI Co. entered into a three-year, non-cancelable contract to buy up to 1 million units of a product each year at $.10 per unit with a minimum annual guarantee purchase of 200,000 units. At year end, LTTI had only purchased 80,000 units and decided to cancel sales of the product. What amount should LTTI report as a loss related to the purchase commitment as of December 31 of the current year?

A
$0
B
$8,000
C
$12,000
D
$52,000
A

Explanation:
When there is a firm commitment to purchase goods in a future period at a set price (i.e., an enforceable contract exists), any loss from a drop in the market value of such goods, or cancellation of the contract by the purchaser, should be recognized in the current period. The contract guaranteed LTTI purchase 200,000 units per year, so LTTI would recognize a loss for the 520,000 units that it had guaranteed to purchase throughout the life of the contract but had not yet purchased (120,000 current year units + 200,000 year 2 units + 200,000 year 3 units = 520,000 units). 520,000 × $0.10 = $52,000.

Option (A) is incorrect as per the above explanation.

Option (B) is incorrect because $8,000 is the current year purchases of 80,000 units at $0.10.

Option (C) is incorrect because it is the loss for year 1 alone, (200,000 units - 80,000 units) 120,000 units at $0.10, while the loss is recognized for all the 3 years in the current year, as per conservative principle of recognizing losses.

237
Q

Matt Co. included a foreign subsidiary in its consolidated financial statements. The subsidiary was acquired six years ago and was excluded from previous consolidations. The change was caused by the elimination of foreign exchange controls. Including the subsidiary in the consolidated financial statements results in accounting change that should be reported
A
By footnote disclosure only.
B
Currently and prospectively.
C
Currently with footnote disclosure of pro forma effects of retroactive application.
D
By retrospective application to the financial statements of all prior periods presented.

A

Explanation:
Since the subsidiary was excluded from previous consolidations, its inclusion in the current year’s consolidated financial statements results in a change in accounting principle in which the financial statements, in effect, are those of a different reporting entity. The change in the reporting entity should be reported by retrospective application to the financial statements of all prior periods presented to reflect the new reporting entity.

238
Q
Blythe Corp. is a defendant in a lawsuit. Blythe's attorneys believe it is reasonably possible that the suit will require Blythe to pay a substantial amount. What is the proper financial statement treatment for this contingency?
A
Accrued and disclosed.
B
Accrued but not disclosed.
C
Disclosed but not accrued.
D
No disclosure or accrual.
A

Explanation:
Where the loss is considered reasonably possible, no charge should be made to income but the nature of the contingency should be disclosed. This treatment also applies to probable losses that cannot be reasonably estimated.

239
Q

On January 1 of the current year, Park Co. signed a 10-year operating lease for office space at $96,000 per year. The lease included a provision for additional rent of 5% of annual company sales in excess of $500,000. Park’s sales for the year ended December 31 were $600,000. Upon execution of the lease, Park paid $24,000 as a bonus for the lease. Park’s rent expense for the current year ended December 31

A
$98,400
B
$101,000
C
$103,400
D
$125,000
A

Explanation:
Accrual accounting recognizes expense in the period incurred, rather than only when the related cash is paid. The lease bonus is allocated ratably over the lease term as part of rent expense.

Equal annual rental payment $96,000
Amortization of lease bonus ($24,000 / 10) 2,400
High sales premium [($600,000 - $500,000) × 5%] 5,000
Rental expense $103,400

240
Q

An entity disposes of a nonmonetary asset in a nonreciprocal transfer. A gain or loss should be recognized on the disposition of the asset when the fair value of the asset transferred is determinable and the nonreciprocal transfer is to
Another entity A stockholder of the entity
A No Yes
B No No
C Yes No
D Yes Yes

A

Explanation:
A transfer of a nonmonetary asset to a stockholder or to another entity in a nonreciprocal transfer should be recorded at the fair value of the asset transferred, and a gain or loss should be recognized on the disposition of the asset.

241
Q

On January 1, year 1, a company capitalized $100,000 of costs for software that is to be sold. The company amortizes the software costs on a straight-line basis over five years. The carrying value of the software costs on January 1, year 3, was $60,000. As of December 31, year 3, the estimated future gross revenue to be generated from the sale of the software is $23,000, and the estimated future cost of disposing of the software is $8,000. What amount should the company expense related to the software costs for the year ended December 31, year 3?

A
$18,400
B
$20,000
C
$37,000
D
$45,000
A

Explanation:
The correct answer is (D).

Software production costs are capitalized and reported at the lower of unamortized cost or net realizable value (NRV) at the point of technological feasibility for the software. Net realizable value is the value of an asset that can be realized upon the sale of the asset, less a reasonable estimate of the costs associated with either the eventual sale or the disposal of the asset in question.

Depreciation on software = Capitalized cost of software/Useful life = $100,000/5 = $20,000.
Total expenses related to software costs are depreciation charges and excess of the CV over the NRV reported at $45,000 ($20,000 + $25,000).
Cost of the software purchased 2 years ago $100,000
Carrying value of the software at the beginning of the year ($100,000 - $20,000 - $20,000) $60,000
Depreciation for the year ($20,000) $20,000
Carrying value at the end of the year ($60,000 - $20,000) $40,000
Expected gross revenue from sale of the software $23,000
Expected future costs to dispose the software $8,000
Net realizable value (NRV) ($23,000 - $8,000) $15,000
Excess of carry value over the NRV ($40,000 - $15,000) $25,000
Total expense for the year ($20,000 Depreciation + $25,000 Impairment). $45,000

242
Q
On January 1, year 3, Pell purchased a machine for $700,000 having an estimated useful life of 10 years and no salvage. The machine was depreciated using double-declining balance for both financial statement and income tax reporting. On January 1, year 6, Pell changed from double-declining balance to the straight-line method of depreciation for both financial statement and income tax reporting. Accumulated depreciation at December 31, year 5, was $341,600. If the straight-line method had been used, the accumulated depreciation would have been $210,000. The amount shown at December 31, year 6, for accumulated depreciation was:
A
$413,080
B
$392,800
C
$210,000
D
$140,000
A

Explanation:
A change in depreciation methods is treated prospectively. The remaining book value of $358,400 ($700,000 - 341,600) is depreciated over the remaining life (7 years), or $51,200 per year. Total accumulated depreciation at the end of year 6 is $341,600 + 51,200 = $392,800.

243
Q

Which of the following is a true statement related to foreign currency accounting and reporting under US GAAP and IFRS?

A
Only IFRS requires the identification of hyperinflationary economies.
B
Both US GAAP and IFRS require foreign currency transactions to be re-measured into an entity’s functional currency with amounts resulting from changes in exchange rates being reported in other comprehensive income.
C
Both US GAAP and IFRS require the “step-by-step” method of consolidation in reporting the consol-idation of foreign operations.
D
Both US GAAP and IFRS require re-measurement into the functional currency before translation into the reporting currency.

A

Explanation:
Both US GAAP and IFRS require re-measurement into the functional currency before translation into the reporting currency. Assets and liabilities are translated at the period-end rate and income statement amounts generally are translated at the average rate, with the exchange differences reported in equity. Both US GAAP and IFRS also require the identification of hyperinflationary economies and foreign currency transactions to be re-measured into an entity’s functional currency with amounts resulting from changes in exchange rates being reported in net income, not other comprehensive income. Under US GAAP, the “step-by-step” method of consol­idation is used when reporting the consolidation of foreign operations, and under IFRS, either the “direct” or the “step-by-step” method of consolidation may be used.

244
Q

GAAP defines ___________________ as any legal structure used to conduct activities or to hold assets.

A
An entity
B
A variable interest entity
C
A business
D
A special purpose entity
A

Explanation:
An “entity” is any legal structure used to conduct activities or to hold assets. A “variable interest entity” is defined based on the characteristics of the entity. To be considered a business, the entity must meet the requirements set forth in GAAP. A special purpose entity (SPE) is not recognized in US GAAP.

245
Q

Brand Co. incurred the following research and development project costs at the beginning of the current year:

Equipment purchased for current and future projects $100,000
Equipment purchased for current projects only 200,000
Research and development salaries for current project 400,000
Equipment has a five-year life and is depreciated using the straight-line method. What amount should Brand record as depreciation for research and development projects at December 31?

A
$ 0
B
$ 20,000
C
$ 60,000
D
$ 140,000
A

Explanation:
Materials,equipment,facilities,or intangibles that are acquired for a current R&D project and have no alternative future use in other R&D projects should be expensed in the period in which acquired.If alternative future uses are expected, whether in other R&D activities or in normal operations ,these items should be recorded as assets and the cost should be amortized over their useful lives by periodic charges to R&D expense.If, at any point, these assets are no longer deemed to have alternative future uses, the remaining unamortized cost is charged to R&D expense for the period. Depreciation expense would be $20,000($100,000/5years). Option (a) is incorrect because the equipment used for current and future projects should be capitalized and depreciation expense should be recorded over 5 years using SLM.Option (c) is incorrect because it capitalized the equipment purchased for the current projects and depreciated. [$60,000 = ($100,000 + $200,000)/5)]. Option (d) is incorrect because it capitalized the equipment purchased for the current projects and R&D salaries for current project, and depreciated over 5 years.$140,000 = ($100,000 + $200,000 + $400,000)/5].

246
Q

On January 1, year 1, Warren Co. purchased a $600,000 machine, with a five-year useful life and no salvage value. The machine was depreciated by an
accelerated method for book and tax purposes. The machine’s carrying amount was $240,000 on December 31, year 2. On January 1, year 3, Warren changed
retroactively to the straight-line method for financial statement purposes. Warren can justify the change. Warren’s income tax rate is 30%. In its year 3 income
statement, what amount should Warren report as a prior period adjustment as result of this change?

A
$120,000
B
$ 84,000
C
$ 36,000
D
$0
A

Explanation:
A change in depreciation method, such as from the accelerated to the straight-line method is a change in accounting principle that is inseparable from a
change in accounting estimate. When the effects of the two changes cannot be separated, the change should be treated as a change in estimate. A change in
accounting estimate should be accounted for in the period of change if the change only affects that period, or in the current and subsequent periods, if the
change affects both, as a component of income from continuing operations. No prior period adjustment is made.

247
Q

Concerning combined financial statements, which of the following statements is true?
A
There are circumstances where combined financial statements of commonly controlled companies are likely to be more meaningful than their separate statements.
B
Combined financial statements are often prepared for a group of related companies or a group of commonly controlled companies.
C
Combined financial statements are prepared by combining the individual companies’ financial statement classifications into one set of financial statements.
D
All of the above are true.

A

Explanation:
There are circumstances where combined financial statements of commonly controlled companies are likely to be more meaningful than their separate statements. Combined financial statements are often prepared for a group of related companies or a group of commonly controlled companies. Combined financial statements are prepared by combining the individual companies’ financial statement classifications into one set of financial statements. Since answers A., B., and C. are all correct, answer D., all of the above are true, is the best choice.

248
Q

On January 1, year 1, a company with a calendar year-end began developing a software program that it intends to market and sell to its customers. The software coding was completed on March 31, year 1, at a cost of $200,000, and the software testing was completed on June 30, year 1, at a cost of $100,000. The company achieved technological feasibility on July 31, year 1, at which time the company began producing product masters at a cost of $125,000. What amount should the company report for the total research and development expense for the year ended December 31, year 1?

A
$100,000
B
$200,000
C
$300,000
D
$425,000
A

Explanation:
The correct answer is (C).

Expense all the costs incurred until the point where technological-feasibility is established as research and development cost.
Capitalize all costs incurred to convert a technologically-feasible program into a final commercial product.
Expense as a cost of goods sold (COGS) costs incurred after software sale begins.
The company achieved technological feasibility on July 31, year 1. All costs incurred before this date will be expensed as research and development cost.

Software Coding Cost incurred till March 31, year 1 $200,000
Software Testing Costs incurred till June 30, year 1 $100,000
Total Research and Development cost $300,000.

249
Q

For a sale to occur in the context of a sale and leaseback transaction, all of the following criteria must be met except

A
A contract exists.
B
The seller-lessee satisfies its performance obligation by transferring control of assets to the buyer-lessor.
C
Leaseback must be an operating lease.
D
The implicit rate of the lease is verified by an independent third-party
A

Explanation:
The correct answer is (D)

Per ASC-842 Transactions must meet the below criteria to qualify as a Sale-Leaseback transaction.

A contract exists that complies with the provisions of ASC Topic 606: “Revenues from Contracts with Customers.”
The seller-lessee satisfies its performance obligation by transferring control of assets to the buyer-lessor.
Leaseback must be an operating lease.
An option for the seller-lessee to repurchase the asset would preclude accounting for the transfer of the asset as a sale of the asset unless both of the following criteria are met:
The exercise price of the option is the fair value of the asset at the time the option is exercised.
There are alternative assets, substantially the same as the transferred asset, readily available in the marketplace.

250
Q

A company leases a machine from Leasing, Inc. on January 1, year 1. The lease terms include a $100,000 annual payment beginning January 1, year 1. The machine’s fair value is $500,000 and the residual value is estimated at $20,000. The company guarantees the residual value. The useful life of the machine is six years, and the lease term is five years. The implicit rate of interest is 6% and is known by the company. The following present value factors are provided:

Five years Six years

Present value of $1 at 6% 0.7473 0.705
Present value of an annuity due at 6% 4.4651 5.2124
Present value of an ordinary annuity at 6% 4.2124 4.9173
What is the value of the machine in the company’s balance sheet at lease inception?

A
$446,510
B
$461,456
C
$520,000
D
$535,340
A

Explanation:
The correct answer is (B).

With a lease term equal to 5 years of the asset’s 6 years useful life, as the lease term is for the major part of the remaining economic life of the underlying asset, the lease is a finance lease and will result in the recognition of an asset and a liability equal to the present value of the minimum lease payments.

The guaranteed residual value on the machine is a one-time payment at the end of the lease term calculated at the PV of $1 at 6% for 5 years.

Annual payments for leasing the machine $100,000
PV of an annuity at 6% for 5 Years 4.4651
PV of annual payments $446,510
Residual value of machine $20,000
PV 0f $1 at 6% 0.7473
PV of residual value of the machine $14,946
Value of machine at lease inception
(PV of annual payments + PV of residual value of the machine) $461,456
(A) is incorrect because it does not consider PV of the residual value at the end of the lease term of $14,946.

(C) is incorrect because it does not consider the PV of the annual payments and residual value.

(D) is incorrect because it considers the PV factors for annuity and PV of the $1 for 6 years instead of 5 years factors.

251
Q

Lore Co. changed from the cash basis of accounting to the accrual basis of accounting during the year. The cumulative effect of this change should be reported in Lore’s financial statements as a
A
Correction of an error.
B
Change in accounting principle.
C
Component of income before extraordinary item.
D
Component of income after extraordinary item.

A

Explanation:
The change from the cash basis of accounting (not GAAP) to the accrual basis of accounting (GAAP) is a correction of an error. The correction of an error in prior period income is reported as a prior-period adjustment by restating prior-period financial statements.

252
Q

Which of the following is an example of activities that would typically be excluded in research and development costs?
A
Design, construction, and testing of preproduction prototypes and modes
B
Laboratory research aimed at discovery of new knowledge
C
Quality control during commercial production, including routine testing of products
D
Testing in search for, or evaluation of, product or process alternatives

A

Explanation:
Quality control during commercial production including routine testing of products is an example of an activity typically excluded in research and development costs. Laboratory research aimed at discovery of new knowledge, testing in search for or evaluation of product or process alternatives design, construction, and testing of pre-production prototypes and models are examples of activities typically included in research and development costs.

253
Q

Clay Company leased a new machine from Saxe Corp. The following data relate to the lease transaction at the inception of the lease:

Lease term 10 years
Lease payment due at the beginning of each year $50,000
Useful life of machine 15 years
Implicit interest rate 10%
Present value of an annuity due of $1 for 10 periods at 10% 6.76
Present value of an ordinary annuity of $1 for 10 periods at 10% 6.15
Fair value of the machine $400,000
The lease has no renewal option, and the possession of the machine reverts to Saxe when the lease terminates. At the inception of the lease, Clay should record a lease liability of

A
$400,000
B
$0
C
$307,500
D
$338,000
A

Explanation:
The correct answer is (D).

Per ASC 842, at lease commencement, the lessee must measure their lease assets and liabilities at the present value of the lease payments using either the lessor’s implicit interest rate or if not readily determinable, at lessee’s incremental borrowing rate.

Since lease payments are payable at beginning of each year, the Present value of an annuity due of $1 is used for 10 periods at 10% (the implicit interest rate is given), which is 6.76. That gives the lease liability of $50,000 x 6.76 = $338,000.

Option (A), (B) and (C) are incorrect as per above explanation.

254
Q

On January 1, year 3, a company changed its inventory costing method from LIFO to FIFO. The company’s year 3 financial statements contain comparative information for year 2. How should the company present the year 1 effect of the change in accounting principle in its year 3 comparative financial statements?

A
As an adjustment to the beginning year 2 inventory balance with an offsetting adjustment to beginning year 2 retained earnings
B
As part of income from continuing operations in the year 2 income statement
C
As a discontinued operations item in the year 2 income statement
D
As a note disclosure only

A

Explanation:
The correct answer is (A).

On January 1, year 3, a company changed its inventory costing method from LIFO to FIFO. The company’s year 3 financial statements contain comparative information for year 2. This is a change in accounting principle with retrospective application of the new accounting principle to all prior periods. Financial Statements for each individual prior period presented shall be adjusted to reflect the period-specific effects of applying the new accounting principle. For the retrospective application of the change in accounting principle, an adjustment to the beginning year 2 inventory balance with an offsetting adjustment to beginning year 2 retained earnings is made.

255
Q

Which of the following should a company classify as a research and development expense?

A
Periodic design changes to existing products
B
Routine design of tools, jigs, molds and dies
C
Redesign of a product pre-release
D
Legal work on patent applications
A

Explanation:
The correct answer is (C).

A redesign of a product pre-release would be classified as research & development. The term “pre-release” is key here. The product has not become technologically-feasible for sale, and therefore any work on it could be classified as an R&D expense.

The other items cannot be classified as R&D. Design changes occur after the product is in saleable condition, routine designs of high-volume items cannot be classified under R&D, and legal work on patent applications is not R&D.

256
Q
On November 1 of the current year, Davis Co. discounted with recourse at 10% a one-year, noninterest bearing, $20,500 note receivable maturing on January 31 of next year. What amount of contingent liability for this note must Davis disclose in its financial statements for the current year ended December 31?
A
$0
B
$20,000
C
$20,333
D
$20,500
A

Explanation:
Davis discounted the note receivable with recourse. If the maker fails to pay at maturity, the note is presented to Davis, who is then liable for $20,500, the amount due at maturity. Davis should disclose this contingent liability in its current year 12/31 financial statements.

257
Q

A company recorded a decommissioning liability and recognized the amount recorded as part of the cost of the related property. After the property was fully depreciated, the decommissioning liability was reviewed and adjusted. How should this change in the decommissioning liability be recognized under IFRS?

A
The change in the liability is recognized in other comprehensive income.
B
The change in the liability is recognized in profit or loss.
C
The change in the liability is recognized as a change in the carrying amount of the property if the liability increases but is otherwise recognized in profit or loss.
D
The change in the decommissioning liability is not recognized until it is settled.

A

Explanation:
Under IFRS, changes in the decommissioning liability are added to or deducted from the cost of the asset, with any reductions in excess of the carrying amount of the asset recognized as a gain in the current period. The change in the liability is therefore recognized in profit or loss, not in other comprehensive income or as a change in the carrying amount of the property.

258
Q
Dahl Co. traded a delivery van and $5,000 cash for a newer van owned by West Corp. Dahl expects much lower maintenance costs and a much greater useful life from the newer van. The lower mainteance costs and greater useful life will have a significant impact on future cash flows. The following information relates to the values of the vans on the exchange date:
Carrying Value	Fair Value
Old Van	$30,000	$45,000
New Van	$40,000	$50,000Dahl's income tax rate is 30%. What amounts should Dahl report as gain on exchange of the vans?
A
$15,000
B
$1,000
C
$700
D
$0
A

Explanation:
This is considered an exchange with commercial substance because it expects to significantly change the configuration of future cash flows. Dahl is also giving boot in this exchange.

Dahl has a realized gain on its old van of $15,000:

Cash paid	$ 5,000
Book value of truck given up	30,000
Total consideration given up	35,000
Fair value of truck received	(50,000)
Gain on exchange	$ 15,000The journal entry to be recorded in Dahl's books is:
Truck received	50,000
Cash paid	5,000
Truck given up	30,000
Gain	15,000
259
Q

When remeasuring foreign currency financial statements into the functional currency, which two of the following items would be remeasured using historical exchange rates?

Inventories carried at cost.
Equity securities reported at market values.
Bonds payable.
Accrued liabilities.
Prepaid expenses.
A
I and II
B
II and V
C
II and III
D
I and V
A

Explanation:
Nonmonetary balance sheet items, including intangibles, fixed assets, inventory, and prepaid expenses, are remeasured at historical rates.

260
Q

On August 31, 20X0, Wood Corp. issued 100,000 shares of its $20 par value common stock for the net assets of Pine, Inc., in a business combination accounted for using the acquisition method. The market value of Wood’s common stock on August 31 was $36 per share. Wood paid a fee of $160,000 to the consultant who arranged this acquisition. Costs of registering and issuing the equity securities amounted to $80,000. No goodwill was involved in the purchase. What amount should Wood capitalize as the cost of acquiring Pine’s net assets?

A
$3,600,000
B
$3,680,000
C
$3,760,000
D
$3,840,000
A

Explanation:
Investment in a subsidiary is recorded at fair value. Wood issued 100,000 shares of $20 par value and market value of $36 to acquire Pine Inc. The fair value of consideration is $3,600,000 (i.e. 100,000 shares x $36 per share). All direct, indirect and general costs for acquisition incurred are expensed. The $160,000 paid to consultant to arrange for the acquisition is expensed. Costs associated with issuance and registration of equity would be netted against the proceeds by debiting additional paid-in capital (APIC). Investment account will not be affected. The $80,000 incurred on registering and issuing equity shares is debited to Wood’s APIC.

Option (B) is incorrect because $3,680,000 includes $80,000 costs of registering and issuing equity shares.

Option (C) is incorrect because $3,760,000 includes $160,00 consulting fee paid for the acquisition.

Option (D) is incorrect because $3,840,000 includes both $160,000 consulting fee and $80,000 cost of issuing and registration of equity.

261
Q

In a period of rising general price levels, Pollard Corp. discloses income on a current cost basis in accordance with FASB Standards. Which of the following contributes to Pollard’s purchasing power loss on net monetary items?

A
Refundable deposits with suppliers.
B
Equity investment in unconsolidated subsidiaries.
C
Warranty obligations.
D
Wages payable.
A

Explanation:
An entity suffers purchasing power losses in a period of rising general price levels from holding monetary assets. A monetary asset is money or a claim to receive a sum of money, the amount of which is fixed or determinable without reference to future prices of specific goods and services. A refundable deposit with a supplier is a monetary asset.

262
Q

Clay Company leased a new machine from Saxe Corp. The following data relate to the lease transaction at the inception of the lease:

Lease term 10 years
Annual rental payable at the beginning of each lease year $50,000
Useful life of machine 15 years
Implicit interest rate 10%
Present value of an annuity due of $1 for 10 periods at 10% 6.76
Present value of an ordinary annuity of $1 for 10 periods at 10% 6.15
Fair value of the machine $400,000
The lease has no renewal option, and the possession of the machine reverts to Saxe when the lease terminates, at which point, Saxe can re-use the machine. At the inception of the lease, Clay should account for the lease as a/an

A
Finance lease
B
Direct financing lease
C
Operating lease
D
Sales-type lease
A

Explanation:
A Finance Lease must meet one of the following criteria:

Present Value equals or exceeds substantially all (90%) of the Fair Value
Option to Purchase (exercise is reasonably certain)
Economic Life - Major part (75%) of asset’s economic life is used
Transfer of Ownership at lease termination
Specialized Nature - No alternative use to the lessor at lease termination
Note: the implementation guidance for ASC 842 uses the 75/90 thresholds, even though the standard is principles-based.

P -The rental payments are at the beginning of each lease year, so the Present Value of an annuity due is used @ 6.76

$50,000 x 6.76 = $338,000 vs Fair Value of $400,000 = 84% vs 90%

O - No renewal option

E - 10 years of economic life used vs 15 years of useful life = 66% vs 75%

T - No transfer

S - The asset is not specialized.

Since none of the criteria are met, it is an operating lease.

263
Q
During the prior year, Haft Co. became involved in a tax dispute with the IRS. At December 31 of the prior year, Haft's tax advisor believed that an unfavorable outcome was probable. A reasonable estimate of additional taxes was $200,000, but could be as much as $300,000. After the prior year financial statements were issued, Haft received and accepted an IRS settlement offer of $275,000. What amount of accrued liability should Haft have reported in its prior year December 31 balance sheet?
A
$200,000
B
$250,000
C
$275,000
D
$300,000
A

Explanation:
A loss from a contingent liability, arising from events or circumstances occurring before the balance sheet date and the resolution of which is contingent upon a future event or circumstance, of which an unfavorable outcome is probable, should be accrued. The amount should be estimated and the minimum amount in the range of estimates should be accrued.

264
Q

Neron Co. has two derivatives related to two different financial instruments, instrument A and instrument B, both of which are debt instruments. The derivative related to instrument A is a fair value hedge, and the derivative related to instrument B is a cash flow hedge. Neron experienced gains in the value of instruments A and B due to a change in interest rates. Which of the gains should be reported by Neron in its income statement?
Gain in value of debt instrument A Gain in value of debt instrument B
A Yes Yes
B Yes No
C No Yes
D No No

A

Explanation:
The gain or loss of a fair value hedge (instrument A) is recognized in earnings in the period of change, together with the offsetting loss or gain in the hedged item. The effective portion of the gain or loss of a cash flow hedge (instrument B) is initially reported as a component of other comprehensive income (OCI) on the balance sheet, and subsequently reclassified into earnings when the forecasted transaction affects earnings.

265
Q

On December 1 of the current year, Bann Co. entered into an option contract to purchase 2,000 shares of Norta Co. stock for $40 per share (the same as the current market price) by the end of the next two months. The time value of the option contract is $600. At the end of December, Norta’s stock was selling for $43, and the time value of the option is now $400. If Bann does not exercise its option until January of the subsequent year, which of the following changes would reflect the proper accounting treatment for this transaction on Bann’s December 31, year-end financial statements?
A
The option value will be disclosed in the footnotes only.
B
Other comprehensive income will increase by $6,000.
C
Net income will increase by $5,800.
D
Current assets will decrease by $200.

A

Explanation:
An option contract is a derivative. Derivatives are recognized as assets or liabilities on the financial statements and measures using fair value. Changes in fair value of non-hedge securities are reported as gains or losses in earnings. On December 1, Bann would initially record the option contract at $80,600 (2,000 shares × $40 market price on December 1 + $600 time value). At the end of December, the fair value of the option contract was $86,400 (2,000 shares × $43 market price on December 31 + $400 time value). Bann would report this change in fair value which would result in net income increasing by $5,800.

266
Q

Damon Co. purchased 100% of the outstanding common stock of Smith Co. in an acquisition by issuing 20,000 shares of its $1 par common stock that had a fair value of $10 per share and providing contingent consideration that had a fair value of $10,000 on the acquisition date. Damon also incurred $15,000 in direct acquisition costs. On the acquisition date, Smith had assets with a book value of $200,000, a fair value of $350,000, and related liabilities with a book and fair value of $70,000. What amount of gain should Damon report related to this transaction?

A
$55,000
B
$70,000
C
$80,000
D
$250,000
A

Explanation:
Ref

Summary

Amount

a

Fair value of the asset

$350,000

b

Fair value of liabilities

70,000

c

Net assets (a-b)

280,000

d

FV of the consideration (20,000 shares issued at $10 each)

200,000

e

Providing for contingent consideration

10,000

f

Total consideration (d+e)

210,000

g

Gain on the transation (c-f)

$70,000

Costs incurred in business acquisition, direct, indirect or general costs are All expensed. $15,000 is direct expenses related to the transaction are not considered.

Option (A) is incorrect because $55,000 has included direct expenses of $15,000 in arriving at the gain.

Option (C) is incorrect because it does not provide $10,000 for contingent consideration.

Option (D) is incorrect because $250,000 includes BV of the consideration paid on stocks at $1 instead of valuing the stock at market value of $10 each.

267
Q

On February 1, Pizza, Inc. acquired 75% of the outstanding common stock of Sausage Co. for $750,000 cash. At February 1, Sausage’s balance sheet showed a carrying amount of net assets of $1,100,000 and the fair value of Sausage’s assets and liabilities equaled their carrying amounts except for property, plant, and equipment which exceeded its carrying amount by $200,000. On February 1, what amount would be attributed as fair value for the noncontrolling interest?

A
$325,000
B
$300,000
C
$275,000
D
$250,000
A

Explanation:
An acquirer can measure the acquisition-date fair value of a noncontrolling interest on the basis of active market prices for the equity shares not held by the acquirer. If the active market prices for equity shares of the acquiree are not available, the acquirer must use another valuation technique to determine the noncon­trolling interest’s fair value. One can infer what the fair value must be based upon the percentage acquired by the acquirer and the amount of consideration the acquirer transferred in the business combination. Dividing the consideration transferred by the percentage acquired results in the amount 100% of the acquiree must be worth. Multiplying that total worth by the percentage owned by the noncontrolling interest results in fair value attribut­able to the noncontrolling interest. Pizza paid $750,000 for 75% of Sausage, thus 25% is considered as noncon­trolling interest. $750,000 ÷ .75 = $1,000,000; then $1,000,000 × .25 = $250,000 fair value for the noncontrolling interest.

268
Q

Vik Auto and King Clothier exchanged goods, held for resale, with equal fair values. Each will use the other’s goods to promote their own products. The goods exchanged are goods they each would have used otherwise for promotions and thus aren’t expected to significantly change future cash flows. The retail price of the car that Vik gave up is less than the retail price of the clothes received. What profit should Vik recognize for the nonmonetary exchange?
A
A profit is not recognized.
B
A profit equal to the difference between the retail prices of the clothes received and the car.
C
A profit equal to the difference between the retail price and the cost of the car.
D
A profit equal to the difference between the fair value and the cost of the car.

A

Explanation:
This transaction does not have commercial substance because it does not significantly change future cash flows. It is an exchange of goods held for resale for other assets to be used not for resale, but merely as a different means of promoting the company’s own goods to customers.

A nonmonetary exchange without commercial substance is measured based on the recorded amount (after reduction, if appropriate, for an indicated impairment of value) of the nonmonetary asset(s) relinquished, and not on the fair values of the assets exchanged. No gains, only losses, are recognized on exchanges without commercial substance.

269
Q

The functional currency of Nash, Inc.’s subsidiary is the euro. Nash borrowed euros as a partial hedge of its investment in the subsidiary. In preparing consolidated financial statements, Nash’s translation loss on its investment in the subsidiary exceeded its exchange gain on the borrowing. How should the effects of the loss and gain be reported in Nash’s consolidated financial statements?
A
The translation loss less the exchange gain is reported in other comprehensive income.
B
The translation loss less the exchange gain is reported in net income.
C
The translation loss is reported in other comprehensive income and the exchange gain is reported in net income.
D
The translation loss is reported in net income and the exchange gain is reported in other comprehensive income.

A

Explanation:
Translation adjustments are not be included in determining net income, but in OCI. Gains and losses on foreign currency transactions that are designated as, and are effective as, economic hedges of a net investment in a foreign entity are not included in determining net income, but are reported in the same manner as translation adjustments. Therefore, the net translation loss is be reported in OCI in Nash’s consolidated financial statements.

270
Q

How should a company report its decision to change from a cash-basis of accounting to accrual-basis of accounting?

A
As a change in accounting principle, requiring the cumulative effect of the change (net of tax) to be reported in the income statement.
B
Prospectively, with no amounts restated and no cumulative adjustment.
C
As an extraordinary item (net of tax).
D
As a prior-period adjustment (net of tax), by adjusting the beginning balance of retained earnings.

A

Explanation:
The change from cash-basis accounting to accrual-basis accounting is a change from an account­ing principle that is not generally accepted to one that is generally accepted, which is considered a correction of an error. Correction of an error requires a prior-period adjustment, which is done net of tax, by adjusting the begin­ning retained earnings.

271
Q

A lease should be classified as a finance lease if which of the following occurs?

A
The present value of the lease payments plus residual value guarantee is over 35% percent of the fair value of the asset at lease inception.
B
The lease transfers ownership of the underlying asset to the lessee at the end of the lease term
C
Leased asset is non-specialized in nature
D
The lease contains a bargain purchase option that the lessee will likely not exercise

A

Explanation:
(B) The lease would be classified as a finance lease if the lease transfers ownership of the underlying asset to the lessee at the end of the lease term.

A Finance Lease must meet one of the following criteria:

Present Value equals or exceeds substantially all (90%) of the Fair Value
Option to Purchase (exercise is reasonably certain)
Economic Life - Major part (75%) of asset’s economic life is used
Transfer of Ownership at lease termination
Specialized Nature - No alternative use to the lessor at lease termination
Note: the implementation guidance for ASC 842 uses the 75/90 thresholds, even though the standard is principles-based.

Options A, C, and D are incorrect because they conflict with the above criteria for Finance Leases.

272
Q

Under IFRS 16, Lease Accounting:

A
IFRS 16 does not distinguish between sales-type and direct financing leases.
B
IFRS 16 has a lessee recognition and measurement exemption for leases of assets with values of less than $50,000.
C
IFRS 16 does not permit recognition of selling profit on direct financing leases at lease commencement.
D
IFRS 16 has a lessor recognition and measurement exemption for leases of assets with values of less than $10,000.

A

Explanation:
The correct answer is (A).

IFRS 16 does not distinguish between sales-type and direct financing leases.

(C) is incorrect IFRS 16 permits recognition of selling profit on direct financing leases at lease commencement as there is no difference between sales-type lease and direct financing lease.

(A) and (D) is incorrect IFRS 16 has a lessee recognition and measurement exemption for leases of assets with values of less than $5,000.

273
Q

A subsidiary, acquired for cash in a business combination, owned inventories with a market value different than the carrying amount as of the date of
combination. A consolidated balance sheet prepared immediately after the acquisition would include this difference as part of

A
Deferred credits
B
Goodwill
C
Inventories
D
Retained earnings
A

Explanation:
Under the acquisition method, a consolidated balance sheet prepared immediately after the acquisition includes the assets of the subsidiary at their fair values.
The difference would be part of inventories.

274
Q
Hunt Co. purchased merchandise for £300,000 from a vendor in London on November 30 of the current year. Payment in British pounds was due on January 30 of the next year. The exchange rates to purchase one pound were as follows:
Nov. 30	Dec. 31
Spot-rate	$1.65	$1.62
30-day rate	1.64	1.59
60-day rate	1.63	1.56In its December 31, current year income statement, what amount should Hunt report as foreign exchange gain?
A
$12,000
B
$9,000
C
$6,000
D
$0
A

Explanation:
Whenever a transaction is denominated (i.e., payable) in a foreign currency, changes in the translation rate (i.e., the spot rate) of the foreign currency with respect to the entity’s functional currency (i.e., the dollar in this case) will result in a gain or loss. The gain or loss should be recognized in income in the period(s) the rate changes, and the related asset or liability (i.e., accounts payable in this case) should be adjusted accordingly.

Initial obligation, 11/30, in U.S. dollars
(£300,000 × $1.65)	$ 495,000
Amount payable, 12/31, in U.S. dollars
(£300,000 × $1.62)	(486,000)
Foreign exchange gain recognized	$ 9,000
275
Q

Included within the financial assets of Z square Co. at December 31, 2017 are the following two recently purchased investments in publicly-traded (listed in NASDAQ) equity shares:

Investment 1: 10% of the issued share capital of ABC Co. This shareholding was acquired as a long-term investment as Z square Co. wishes to participate as an active shareholder of ABC Co.

Investment 2: 10% of the issued share capital of XYZ Co. This shareholding was acquired for speculative purposes and Z square Co. expects to sell these shares in the near future.

Neither of these shareholdings gives Z square Co. significant influence over the investee companies. Wherever possible, the directors of Z square Co. wish to avoid taking any fair value movements to income statement, so as to minimize volatility in reported earnings. How should the fair value movements in these investments be reported in Z square Co’s financial statements for the year ended December 31, 2017?

A
In income statement for both investments
B
In other comprehensive income for both investments
C
In income statement for investment 1 and in other comprehensive income for investment 2
D
In other comprehensive income for investment 1 and in income statement for investment 2

A

Explanation:
The correct answer is (A).

As the shares of both the companies ABC Co. and XYZ Co. are listed in NASDAQ, thus, the fair value of both investments is readily determinable and for any movement in fair value should be reported in the income statement as Fair Value Through Net Income (FVTNI).

ASC 321 require equity investments (except those accounted for under the equity method of accounting or those that result in consolidation of the investee) to be measured at fair value with changes in fair value recognized in net income (FVTNI). However, an entity may choose to measure equity investments that do not have readily determinable fair values at cost minus impairment, if any, plus or minus changes resulting from observable price changes in orderly transactions for the identical or a similar investment of the same issuer. An equity security has a readily determinable fair value if it meets any of the following conditions:

The fair value of an equity security is readily determinable if sales prices or bid-and-asked quotations are currently available on a securities exchange registered with the U.S. Securities and Exchange Commission (SEC) or in the over-the-counter market, provided that those prices or quotations for the over the-counter market are publicly reported by the National Association of Securities Dealers Automated Quotations (NASDAQ) systems or by OTC Markets Group Inc. Restricted stock meets that definition if the restriction terminates within one year.
The fair value of an equity security traded only in a foreign market is readily determinable if that foreign market is of a breadth and scope comparable to one of the U.S. markets referred to above.
The fair value of an equity security that is an investment in a mutual fund or in a structure similar to a mutual fund (that is, a limited partnership or a venture capital entity) is readily determinable if the fair value per share (unit) is determined and published and is the basis for current transactions.
Options (B), (C) and (D) are incorrect because for movement in fair value for investment in both securities should be reported in income statement only and not through other comprehensive income.

276
Q
Potter Co. has the following contingencies, all resulting from lawsuits in progress during the current year:
Probable loss contingency	$1,500,000
Reasonably possible loss contingency	500,000
Probable gain contingency	700,000
Reasonably possible gain contingency	300,000Potter’s accountant believes the financial statements will be misleading if the probable loss contingency is not disclosed. How much should be disclosed, and how much should be accrued in Potter’s financial statements for the current year?
Disclosed	Accrued
A	$ 500,000 loss
$ 1,000,000 gain	$ 1,500,000 loss
$ 700,000 gain
B	$ 2,000,000 loss
$ 1,000,000 gain	$ 1,500,000 loss
C	$ 1,000,000 gain	$ 1,500,000 loss
$ 500,000 loss
D	$ 500,000 loss
$ 300,000 gain	$ 1,500,000 loss
A

Explanation:
Where the likelihood of confirmation of a contingency loss is considered probable and the loss can be reasonably estimated, the estimated loss should be accrued by a charge to income and the nature of the contingency should be disclosed. Where the contingency loss is considered reasonably possible, no charge should be made to income but the nature of the contingency should be disclosed. Gain contingencies, whether probable or reasonable possible, should be disclosed but not recognized as income. Potter would disclose the $2,000,000 in loss contingencies and the $100,000 in gain contingencies. Potter would accrue just the $1,500,000 probable loss contingency.

277
Q

On January 1, Dallas, Inc. purchased 80% of Style, Inc.’s outstanding common stock for $120,000. On that date, the carrying amounts of Style’s assets and
liabilities approximated their fair values. During the year, Style paid $5,000 cash dividends to its stockholders. Summarized balance sheet information for the
two companies follows:

Dallas Style


12/31 12/31 01/01

Investment in Style (equity method) $132,000
Other assets 138,000 $115,000 $100,000
$270,000 $115,000 $100,000
Common stock $ 50,000 $ 20,000 $ 20,000
Additional paid-in capital 80,250 44,000 44,000
Retained earnings 139,750 51,000 36,000
$ 270,000 $ 115,000 $ 100,000
What amount should Dallas report as earnings from subsidiary in its year-end income statement?

A
$12,000
B
$15,000
C
$16,000
D
$20,000
A

Explanation:
During the year, the subsidiary’s retained earnings increased by $15,000 (i.e., $51,000 - $36,000). Since the subsidiary declared and paid a $5,000 cash
dividend, the subsidiary’s net income was $20,000 (i.e., $15,000 + $5,000). Since at the date of the 80% purchase, the carrying amount of the subsidiary’s
assets and liabilities approximated their fair values, the parent company’s earnings from the subsidiary is $16,000 (i.e., $20,000 × 80%).

Beg Retained Earnings + Net Income – Dividends = Ending Retained Earnings
Net Income = Ending Retained Earnings + Dividends – Beg Retained Earnings
Net Income = 51,000 + 5,000 – 36,000
Net Income = 20,000

0.8*20,000 = 16,000

278
Q

On January 1, of the current year, Tree Co. enters into a five-year lease agreement for production equipment. The lease requires Tree to pay $12,500 per year in lease payments. At the end of the five- year lease term, Tree can purchase the equipment for $30,000. The fair value of the equipment is $75,000. The estimated useful life of the equipment is 10 years. The present value of the lease payments is $50,000. The present value of the purchase option is $20,000. Tree’s controller believes the purchase option price is sufficiently below the expected fair value of the equipment at the date the option becomes exercisable to reasonably assure its exercise. Tree would normally depreciate equipment of this type using the straight-line method. What amount is the carrying value of the asset related to this lease at December 31, of the current year?

A
$40,000
B
$45,000
C
$56,000
D
$63,000
A

Explanation:
The correct answer is (D).

The lease is a finance lease as the lessee has an option to purchase the asset that the lessee is reasonably certain to exercise.

The asset will be capitalized atPV of the lease payments of $50,000 + the PV of the purchase option of$20,000 = $70,000.

December 31st = Depreciate the asset by 10% (10 years) $70,000 / 10 = $7,000 = $70,000 - 63,000.

The equipment will be depreciated using the straight line method over the asset life of 10 years. Thus, the depreciation for the first year = $70,000 / 10 = $7,000. The carrying value of the equipment as of December 31, year 1 = $70,000 - $7,000 = $63,000.

279
Q

A six-year Finance Lease entered into on December 31 of the current year specified equal annual lease payments due on December 31 of each year. The first annual lease payment, paid on December 31 this year, consists of which of the following?

Interest expense	Lease liability
A	Yes	Yes
B	Yes	No
C	No	Yes
D	No	No
A

Explanation:
When the first annual lease payment is paid at the inception of a Finance Lease, the whole payment reduces the lease liability (no time has passed for interest to accrue). Subsequent payments are allocated between interest expense and lease liability reduction.

280
Q
On January 1 of the current year, Jambon purchased equipment for use in developing a new product. Jambon uses the straight-line depreciation method. The equipment could provide benefits over a 10-year period. However, the new product development is expected to take five years, and the equipment can be used only for this project. Jambon's current year expense equals
A
The total cost of the equipment.
B
One-fifth of the cost of the equipment.
C
One-tenth of the cost of the equipment.
D
Zero.
A

Explanation:
The cost of equipment or facilities that are acquired or constructed for research and development (R&D) activities and have alternative future uses (in R&D or otherwise) should be capitalized when acquired or constructed. However, the cost of equipment or facilities that are acquired or constructed for a particular R&D project and have no alternative future uses (in other R&D projects or otherwise) are expensed as R&D costs at the time the costs are incurred.

281
Q

Wall Co. leased office premises to Fox, Inc. for a five-year term beginning January 1 of the current year. Under the terms of the operating lease, rent for the first year is $8,000 and rent for years 2 through 5 is $12,500 per annum. However, as an inducement to enter the lease, Wall granted Fox the first six months of the lease rent-free. In its current year December 31 income statement, what amount should Wall report as rental income?

A
$12,000
B
$11,600
C
$10,800
D
$ 8,000
A

Explanation:
The correct answer is (C).

Total rent income receivable for the entire lease term is divided evenly over each period in line with matching principle irrespective of the pattern of payments. Whether payments increase or decrease during the term of the lease, or whether the lease contains periods that may be rent-free, or involves non-refundable deposits, the total of the payments received is recognized evenly over the term of the lease.

Over the 5-year term of the lease, Wall will receive the following rents:

The rent for the first year, for 12 months is $8,000.

However, rent for the first 6 months is free. Rent received for the first year is only for the remaining 6 months = $4,000.

Rent received for the remaining 4 years is $12,500 per annum = $12,500 x 4 = $50,000.

Total rent received = $54,000.

This rent of $54,000 is to be recognized uniformly over the 5-year term of the lease.

Wall will recognize $54,000 / 5 years or $10,800 per year.

282
Q

At December 31 of the current year, Date Co. awaits judgment on a lawsuit for a competitor’s infringement of Date’s patent. Legal counsel believes it is probable that Date will win the suit and indicated the most likely award together with a range of possible awards. How should the lawsuit be reported in Date’s current year financial statements?
A
In note disclosure only
B
By accrual for the most likely award
C
By accrual for the lowest amount of the range of possible awards
D
Neither in note disclosure nor by accrual

A

Explanation:
Gain contingencies are not to be accrued before realization, thus no gain contingency can be accrued until the lawsuit is settled; however, the gain should be disclosed in the financial statement notes.

283
Q

In a sale-leaseback transaction, the seller-lessee has retained the property. The gain on the sale should be recognized at the time of the sale-leaseback when the lease is classified as:

Finance Lease	Operating Lease
A	Yes	Yes
B	No	No
C	No	Yes
D	Yes	No
A

Explanation:
The correct answer is (C).

If the leaseback is a finance lease from the seller-lessee’s perspective, the transfer of the asset is not a sale i.e., treat as if no sale has happened.

The rationale for this is that seller-lessee has not satisfied any performance obligation per the Revenue Recognition standards as control of the asset is not transferred to the buyer-lessor.

The seller-lessee will not derecognize the transferred asset as no sales have happened per the Revenue Recognition standards.

The amount received from the buyer-lessor would be accounted for as a financial liability and the gain on sale would not be recognized.

284
Q

Which of the following is not a characteristic of market participants when measuring fair value?

A
They are independent of each other.
B
They are knowledgeable and sufficiently informed about the transaction.
C
They are related parties.
D
They are willing and able to enter into the transaction.
A

Explanation:
Market participants are buyers and sellers in the market for the asset or liability that are independent of the reporting entity, knowledgeable, willing and able to transact for the asset or liability (not forced).

285
Q

What are the components of the Net Investment in Lease for a lessor involved in a direct-financing lease?

A
The present value of lease payments not yet received plus any initial direct costs
B
Present Value of Lease payments not yet received plus present value of guaranteed and unguaranteed residual value minus Selling Profit
C
The present value of lease payments not yet received plus present value of unguaranteed residual value minus Selling Profit
D
The present value of lease payments not yet received less initial direct costs

A

Explanation:
In a direct financing lease, the lessee does not individually obtain control of the asset but the lessor does relinquish control. This would occur if (1) the present value of the lease payments and any residual value guarantee (which could be provided entirely by a third party or consist of a lessee guarantee coupled with a third-party guarantee) represents substantially all of the fair value of the underlying asset and (2) it is probable that the lessor will collect the lease payments and any amounts related to the residual value guarantee(s).

At the commencement date, a lessor shall recognize a net investment in the lease (Present Value of Lease payments not yet received + PV of guaranteed and unguaranteed residual value – Selling Profit).

Option (A), (C) and (D) are incorrect as per above explanation.

286
Q
When remeasuring foreign currency financial statements into the functional currency, which of the following items would be remeasured using historical exchange rates?
A
Inventories carried at cost.
B
Marketable equity securities reported at market values.
C
Bonds payable.
D
Accrued liabilities.
A

Explanation:
If an entity does not maintain its books in its functional currency, remeasuring into the functional currency is required prior to translation into the reporting currency (i.e., the parent company’s currency). In this process, nonmonetary balance sheet items are remeasured using historical exchange rates. Hence, inventories carried at cost should be remeasured using historical exchange rates because it is an example of a nonmonetary balance sheet item. Marketable equity securities, bonds payable, and accrued liabilities balance sheet items are remeasured using the current exchange rate.

287
Q

Toigo Co. purchased merchandise from a vendor in England on November 20 for 500,000 British pounds. Payment was due in British pounds on January 20. The spot rates to purchase one pound were as follows:
November 20 $1.25
December 31 1.20
January 20 1.17How should the foreign currency transaction gain be reported on Toigo’s financial statements at December 31?
A
A gain of $40,000 as a separate component of stockholders’ equity.
B
A gain of $40,000 in the income statement.
C
A gain of $25,000 as a separate component of stockholders’ equity.
D
A gain of $25,000 in the income statement.

A

Explanation:
At the date the transaction is recognized, each asset, liability, revenue, expense, gain, or loss arising from the transaction should be measured and recorded in the functional currency of the recording entity by use of the exchange rate in effect at that date. Toiga would record a liability of $625,000 (500,000 x $1.25). At each balance sheet date, recorded balances that are denominated in a currency other than the functional currency of the recording entity should be adjusted to reflect the current exchange rate. These adjustments should be currently recognized as transaction gains or losses and reported as a component of income from continuing operations. Toigo would recognize a gain of $25,000 on the December 31 financial statements based on 500,000 pounds × $1.20 = $600,000 as of December 31.

288
Q

A lease is classified as a Finance Lease because it contains a purchase option that the lessee is reasonably-certain to exercise. Over what period of time should the lessee amortize the leased property?

A
The term of the lease.
B
The useful life of the asset.
C
The lease term or the useful life of the asset, whichever is shorter.
D
The useful life of the asset, not to exceed 40 years.
A

Explanation:
A Finance Lease must meet one of the following criteria:

Present Value equals or exceeds substantially all (90%) of the Fair Value
Option to Purchase (exercise is reasonably certain)
Economic Life - Major part (75%) of asset’s economic life is used
Transfer of Ownership at lease termination
Specialized Nature - No alternative use to the lessor at lease termination
Note: the implementation guidance for ASC 842 uses the 75/90 thresholds, even though the standard is principles-based.

Under a Finance Lease, the amortization period for an ROU asset is from the lease commencement date to the end of the lease term or the end of the asset’s useful life, whichever is earlier.

Exception: In the case of a purchase option that the lessee is reasonably-certain to exercise, the amortization period runs through the end of the asset’s useful life.

289
Q

Sayon Co. issues 200,000 shares of $5 par value common stock to acquire Trask Co. in an acquisition method business combination. The market value of Sayon’s common stock is $12. Legal and consulting fees incurred in relationship to the purchase are $110,000. Registration and issuance costs for the common stock are $35,000. What should be recorded in Sayon’s additional paid-in capital account for this business combination?

A
$1,545,000
B
$1,400,000
C
$1,255,000
D
$1,365,000
A

Explanation:
Below Journal Entry is recorded for the acquisition of Trask Co.:

Dr Investment in Trask $2,400,000
Cr Cash 35,000
Cr Common Stock 1,000,000
Cr Additional paid-in-capital (APIC) 1,365,000
Direct, indirect or general costs are all expenses. E.g., Legal, accounting, consulting, finder’s fees, G&A. Accordingly, legal and consulting fees $110,000 are expensed. Costs associated with the issuance and registration of debt or equity securities, are netted against the proceeds. Accordingly, $35,000 is netted against the proceeds and impacts APIC, but has no net effect on the investment price.

Option (A) is incorrect because legal & consulting fees and registration & issuance costs are debited to investment account. Legal & consulting fees should be expensed and issuance & registration costs should be netted from proceeds of the stock and effects APIC.

Option (B) is incorrect because issuance and registration costs should be netted against the proceeds, however they are expensed.

Option (C) is incorrect because legal and consulting fees are netted against the APIC which should be expensed.

290
Q

Miller Co. incurred the following computer software costs for the development and sale of software programs during the current year:

Planning costs $ 50,000
Design of the software 150,000
Substantial testing of the project’s initial stages 75,000
Production and packaging costs for the first month’s sales x 500,000
Costs of producing product masters after technology feasibility was established 200,000
The project was not under any contractual arrangement when these expenditures were incurred. What amount should Miller report as research and development expense for the current year?

A
$200,000
B
$275,000
C
$500,000
D
$975,000
A

Explanation:
R&D costs incurred internally in creating a computer software product are charged to expense when incurred as research and development until technological feasibility has been established for the product.Tech­nological feasibility is established only upon completion of a detailed program design or, in its absence, com­pletion of a working model.All costs of planning, designing, coding, and testing activities that are necessary to establish technological feasibility are expensed as research and development when incurred.The costs of producing product masters incurred subsequent to establishing technological feasibility ($200,000) are capitalized. Capitalization of computer software costs ceases when the product is available for general release to customers .Capitalized software production costs are reported at the lower of unamortized cost or net realizable value. Costs incurred for (A) duplicating the computer software and training materials from product masters and (B) physically packaging the product for distribution ($75,000) are capitalized as inventory. Capitalized inventory costs are expensed when the inventory is sold. Options (a), (c) and (d) are incorrect based on the above explanations.

291
Q
A foreign subsidiary's functional currency is its local currency, which has not experienced significant inflation. The weighted average exchange rate for the current year would be the appropriate exchange rate for translating
Sales to customers	Wages expense
A	No	No
B	Yes	Yes
C	No	Yes
D	Yes	No
A

Explanation:
Since the foreign subsidiary’s functional currency is its local currency, which has not experienced significant inflation, it is appropriate to translate the amounts of its revenues, expenses, gains and losses at a weighted average exchange rate for the period.

292
Q

Which of the following is true regarding lease payments?

A
Lease payments would include payments for variable leases not dependent on index or rate.
B
Lease payments would include guaranteed residual value minus expected residual value.
C
Lease payments would include the full value of guaranteed residual value.
D
Lease payments include contingent rental.

A

Explanation:
The correct answer is (B).

A Residual Value Guarantee is a commitment to compensate the lessor for a shortfall in the value of the underlying asset at the end of the lease term. Hence, lease payments will include guaranteed residual value minus expected residual value.

Option (A), (C) and (D) are incorrect as per above explanation

293
Q

West, Inc. acquired 60% of East Co.’s outstanding common stock. West paid $800,000 to acquire the stock. West plans to relocate East’s company headquarters, which is expected to cost between $100,000 and $300,000. The present value of the probability-adjusted relocation cost is $240,000. What is West’s acquisition cost?

A
$800,000
B
$900,000
C
$1,040,000
D
$1,100,000
A

Explanation:
When one corporation (i.e., acquirer or parent) acquires a controlling financial interest by ownership of majority of voting shares of stock normally over 50% (i.e., control) of another corporation (i.e., acquiree or subsidiary), this creates a relationship requiring the preparation of consolidated Financial Statements at each Balance Sheet date. Investment in Sub = Consideration paid @ Fair Value = $800,000.

Option (B) is incorrect because $100,000 which is the minimum expected cost of relocating headquarters is included.

Option (C) is incorrect because $240,000 which is the Present value of the probability-adjusted cost of relocating headquarters is included.

Option (D) is incorrect because $300,000 which is the minimum expected cost of relocating headquarters is included.

294
Q

The present value of lease payments should be used by the lessee in determining the amount of a lease liability under a lease classified by the lessee as a (an)

Finance Lease	Operating Lease
A	Yes	Yes
B	Yes	No
C	No	No
D	No	Yes
A

Explanation:
The correct answer is (A).

Per ASC 842, for both finance and operating leases, a lessee will use the present value of lease payments to determine the lease liability.

Option (B), (C) and (D) are incorrect as per above explanation.

295
Q

Tanker Oil Co., a developmental stage enterprise, incurred the following costs during its first year of operations:
Legal fees for incorporation and other related matters $55,000
Underwriters’ fees for initial stock offering 40,000
Exploration costs and purchases of mineral rights 60,000Tanker had no revenue during its first year of operation. What amount may Tanker capitalize as organizational costs?
A
$155,000
B
$ 95,000
C
$ 55,000
D
$ 0

A

Explanation:
Financial statements issued by a development stage enterprise should be presented in conformity with generally accepted accounting principles that apply to established operating enterprises. These accounting principles determine whether a cost incurred by a development stage enterprise should be charged to expense when incurred or should be capitalized or deferred. Organization costs include (1) accounting services incidental to organization, (2) legal services for drafting the corporate charter and bylaws, (3) state incorporation filing fees, and (4) costs of temporary directors and of organizational meetings. Start-up activities, including organization costs, should be expensed as incurred.

296
Q

A company from the United Kingdom uses British pounds in its normal operations, reports in the European Union in euros, and reports in the United States in U.S. dollars. The company is owned by a private equity firm in Japan. What is the company’s functional currency?

A
The Euro
B
The British pound
C
The U.S. dollar
D
The Japanese yen
A

Explanation:
The correct answer is (B).

Functional currency represents the primary economic environment in which an entity generates cash and expends cash. The functional currency is the currency primarily used by a business or business unit. As a monetary unit of account, a functional currency represents the primary economic environment in which that entity operates. It would be the British Pound for a company from the United Kingdom that uses British pounds in its normal operations.

297
Q

The authoritative guidance on business combinations states which of the following?
A
A business combination occurs when an acquirer obtains control over one or more businesses.
B
If intercorporate stock ownership arrangements result in a parent company owning less than 50 percent of a third lower-level subsidiary, through the parent’s second subsidiary (a chain of interests), consolidated financial statements should not be prepared.
C
Consolidation of majority-owned subsidiaries is not required if a large minority interest exists in the subsidiaries.
D
Consolidation of majority-owned subsidiaries is not required if a subsidiary is in a foreign location

A

Explanation:

A business combination occurs when an acquirer obtains control over one or more businesses.

298
Q
On September 29, Wall Co. paid $860,000 for all the issued and outstanding common stock of Hart Corp. On that date, the carrying amounts of Hart's recorded assets and liabilities were $800,000 and $180,000, respectively. Hart's recorded assets and liabilities had fair values of $840,000 and $140,000, respectively. In Wall's September 30 balance sheet, what amount should be reported as goodwill?
A
$ 20,000
B
$160,000
C
$180,000
D
$240,000
A

Explanation:
Goodwill is recognized and recorded at an amount equal to the excess of the cost of the enterprise acquired, plus the fair value of any noncontrolling interest, over the fair value of the identifiable net assets.

Purchase price of 100% of Hart Corp’’s O/S common stock $860,000
Less: Fair value of identifiable net assets of Hart Corp. (700,000)
Goodwill $ 160,000

299
Q
Pine City owned a vacant plot of land zoned for industrial use. Pine gave this land to Medi Corp. solely as an incentive for Medi to build a factory on the site. The land had a fair value of $300,000 at the date of the gift. This nonmonetary transaction should be reported by Medi as
A
Extraordinary income.
B
Additional paid-in capital.
C
A credit to retained earnings.
D
A memorandum entry.
A

Explanation:
The contribution of land by a governmental unit to an enterprise for industrial use is an example of a nonreciprocal transfer. A nonmonetary asset received in a nonreciprocal transfer should be recorded at the fair value of the asset received. The corresponding credit for a corporation is ‘Additional Paid-in Capital– Donated Assets.’ Donated assets should not be recorded as income or gain or added to retained earnings when received from governmental entities. Assets donated by entities other than governmental units should be included in revenue in the period of receipt.

300
Q

Brill Co. made the following expenditures during the current year:
Costs to develop computer software for internal use in Brill’s general management information system $100,000
Costs of market research activities 75,000What amount of these expenditures should Brill report in its current year income statement as research and development expenses?
A
$175,000
B
$100,000
C
$75,000
D
$0

A

Explanation:
Neither of the two activities identified is an example of an activity that is typically considered a research and development (R&D) activity. Most costs in the application development stage of developing computer software for internal use are capitalized and not expensed as incurred. Market research activities related to the selling and marketing, not R&D.

301
Q

A parent decides to spin off a subsidiary and deconsolidates it through a nonreciprocal transfer to owners. How shall the parent account for and report the transaction?

A
As a noncontrolling interest
B
As a nonmonetary transaction
C
As an item in Discontinued Operations
D
As a gain or loss in net income attributable to the parent
A

Explanation:
The correct answer is (B).

If a parent deconsolidates a subsidiary through a nonreciprocal transfer to owners, such as a spinoff, they shall use nonmonetary transaction accounting guidance in reporting the transaction.

Otherwise, a parent accounts for the deconsolidation by recognizing a gain or loss in net income attributable to the parent. The gain or loss is measured as the difference between the aggregate of the value of any consideration received plus the fair value of any retained noncontrolling investment in the former subsidiary at the date of deconsolidation plus the carrying amount of any noncontrolling interest in the former subsidiary (including any accumulated OCI attributable to the noncontrolling interest) at the date of deconsolidation versus the carrying amount of the former subsidiary’s assets and liabilities.

302
Q

The primary purpose of a quasi-reorganization is to give a corporation the opportunity to
A
Obtain relief from its creditors.
B
Revalue understated assets to their fair values.
C
Eliminate a deficit in retained earnings.
D
Distribute the stock of a newly-created subsidiary to its stockholders in exchange for part of their stock in the corporation.

A

Explanation:
The primary purpose of a quasi-reorganization is to eliminate an accumulated deficit (negative retained earnings balance) so that the corporation has a ‘fresh start’ with a zero balance in retained earnings. Although the accounting procedures for a quasi-reorganization involve restating assets of the enterprise to their fair values, this is not the primary purpose.

303
Q

On March 21, year 2, a company with a calendar year end issued its year 1 financial statements. On February 28, year 2, the company’s only manufacturing plant was severely damaged by a storm and had to be shut down. Total property losses were $10 million and determined to be material. The amount of business disruption losses is unknown. How should the impact of the storm be reflected in the company’s year 1 financial statements?
A
Provide no information related to the storm losses in the financial statements until losses and expenses become fully known.
B
Accrue and disclose the property loss with no accrual or disclosure of the business disruption loss.
C
Do not accrue the property loss or the business disruption loss, but disclose them in the notes to the financial statements.
D
Accrue and disclose the property loss and additional business disruption losses in the financial statements.

A

Explanation:
An entity shall not recognize subsequent events that provide evidence about conditions that did not exist at the date of the balance sheet but arose after the balance sheet date but before financial statements are issued or are available to be issued. Some nonrecognized subsequent events may be of such a nature that they must be disclosed to keep the financial statements from being misleading. Since property losses were of a material nature, the entity should disclose the nature of the event and an estimate of its financial effect or a statement that such an estimate cannot be made.

304
Q

On December 29, year 1, Action Corp. signed a 7-year finance lease for an airplane to transport its sports team around the country. The airplane’s fair value was $841,500. Action made the first annual lease payment of $153,000 on December 31, year 1. Action’s incremental borrowing rate was 12%, and the interest rate implicit in the lease, which was known by Action, was 9%. The following are the rounded present value factors for an annuity due:

9% for 7 years 5.5
12% for 7 years 5.1
What amount should Action report as finance lease liability in its December 31, year 1 balance sheet?

A
$841,500
B
$780,300
C
$688,500
D
$627,300
A

Explanation:
The lessee records a finance lease as ROU asset and lease liability. Lease Liability (regardless of lease classification) is calculated as the present value of the lease payments not yet paid by using the lease term and discount rate determined at lease commencement. The PV of Lease Payments is computed using the lessor’s implicit rate in the lease of 9%, since it is known to the lessee. Since the annual rental payment is payable at the beginning of each lease year, the PV factor for an annuity due is used. The finance lease obligation is recorded at the PV of the Lease Payments ($153,000 × 5.5 = $841,500) on 12/29. The finance lease liability in the 12/31, year 1 balance sheet reflects the first annual rental payment made 12/29 ($841,500 – $153,000 = $688,500).

305
Q

Which of the following examples would require restatement of prior years’ financial statements?
A
A calculation change of warranty obligations based on updated claim information for the prior year
B
A change from the income tax basis of accounting to the accrual basis
C
An insurance premium that was due in the prior year but that lapsed because the policy was not paid
D
An intangible asset with a remaining estimated amortization period of two years, which is determined to be obsolete

A

Explanation:
A change in accounting principle is a change from one generally accepted accounting principle to another when there are two or more generally accepted accounting principles that apply or the principle formerly used is no longer generally accepted. A change in the method of applying an accounting principle is also a change in accounting principle. Hence a change from the income tax basis of accounting to the accrual basis is a change in accounting principle. A change in accounting principle is reported through retrospective application of the new principle to all prior periods (restatement). A calculation change of warranty obligations based on updated claim information for the prior year and an intangible asset with a remaining estimated amortization period of two years which is determined to be obsolete would both be a change in accounting estimate. A change in estimate is applied prospectively; it does not require restatement or retrospective adjustment to prior period financial statements. An insurance premium that was due in the prior year but that lapsed because the policy was not paid would require no special accounting change.

306
Q
Nolan owns 100% of the capital stock of both Twill Corp. and Webb Corp. Twill purchases merchandise inventory from Webb at 140% of Webb's cost. During the current year, merchandise that cost Webb $40,000 was sold to Twill. Twill sold all of this merchandise to unrelated customers for $81,200 during this year. In preparing combined financial statements for the year, Nolan's bookkeeper disregarded the common ownership of Twill and Webb. What amount should be eliminated from cost of goods sold in the combined income statement for the year?
A
$56,000
B
$40,000
C
$24,000
D
$16,000
A

Explanation:
In recording the sale of inventory to Twill, Webb recognized cost of goods sold of $40,000. In recording the later sale this inventory to an unrelated customer, Twill recognized cost of goods sold of $56,000 ($40,000 × 140%). However, from a combined perspective, the sale to Twill (the affiliated company) did not occur, and thus the cost of the goods sold to the unaffiliated company is $40,000. Therefore, $56,000 is eliminated from cost of goods sold in the combined income statement.

307
Q

On December 30 of the current year, Haber Co. leased a new machine from Gregg Corp. The following data relate to the lease transaction at the inception of the lease:

Lease term 10 years
Annual rental payable at the end of each lease year $100,000
Useful life of machine 12 years
Implicit interest rate 10%
Present value of an annuity due of $1 for 10 periods at 10% 6.76
Present value of an ordinary annuity of $1 for 10 periods at 10% 6.15
Fair value of the machine $700,000
The lease has no renewal option, and the possession of the machine reverts to Gregg when the lease terminates. At the inception of the lease, Haber should record a lease liability of

A
$0
B
$615,000
C
$630,000
D
$676,000
A

Explanation:
The correct answer is (B).

Per ASC 842, at lease commencement, the lessee must measure their lease assets and liabilities at the present value of the lease payments using either the lessor’s implicit interest rate or if not readily determinable, at lessee’s incremental borrowing rate.

Since lease payments are payable at end of each year, we will use the Present Value of an ordinary annuity of $1 for 10 periods at 10% (the implicit interest rate is given), which is 6.15. That will give us the lease liability of $100,000 x 6.15 = $615,000.

Option (A), (C) and (D) are incorrect as per above explanation.

308
Q

Question #178, Blueprint Area: Derivatives and Hedge Accounting (e.g. Swaps, Options, Forwards)
How should gains or losses from fair value hedges be recognized?
A
As an extraordinary item in the period of fair value change because of the unusual and infrequent nature of derivative contracts.
B
The gain or loss, along with the offsetting loss or gain attributable to the hedged risk, should be recognized currently in earnings in the same accounting period.
C
As a component of other comprehensive income in the period of fair value change and subsequently in earnings in the period net settlement occurs.
D
No gain or loss recognition in the period of fair value change, but subsequent recognition of gain or loss in earnings in the period net settlement occurs.

A

Explanation:
With fair value hedges, the gain or loss, along with the offsetting loss or gain attributable to the hedged risk, should be recognized currently in earnings in the same accounting period. Net loss or gain in the hedging activity indicates the effectiveness of the hedge.

309
Q

Which of the following is not a criterion for the lessor to classify a lease as either a sales-type or direct-financing-type lease?

A
The lease is a finance lease for the lessee.
B
The lease transfers ownership of the underlying asset to the lessee by the end of the lease term.
C
The lease grants the lessee an option to purchase the underlying asset that the lessee is reasonably certain to exercise.
D
The underlying asset is of such a specialized nature that it is expected to have no alternative use to the lessor at the end of the lease term.

A

Explanation:
“The lease is a finance lease for the lessee” is not a criterion for determination of lease classification by the lessor.

A lessor should classify a lease as a sales-type lease when the lease meets any one of the following criteria at lease commencement (these are the same criteria for a Finance Lease):

Present Value equals or exceeds substantially all (90%) of the Fair Value
Option to Purchase (exercise is reasonably certain)
Economic Life - Major part (75%) of asset’s economic life is used
Transfer of Ownership at lease termination
Specialized Nature - No alternative use to the lessor at lease termination
Note: the implementation guidance for ASC 842 uses the 75/90 thresholds, even though the standard is principles-based.

310
Q

Sun Co. is a wholly owned subsidiary of Star Co. Both companies have separate general ledgers, and prepare separate financial statements. Sun requires stand-alone financial statements. Which of the following statements is correct?
A
Consolidated financial statements should be prepared for both Star and Sun.
B
Consolidated financial statements should only be prepared by Star and not by Sun.
C
After consolidation, the accounts of both Star and Sun should be changed to reflect the consolidated totals for future ease in reporting.
D
After consolidation, the accounts of both Star and Sun should be combined together into one general-ledger accounting system for future ease in reporting.

A

Explanation:
Consolidated statements are required when a company owns more than 50% of the voting stock in another company. Star owns more than 50% of Sun; Sun does not own more than 50% of Star, therefore, Star would prepare consolidated financial statements and Sun would not. Consolidating entries are prepared on a worksheet only and are not formally entered into the books of either company. Consolidating procedures must be preformed every period in which financial statements are presented.

311
Q
On January 1, year 1, Newport Corp. purchased a machine for $100,000. The machine was depreciated using the straight-line method over a 10-year period with no residual value. Because of a bookkeeping error, no depreciation was recognized in Newport's year 1 financial statements, resulting in a $10,000 overstatement of the book value of the machine on December 31, year 1. The oversight was discovered during the preparation of Newport's year 2 financial statements. What amount should Newport report for depreciation expense on the machine in the year 2 financial statements?
A
$ 9,000
B
$10,000
C
$11,000
D
$20,000
A

Explanation:
Errors in financial statements result from mathematical mistakes, mistakes in the application of accounting principles, or the oversight or misuse of facts that existed at the time the financial statements were prepared. For an item to be classified as a prior period adjustment, it must be an item of profit or loss related to the correction of an error in the financial statements of a prior period. Any error in the financial statements of a prior period discovered subsequent to their issuance shall be reported as an adjustment to the beginning balance of retained earnings, net of their income tax effect, in the statement of retained earnings. Therefore, Newton would report normal depreciation expense of $10,000 in the year 2 financial statements.

312
Q

On January 1 of the current year, Tell Co. leased equipment from Swill Co. under a nine-year lease. The equipment had a cost of $400,000, and an estimated useful life of 15 years. Semiannual lease payments of $44,000 are due every January 1 and July 1. The present value of lease payments at 12% was $505,000, which equals the sales price of the equipment. Using the straight-line method, what amount should Tell recognize as depreciation expense on the equipment in the current year?

A
$26,667
B
$33,667
C
$44,444
D
$56,111
A

Explanation:
A Finance Lease must meet one of the following criteria:

Present Value equals or exceeds substantially all (90%) of the Fair Value
Option to Purchase (exercise is reasonably certain)
Economic Life - Major part (75%) of asset’s economic life is used
Transfer of Ownership at lease termination
Specialized Nature - No alternative use to the lessor at lease termination
Note: the implementation guidance for ASC 842 uses the 75/90 thresholds, even though the standard is principles-based.

Since the present value of the sum of lease payments ($505,000 > $400,000) substantially exceeds all of the fair value of underlying asset the lease is classified as a finance lease, the lessee will amortize the ROU asset over the lease term). Depreciation is calculated on the present value of the lease payments because that amount will be capitalized to ROU asset at inception. ($505,000 / 9 years = $56,111)

313
Q
On December 31 of the previous year, Byte Co. had capitalized software costs of $600,000 with an economic life of four years. Sales for the current year were 10% of expected total sales of the software. At December 31 of the current year the software had a net realizable value of $480,000. In its December 31, current year balance sheet, what amount should Byte report as net capitalized cost of computer software?
A
$432,000
B
$450,000
C
$480,000
D
$540,000
A

Explanation:
The annual amortization of computer software costs is the greater of the amount computed using the percentage of revenue approach and the straight-line method applied over the product’s remaining estimated economic life. $600,000 - ($600,000/4) = $450,000. Capitalized computer software costs are carried at the lower of unamortized cost or net realizable value.

314
Q

Conlon Co. is the plaintiff in a patent-infringement case. Conlon has a high probability of a favorable outcome, and can reasonably estimate the amount of the settlement. What is the proper accounting treatment of the patent infringement case?
A
A gain contingency for the minimum estimated amount of the settlement.
B
A gain contingency for the estimated probable settlement.
C
Disclosure in the notes only.
D
No reporting is required at this time.

A

Explanation:
Gain contingencies should be disclosed but not recognized as income. Care should be taken to avoid misleading implications as to the likelihood of realization.

315
Q

Under IFRS 9, an entity’s financial assets can be classified and measured in which ways?

A
Held to Maturity, Available-for-sale and Trading
B
Fair Value through Profit of Loss and Amortized Cost
C
Fair Value through Profit or Loss, Amortized Cost and Fair Value through Other Comprehensive Income
D
Available-for-Sale and Amortized Cost

A

Explanation:
Under IFRS 9, the default classification of financial assets is “Fair Value through Profit or Loss’ (FVTPL). FVTPL includes any financial assets that are held for trading and are also derivatives. Changes in fair value are reported through profit or loss at each reporting date. A second classification of financial assets is “Fair Value through Other Comprehensive Income” (FVTOCI). Changes in fair value are reported through other comprehensive income at each reporting date. Finally the third classification that can be measured under IFRS is amortized cost.

Options (A), (B) and (D) are incorrect as per above explanation.

316
Q

Invern Inc. has a self-insurance plan. Each year, retained earnings is appropriated for contingencies in an amount equal to insurance premiums saved less recognized losses from lawsuits and other claims. As a result of an accident this year, Invern is a defendant in a lawsuit in which it will probably have to pay damages of $190,000. What are the effects of this lawsuit’s probable outcome on Invern’s current year financial statements?
A
An increase in expenses and no effect on liabilities
B
An increase in both expenses and liabilities
C
No effect on expenses and an increase in liabilities
D
No effect on either expenses or liabilities

A

Explanation:
The potential loss for damages that may be paid should be reported by accruing a loss in the income statement and a liability in the balance sheet. Accrual is required because both of the following conditions are met: (1) it is considered probable that a liability has been incurred, and (2) the amount of the loss can be reasonably estimated. In addition, the nature of the lawsuit should be separately disclosed in the notes to the financial statements. The loss should not be charged to the appropriation of retained earnings for contingencies.

317
Q

Regarding computer software, what action is not taken for the costs of producing product masters incurred subsequent to establishing technological feasibility?
A
The costs are charged to expense when incurred, as part of research and development.
B
The costs are capitalized.
C
The costs are reported at the lower of unamortized cost or net realizable value.
D
The costs are amortized on a product-by-product basis.

A

Explanation:
Costs incurred internally in creating a computer software product are charged to expense when incurred, as research and development until technological feasibility has been established for the product. Costs of producing product masters incurred subsequent to establishing technological feasibility are capitalized.

318
Q

On June 1 of the current year, a company entered into a real estate lease agreement for a new building. The lease is an operating lease and is fully executed on that day. According to the terms of the lease, payments of $28,900 per month are scheduled to begin on October 1 of the current year and to continue each month thereafter for 56 months. The lease term spans five years. The company has a calendar year-end. What amount is the company’s lease expense for the current calendar year?

A
$ 86,700
B
$161,838
C
$188,813
D
$202,300
A

Explanation:
The correct answer is (C).

For an operating lease, when uneven lease payments are made, total rent expense payable for the entire lease term is divided evenly over each period in line with matching principle.

a Building lease per month $28,900
b Actual rent paid for number of months 56
c Lease terms in months (5 years x 12 months) 60
d Total rent paid for 56 months (a x b) $1,618,400
e Rent expense per month (d/c) $26,973
f Year 1 Rent from 1st Jun to 31st Dec (e x 7 Months) $188,813

319
Q

Whether recognized or unrecognized in an entity’s financial statements, disclosure of the fair values of the entity’s financial instruments is required when
It is practicable to estimate those values Aggregated fair values are material to the entity
A No No
B No Yes
C Yes No
D Yes Yes

A

Explanation:
An entity is required to disclose, either in the body of the financial statements or in the accompanying notes, the fair value of financial instruments for which it is practicable to estimate that value and the method(s) and significant assumptions used to estimate the fair value of financial instruments. These provisions need not be applied to immaterial items.

320
Q

During the current year, Orca Corp. decided to change from the FIFO method of inventory valuation to the weighted-average method. Inventory balances under each method were as follows:

FIFO Weighted-average

January 1 $71,000 $77,000
December 31 79,000 83,000
Orca’s income tax rate is 30%. In its year-end financial statements, what amount should Orca report as the cumulative effect of this accounting change?

A
$2,800
B
$4,000
C
$4,200
D
$6,000
A

Explanation:
A change from the FIFO method of inventory valuation to the weighted-average method is a change in accounting principle.

The cumulative effect of the change in accounting principle is the difference between the amount of retained earnings at the beginning of the period of change and the amount of retained earnings that would have been reported at that date if the new accounting principle had been applied retrospectively for all affected periods.

Since the new (weighted-average) method results in a $6,000 higher inventory valuation than the old (FIFO) method at the beginning of the period of change, the amount of expense recognized as cost of goods sold in prior periods is reduced by $6,000.

This has the effect of increasing the beginning balance of retained earnings.

The beginning balance of retained earnings cannot be increased by the full $6,000, because the reduction in the cost of goods sold would have increased the amount of income tax expense by $1,800 (i.e., $6,000 × 30%).

Since it’s a change from FIFO to Weighted Average, the Beginning Inventory, which is Ending Inventory last year, would be $6,000 more.

Retained earnings would be $6000 x .7 = $4,200 higher.

321
Q

To determine a lease’s discount rate, a lessee should use

A
The rate implicit in the lease + prime rate
B
The prime rate + lessee's incremental borrowing rate
C
The prime rate
D
The rate implicit in the lease
A

Explanation:
The correct answer is (D).

Per ASC 842, to determine a lease’s discount rate, a lessee should use the rate implicit in the lease whenever that rate is readily determinable.

If the rate implicit in the lease is not readily determinable (a CPA Exam question will likely tell you that it’s not readily determinable), a lessee should use its own Incremental Borrowing Rate (IBR) as the discount rate.

322
Q

Which of the following statements is correct regarding fair value measurement?

A
Fair value is a market-based measurement
B
Fair value is an entity-specific measurement
C
Fair value measurement does not consider risk
D
Fair value measurement does not consider restrictions

A

Explanation:
The correct answer is (A).

Fair value measurement is a market-based measurement as the fair value is measured based on the principal market. Fair value is defined as the price that would be received to sell an asset or paid to transfer a liability in an orderly arm’s length transaction between market participants at the measurement date.

323
Q

On December 31 of the current year, Day Co. leased a new machine from Parr with the following pertinent information:

Lease term 6 years
Annual rental payable at beginning of each year $50,000
Useful life of machine 8 years
Day’s incremental borrowing rate 15%
Implicit interest rate in lease (known by Day) 12%
Present value of an annuity due of 1 for 6 periods at 12% 4.61
Present value of an annuity due of 1 for 6 periods at 15% 4.35
The lease is not renewable, and the machine reverts to Parr at the termination of the lease. The cost of the machine on Parr’s accounting records is $375,500.
At the beginning of the lease term, Day should record a lease liability of

A
$375,500
B
$230,500
C
$217,500
D
$0
A

Explanation:
The correct answer is (B).

A lessee will record as an initial liability, the lower of cost (present value of minimum lease payments) or fair market value. The present value is discounted at the rate implicit in the lease or the rate available in the market to the lessee (the lessee’s incremental borrowing rate) if the implicit rate is unknown.

A Finance Lease must meet one of the following criteria:

Present Value equals or exceeds substantially all (90%) of the Fair Value
Option to Purchase (exercise is reasonably certain)
Economic Life - Major part (75%) of asset’s economic life is used
Transfer of Ownership at lease termination
Specialized Nature - No alternative use to the lessor at lease termination
Note: the implementation guidance for ASC 842 uses the 75/90 thresholds, even though the standard is principles-based.

The machine leased by Day Co would be accounted for as a finance lease since the lease is for 6 years and the asset’s machine life is 8 years, the lease is for the majority of the useful life of the machine.

The lease obligation will be recorded at the present value of the minimum lease payments using the rate implicit in the lease since it is known to Day.

Therefore lease obligation = $50,000 x 4.61 = $230,500.

Option (A) is incorrect because $375,500 is the value of the machine in the Parr’s accounting record.

Option (C) is incorrect because the present value is calculated using the discount rate of 15% since the implicit rate in the lease is known.

Option (D) is incorrect because the liability cannot be recorded at zero.

324
Q

Which of the following statements is correct as it relates to changes in accounting estimates?

A
Most changes in accounting estimates are accounted for retrospectively.
B
Whenever it is impossible to determine whether a change in an estimate or a change in accounting principle occurred, the change should be considered a change in principle.
C
Whenever it is impossible to determine whether a change in accounting estimate or a change in accounting principle has occurred, the change should be considered a change in estimate.
D
It is easier to differentiate between a change in accounting estimate and a change in accounting principle than it is to differentiate between a change in accounting estimate and a correction of an error.

A

Explanation:
When there is a change in accounting principle inseparable from a change in accounting estimate, it is treated as a change in accounting estimate.

Option (A) is incorrect because accounting estimates are treated prospectively.
Option (B) is incorrect because the change should be considered a change in estimate.
Option (D) is incorrect because facts and circumstances determine the difficulty level of accounting changes and error corrections.

325
Q

Bain Co. entered into a 10-year lease agreement for a new piece of equipment worth $500,000. At the end of the lease, the asset will be transferred to Bain Co. Which of the following would require the lease to be accounted for as a finance lease?

A
The lease includes an option to purchase stock in the company.
B
The estimated useful life of the leased asset is 18 years.
C
The present value of the minimum lease payments is $400,000.
D
The lease transfers ownership of the underlying asset to the Bain Co. by the end of the lease term.

A

Explanation:
The correct answer is (D).

Since the equipment is transferred to Bain Co. at the end of lease term, it should be classified as Finance Lease.

A Finance Lease must meet one of the following criteria:

Present Value equals or exceeds substantially all (90%) of the Fair Value
Option to Purchase (exercise is reasonably certain)
Economic Life - Major part (75%) of asset’s economic life is used
Transfer of Ownership at lease termination
Specialized Nature - No alternative use to the lessor at lease termination
Note: the implementation guidance for ASC 842 uses the 75/90 thresholds, even though the standard is principles-based.

(A) is incorrect because the option is for purchasing stock in the company, not the asset.

(B) is incorrect because the 10-year lease is less than 75% (56%) of the asset’s economic life.

(C) is incorrect because $400,000 is less than 90% (80%) of the FV of the asset.

326
Q

How should NSB, Inc. report significant research and development costs incurred?
A
Expense all costs in the year incurred.
B
Capitalize the costs and amortize over a five-year period.
C
Capitalize the costs and amortize over a 40-year period.
D
Expense all costs two years before and five years after the year incurred.

A

Explanation:
Research activities are those aimed at the discovery of knowledge that will be useful in developing or significantly improving products or processes. Development activities are those concerned with translating research findings and other knowledge into plans or designs for new or significantly improved products or processes. Since future economic benefits deriving from R&D activities are uncertain in their amount and timing, most R&D costs are required to be charged to expense the year in which incurred.

327
Q

Which of the following is a research and development cost?
A
Development or improvement of techniques and processes
B
Offshore oil exploration that is the primary activity of a company
C
Research and development performed under contract for others
D
Market research related to a major product for the company

A

Explanation:
Research activities are those aimed at the discovery of knowledge that will be useful in developing or significantly improving products or processes and development activities are those concerned with translating research findings and other knowledge into plans or designs for new or significantly improved products, techniques, or processes. Offshore oil exploration as a primary activity is the main job function and not a R & D cost. Research and development under contract for others is a service and not a R & D cost. Market research for a product is a company sales expense and not a R & D cost.

328
Q

Smythe Co. invested $200 in a call option for 100 shares of Gin Co. $.50 par common stock, when the market price was $10 per share. The option expired in three months and had an exercise price of $9 per share. What was the intrinsic value of the call option at the time of initial investment?

A
$50
B
$100
C
$200
D
$900
A

Explanation:
A call option represents the right, but not the obligation, to buy a set number of shares of stock at a predetermined ‘strike price’ before the option reaches its expiration date. A call option is purchased in hopes that the underlying stock price will rise in the future well above the strike price. Entities can use the BlackScholes, binomial, or similar pricing models to value the options. In this case, limited information is provided, so the value of the options would be the difference between the current/ market price and the strike/exercise price: 100 * ($10 – $9) = $100.

329
Q

On December 30 of the current year, Frey Company leased equipment from Hull. Pertinent lease transaction data are as follow:

The estimated seven-year useful equipment life coincides with the lease term.
The first of the seven equal annual $200,000 lease payments was paid on December 31 of the current year.
Hull’s implicit interest rate of 12% is known to Frey.
Frey’s incremental borrowing rate is 14%.
Present values of an annuity due of $1 for seven periods are 5.11 at 12%, and 4.89 at 14%, respectively.
At the inception of the lease, Frey should record a finance lease liability of

A
$1,400,000
B
$1,022,000
C
$978,000
D
$0
A

Explanation:
The correct answer is (B).

A Finance Lease must meet one of the following criteria:

Present Value equals or exceeds substantially all (90%) of the Fair Value
Option to Purchase (exercise is reasonably certain)
Economic Life - Major part (75%) of asset’s economic life is used
Transfer of Ownership at lease termination
Specialized Nature - No alternative use to the lessor at lease termination
Note: the implementation guidance for ASC 842 uses the 75/90 thresholds, even though the standard is principles-based.

If none of the above criteria is met, the lease is an operating lease for the lessee.

The lease qualifies as a finance lease since the lease term is for the majority of the useful life of the property (7-year lease ÷ 7-year economic life = 100%).

The finance lease liability is computed by multiplying the periodic minimum lease payments by the appropriate factor for the
implicit rate of the lease. The implicit interest rate of the lease of 12% is used to capitalize the lease liability because it is known by the lessee. If the implicit rate was not known, then the incremental borrowing rate of the lessee would have been used.

Annual minimum lease payments $ 200,000
Appropriate factor for implicit interest rate of 12% × 5.11
Finance lease liability $1,022,000

330
Q

Shore Co. records its transactions in U.S. dollars. A sale of goods resulted in a receivable denominated in Japanese yen, and a purchase of goods resulted in a payable denominated in euros. Shore recorded a foreign exchange gain on collection of the receivable and an exchange loss on settlement of the payable. The exchange rates are expressed as so many units of foreign currency to one dollar. Did the number of foreign currency units exchangeable for a dollar increase or decrease between the contract and settlement dates?
Yen exchangable for $1 Euros exchangable for $1
A Increase Increase
B Decrease Decrease
C Decrease Increase
D Increase Decrease

A

Explanation:
To record a foreign exchange gain on collection of the receivable, the exchange rate of Japanese yen to one U.S. dollar must have decreased. For instance, assume a receivable for l,000 yen and an exchange rate of 10 yen to 1 dollar at the contract date. If the exchange rate changed to 5 yen to 1 dollar, the 1,000 yen when collected could be converted into 200 U.S. dollars at the settlement date. This $200 is twice the $100 that could have been obtained from the conversion of currency at the contract date (1,000 / 10 = $100). To record an exchange loss on settlement of the payable, the exchange rate of euros to one dollar must have decreased. For example, assume a payable for 1,000 euros and an exchange rate of 4 euros to 1 dollar at the contract date. If the exchange rate changed to 2 euros to 1 dollar, the 1,000 euross paid at the settlement date required more U.S. dollars than would have been required at the date of contract (1,000 / 2 = $500 at date of settlement versus 1,000 / 4 = $250 at date of contract).

331
Q

On June 1, year 1, ABC Co. issued a 200,000 euro purchase order for equipment to be supplied by a German company. ABC’s functional currency is the U.S. dollar. The equipment was delivered to ABC on November 1, year 1, and ABC recorded a payable due to the German company. ABC paid for the equipment on January 31, year 2. The following are the exchange rates in effect:

June 1, year 1 1 euro = 1.40 U.S. dollars
November 1, year 1 1 euro = 1.50 U.S. dollars
December 31, year 1 1 euro = 1.35 U.S. dollars
January 31, year 2 1 euro = 1.30 U.S. dollars
Under IFRS, what is the foreign currency gain or loss that ABC should record for the year ended December 31, year 1?

A
A loss of $30,000.
B
A loss of $20,000.
C
A gain of $10,000.
D
A gain of $30,000.
A

Explanation:
The correct answer is (D).

A foreign currency transaction gain or loss is recognized in the period when the exchange rate changes, e.g., at a balance sheet date and at the settlement date. ABC’s functional currency is the US dollar. Therefore, the transaction must be measured in the US dollar.

A gain of $30,000 is reported on 31st Dec, year 1.

Date Unit Exchange Rate USD / GBP Amount (USD)
01- Nov 200,000 $1.50 300,000
31- Dec 200,000 $1.35 270,000
Gain $30,000
(A) and (B) are incorrect because transaction on 31st Dec will result in gain; not a loss.

(C) is incorrect because $10,000 gain is recognized when the payment is made, on Dec 31, gain of $30,000 is recognized.

332
Q
On July 1, in an exchange with commercial substance, Balt Co. exchanged a truck for 25 shares of Ace Corp.'s common stock. On that date, the truck's carrying amount was $2,500, and its fair value was $3,000. Also, the book value of Ace's stock was $60 per share. On December 31, Ace had 250 shares of common stock outstanding and its book value per share was $50. What amount should Balt report in its December 31 balance sheet as investment in Ace?
A
$3,000
B
$2,500
C
$1,500
D
$1,250
A

Explanation:
Except for certain transactions with employees, all transactions in which goods or services are the consideration received for the issuance of equity instruments shall be accounted for based on the fair value of the consideration received or the fair value of the equity instruments issued, whichever is more reliably measurable. An exchange of nonmonetary assets for an equity interest is not an exchange of productive assets.

333
Q

On April 1, year 1, Hall Fitness Center leased its gym to Dunn Fitness Center under a four-year operating lease. Hall normally charges $6,000 per month to lease its gym, but as an incentive, Hall gave Dunn half off the first year’s rent, and one quarter off the second year’s rent. Dunn’s rental payments were as follows:

Year 1	12 × $3,000 = $36,000
Year 2	12 × $4,500 = $54,000
Year 3	12 × $6,000 = $72,000
Year 4	12 × $6,000 = $72,000
Dunn's rent payments were due on the first day of the month, beginning on April 1, year 1. What amount should Dunn report as rent expense in its monthly income statement for April, year 3?
A
$3,000
B
$4,500
C
$4,875
D
$6,000
A

Explanation:
The correct answer is (C).

Total rent expense payable for the entire lease term is divided evenly over each period in line with matching principle.

 	Amount
Year 1	$36,000
Year 2	$54,000
Year 3	$72,000
Year 4	$72,000
Total Rent	$234,000
No. of Months	48
Rent per month	$4,875
Option (A) is incorrect because $3,000 is the actual rent paid in year 1 April.

Option (B) is incorrect because $4,500 is the per-month rent for the year 2 if the actual rent paid is evenly spread ($54,000/9).

Option (D) is incorrect because $6,000 is the actual rent paid per month

334
Q

Company J acquired all of the outstanding common stock of Company K in exchange for cash. The acquisition price exceeds the fair value of net assets acquired. How should Company J determine the amounts to be reported for the plant and equipment and long-term debt acquired from Company K?
Plant and equipment Long-term debt
A K’s carrying amount K’s carrying amount
B K’s carrying amount Fair value
C Fair value K’s carrying amount
D Fair value Fair value

A

Explanation:
All identifiable assets acquired and liabilities assumed are assigned a portion of the cost, normally equal to their fair values at the date of acquisition. The excess of the acquisition price over the fair value of the net identifiable assets acquired is goodwill.

335
Q

Which of the following items would not be considered a change in accounting estimate?
A
A change in the useful lives or salvage values of depreciable assets
B
A change in the recovery periods benefited by a deferred cost
C
A change in the method of accounting for long-term construction contracts
D
A change in expected losses on receivables

A

Explanation:
A change in the method of accounting for long-term construction contracts would be considered a change in accounting principle, not a change in accounting estimate.

336
Q

On August 31, year 2, Harvey Co. decided to change from the FIFO periodic inventory system to the weighted average periodic inventory system. Harvey is on a calendar year basis. The cumulative effect of the change is determined
A
As of January 1, year 2.
B
As of August 31, year 2.
C
During the eight months ending August 31, year 2, by a weighted average of the purchases.
D
During year 2 by a weighted average of the purchases.

A

Explanation:
A change from the FIFO periodic inventory system to the weighted average periodic inventory system is a change in accounting principle. The cumulative effect of the change is the difference between the amount of retained earnings at the beginning of the period of change and the amount of retained earnings that would have been reported at that date if the new accounting principle had been retrospectively applied for all affected periods. Since Harvey is on a calendar year basis, the cumulative effect of the change in accounting principle should be determined as of January 1.

337
Q

On December 31, Pell, Inc., sold a machine to Flax, and simultaneously leased it back for one year. Pertinent information on this date is as follows:

Sales price $360,000
Carrying amount 315,000
Estimated remaining useful life 12 years
Present value of lease rentals ($3,000 for 12 months @ 12%) 34,100
At December 31, how much should Pell report as deferred revenue from the sale of the machine?

A
$0
B
$10,900
C
$34,100
D
$45,000
A

Explanation:
If the leaseback is an operating lease from seller-lessee’s perspective, transfer of the asset is a Sale.

The Seller-Lessee will recognize the gain on Sale of the transferred asset immediately and no gain will be deferred.

In this problem, the gain of $45,000 will be recognized and immediately.

In this case, the sale and leaseback are accounted for as two separate transactions (i.e., the entire gain is recognized upon the sale of the property).

338
Q

A company has an equity investment with a historical cost of $500,000 that is traded in an active market. At December 31, year 1, the quoted price for an identical investment was $400,000 and the quoted price for a similar investment was $430,000. Using the company’s internal present value of cash flows model, the company arrived at a value of $410,000. What amount is the value of the investment on December 31, year 1?

A
$400,000
B
$410,000
C
$430,000
D
$500,000
A

Explanation:
The correct answer is (A)

When a company has marketable equity investments, they are to be classified at their fair values at year-end. For valuing an asset at its fair value, US GAAP states the following hierarchy of levels:

Level 1: Use quoted price from active markets for an identical asset
Level 2: In the absence of an identical asset, use directly or unobservable inputs like a quoted price from active markets for a similar asset
Level 3: In absence of both levels 1 and 2, use unobservable inputs. This includes the entity’s own assumptions about the market like financial forecasts or a cash flow model using internal present value.
In the given case, the quoted price of an identical investment from an active market is available. Hence, consideration of other values is not necessary.

The company should value its equity investment at $400,000

339
Q

A collection agency spent $50,000 in staff payroll costs investigating the feasibility of developing its own software program for tracking customer contacts. After committing to funding the project, software developers were paid $200,000 to write the code, and the company incurred $70,000 in general and administrative costs related to training and software maintenance. What amount should be capitalized?

A
$200,000
B
$250,000
C
$270,000
D
$320,000
A

Explanation:
The correct answer is (A).

Software development process:

Preliminary project stage: All costs in this stage, including investigating the feasibility of developing its own software, should be expensed. Hence, $50,000in staff payroll costs investigating the feasibility of developing its own software program should be expensed here.
Application development stage: Costs incurred in the application development stage, including writing the code, are usually capitalized, except for training costs, which are expensed. The $200,000 paid to write the code should be capitalized whereas $70,000 in general and administrative costs related to training and software maintenance should be expensed.
Post-implementation stage: Once the product is market ready and in saleable condition, all costs should be expensed.
Thus, the company should only capitalize the $200,000 paid to the developers.

340
Q

A company acquires another company for $3,000,000 in cash, $10,000,000 in stock, and the following contingent consideration: $1,000,000 after year 1, $1,000,000 after year 2, and $500,000 after year 3, if earnings of the subsidiary exceed $10,000,000 in each of the three years. The fair value of the contingent-based consideration portion is $2,100,000. What is the total consideration transferred for this business combination?

A
$15,500,000
B
$15,100,000
C
$13,000,000
D
$5,100,000
A

Explanation:
The correct answer is (B).

In a business combination, consideration transferred is valued at fair market value. When contingent consideration is transferred by the acquirer to the acquiree, it is also valued at fair market value. The total consideration transferred for the business combination = $3,000,000 (cash) + $10,000,000 (stock) + $2,100,000 (fair value of contingent-based consideration) = $15,100,000.

341
Q

Which of the following phrases best describes a Level 1 input for measuring the fair value of an asset or liability?
A
Inputs for the asset or liability based on the reporting entity’s internal data.
B
Quoted prices for similar assets or liabilities in active markets.
C
Inputs that are principally derived from or corroborated by observable market data.
D
Unadjusted quoted prices for identical assets or liabilities in active markets.

A

Explanation:
Level 1 inputs for measuring the fair value of an asset or liability are quoted prices (unadjusted) in active markets for identical assets or liabilities that can be accessed on the measurement date by the reporting entity. Level 2 Inputs are inputs other than quoted prices included within Level 1 that are observable for the asset or liability, either directly or indirectly. Level 2 inputs include quoted prices for similar (not identical) assets or liabilities in active markets. Level 3 inputs are unobservable inputs for the asset or liability.

342
Q

If an entity has a change in accounting estimate during an accounting period, it should report the change in estimate by doing which of the following?

A
Accounting for it as a change in accounting principle, when, within the same situation, the effects of a change in an accounting estimate and a change in an accounting principle cannot be separated.
B
Accounting for it only in the period of change, and not in subsequent periods, as a component of income from continuing operations.
C
Disclosing the effects of the change on income from continuing operations, net income and related per share amounts in the period of the change or in future periods, if the change also affects those future periods.
D
All of the above.

A

Explanation:
The correct answer is (C).

The effects of the change in estimate on income from continuing operations, net income, and related per share amounts should be disclosed in the period of the change or in future periods if the change affects those periods.

Option (A), (B) and (D) are incorrect as per above explanation.

343
Q

East Company leased a new machine from North Company on May 1 under a lease with the following information:

Lease term 10 years
Lease payment due at the beginning of each year $40,000
Estimated life of machine 12 years
Implicit interest rate 14%
Present value of an annuity due of $1 for 10 periods at 14% 5.95
Present value of $1 for 10 periods at 14% 0.27
East has the option to purchase the machine at the end of the lease by paying $50,000, which approximates the expected fair value of the machine on the option exercise date. On May 1, East should record a finance lease asset of

A
$251,500
B
$238,000
C
$224,500
D
$198,000
A

Explanation:
The correct answer is (B).

A Finance Lease must meet one of the following criteria:

Present Value equals or exceeds substantially all (90%) of the Fair Value
Option to Purchase (exercise is reasonably certain)
Economic Life - Major part (75%) of asset’s economic life is used
Transfer of Ownership at lease termination
Specialized Nature - No alternative use to the lessor at lease termination
Note: the implementation guidance for ASC 842 uses the 75/90 thresholds, even though the standard is principles-based.

The lease qualifies as a finance lease since the lease term is for the majority of the useful life of the property.

East will record the finance lease by recognizing an asset and a liability at the inception of the lease.

At commencement, the initial measurement of the asset (regardless of lease classification) is calculated as the lease liability, increased by any initial direct costs and prepaid lease payments, reduced by any lease incentives received before commencement.

Since there are no initial direct costs, prepaid lease payments or lease incentives, the lease asset would be equal to the lease liability which is the present value of minimum lease payments calculated as follows:

= Annual Rental x Present value of an annuity due of $1 for 10 periods at 14%

= $40,000 x 5.95 = $238,000.

Option (A), (C) and (D) are incorrect as per above explanation.

344
Q
Ace Co. settled litigation on February 1 of the current year for an event that occurred during the previous year. An estimated liability was determined as of December 31 of the previous year. This estimate was significantly less than the final settlement. The transaction is considered to be material. The financial statements for the previous year-end have not been issued. How should the settlement be reported in Ace's previous year-end financial statements?
A
Disclosure only of the settlement
B
Only an accrual of the settlement
C
Neither a disclosure nor an accrual
D
Both a dislosure and an accrual
A

Explanation:
An estimated loss from a loss contingency shall be accrued by a charge to income if both of the following conditions are met: (1) it is probable that an asset has been impaired or a liability has been incurred, and (2) the amount of the loss can be reasonably estimated. When the amount of loss is known, as with a settlement after the balance sheet date but before issuance of the financial statements, that amount should be accrued and disclosed in the financial statements.

345
Q
A balance arising from the translation or remeasurement of a subsidiary's foreign currency financial statements is reported in the consolidated income statement when the subsidiary's functional currency is the:
Foreign currency	U.S. dollar
A	No	No
B	No	Yes
C	Yes	No
D	Yes	Yes
A

Explanation:
The functional currency of a foreign entity may be its local currency (i.e., the foreign currency) or the reporting currency (e.g., the U.S. dollar). If the functional currency is the foreign currency, the foreign currency financial statements must be translated into U.S. dollars. The translation adjustments which result from this process are not reported in the consolidated income statement but are reported in other comprehensive income. If the functional currency is the U.S. dollar, the foreign currency financial statements must be remeasured into U.S. dollars. The foreign exchange gains/losses which result from this process are reported in the consolidated income statement.

346
Q
On September 22 of the previous year, Yumi Corp. purchased merchandise from an unaffiliated foreign company for 10,000 units of the foreign company's local currency. On that date, the spot rate was $.55. Yumi paid the bill in full on March 20 of the current year, when the spot rate was $.65. The spot rate was $.70 on December 31 of the previous year. What amount should Yumi report as a foreign currency transaction loss in its income statement for the previous year ended December 31?
A
$0
B
$ 500
C
$1,000
D
$1,500
A

Explanation:
Whenever a transaction is denominated (i.e., payable) in a foreign currency, changes in the translation rate of the foreign currency with respect to the entity’s functional currency (i.e., the dollar in this case) will result in a transaction gain or loss. The gain or loss should be included in the determination of net income in the period(s) the rate changes, and the related asset or liability (i.e., accounts payable in this case) should be adjusted accordingly.

Accounts payable, 12/31 of previous year, in $US
(10,000 local currency units × $0.70) $7,000
Initial obligation, 9/22 or previous year, in $US
(10,000 local currency units × $0.55) (5,500)
Foreign exchange loss recognized in the previous year $1,500

347
Q

On July 1 of the current year, Glen Corp. leased a new machine from Ryan Corp. The lease contains the following information:

Lease term 10 years
Useful life of the machine 12 years
Present value of the minimum lease payments $120,000
Fair value of the machine $200,000
Executory costs $ 3,000
No purchase option is provided, and the machine reverts to Ryan when the lease expires. What amount should Glen record as a leased asset at the inception of the lease?

A
$0
B
$120,000
C
$123,000
D
$200,000
A

Explanation:
The correct answer is (B).

A Finance Lease must meet one of the following criteria:

Present Value equals or exceeds substantially all (90%) of the Fair Value
Option to Purchase (exercise is reasonably certain)
Economic Life - Major part (75%) of asset’s economic life is used
Transfer of Ownership at lease termination
Specialized Nature - No alternative use to the lessor at lease termination
Note: the implementation guidance for ASC 842 uses the 75/90 thresholds, even though the standard is principles-based.

If none of the above criteria is met, the lease is an operating lease for the lessee.

The lease qualifies as a finance lease since the lease term is for the majority of the useful life of the property. Glen will record the finance lease by recognizing an asset and a liability at the inception of the lease. Executory costs are recurring expenses, which if incurred by the lessee, is expensed as incurred. From the annual payments, $3,000 annual executory costs are expensed and the present value of minimum lease payments of $120,000 will be capitalized as a leased asset at the inception of the lease

Options (A) is incorrect because this is a lease must be capitalized unless it is an operating lease for a period of less than 1 year.

Option (C) is incorrect because executory costs are recurring expenses which are expensed when incurred and not capitalized.

Option (D) is incorrect because the asset is recorded at lesser of present value of minimum lease payments or fair market value. Since the present value of minimum lease payments is lower, it is recorded at $120,000

348
Q

Which of the following are not one of the criteria for a sale-leaseback transaction?

A
A contract exists
B
The seller-less satisfies a performance obligation by transferring control to the buyer-lessor
C
Leaseback is a finance lease
D
Leaseback is an operating lease
A

Explanation:
The correct answer is (C). The Leaseback must be an operating lease, not a finance lease.

Per ASC-842, transactions must meet the below criteria to qualify as a Sale-Leaseback transaction.

A contract exists that comply with the provisions of ASC Topic 606: “Revenues from Contracts with Customers.”
The seller-lessee satisfies its performance obligation by transferring control of assets to the buyer-lessor.
ASC Topic 606 identifies five indicators that a customer (in this case a buyer-lessor) has obtained control of an asset:
Customer has legal title
Customer has physical possession
Customer has the significant risks and rewards of ownership
Customer has accepted the asset
Seller has a present right to payment.
4. Leaseback must be an operating lease.

  1. An option for the seller-lessee to repurchase the asset would preclude accounting for the transfer of the asset as a sale of the asset unless both of the following criteria are met:

The exercise price of the option is the fair value of the asset at the time the option is exercised.
There are alternative assets, substantially the same as the transferred asset, readily available in the marketplace.

349
Q

On January 1, year 1, Frost Co. entered into a two-year lease agreement with Ananz Co. to lease 10 new computers. The lease term begins on January 1, year 1 and ends on December 31, year 2. The lease agreement requires Frost to pay Ananz two annual lease payments of $8,000. The present value of the minimum lease payments is $13,950. Which of the following circumstances would require Frost to classify and account for the arrangement as a finance lease?

A
The economic life of the computers is three years.
B
The fair value of the computers on January 1, year 1 is $14,000
C
Frost Co. does not have the option of purchasing the computers at the end of the lease term.
D
Ownership of the computers remains with Ananz Co. throughout the lease term and after the lease ends.

A

Explanation:
The correct answer is (B).

A Finance Lease must meet one of the following criteria:

Present Value equals or exceeds substantially all (90%) of the Fair Value
Option to Purchase (exercise is reasonably certain)
Economic Life - Major part (75%) of asset’s economic life is used
Transfer of Ownership at lease termination
Specialized Nature - No alternative use to the lessor at lease termination
Note: the implementation guidance for ASC 842 uses the 75/90 thresholds, even though the standard is principles-based.

If none of the owner criteria is met, the lease is an operating lease for the lessee

The lease qualifies as a finance lease since the PV of the sum of the lease payments and any residual value guaranteed by the lessee equals or exceeds substantially all of the FV of the underlying asset.

Here, PV of Minimum Lease Payments ($13,950) equals substantially all of the FV of the underlying asset ($14,000)

Option (A), (C) and (D) are incorrect as per above explanation.

350
Q
Lind Corp. was a development stage enterprise from its inception on October 10, year 1 to December 31, year 2. The following were among Lind's expenditures for this period:
Leasehold improvements, equipment, and furniture	$1,200,000
Research and development	850,000
Laboratory operations	175,000
General and administrative	275,000The year ended December 31, year 3 was the first year in which Lind was an established operating enterprise. For the period ended December 31, year 2, what total amount of expenditures should Lind have capitalized?
A
$2,500,000
B
$2,225,000
C
$2,050,000
D
$1,200,000
A

Explanation:
Generally accepted accounting principles determine whether a cost incurred by a development stage enterprise should be charged to expense when incurred or should be capitalized or deferred. Therefore, only the $1,200,000 cost of the leasehold improvements, equipment, and furniture should have been capitalized by Lind. The research and development costs, laboratory operations costs, and the general and administrative costs should have been expensed when incurred.

351
Q
Which of the following is not a valuation technique approach used to measure fair value?
A
Market approach
B
Income approach
C
Cost approach
D
Sales approach
A

Explanation:
The sales approach is not a valuation technique used to measure fair value. Valuation techniques consistent with the market approach, income approach, and/or cost approach shall be used to measure fair value. The technique(s) used shall be the best appropriate in the circumstance and for which sufficient data is available.

352
Q
On April 1, Dart Co. paid $620,000 for all the issued and outstanding common stock of Wall Corp. in a transaction properly accounted for as a business combination. The recorded assets and liabilities of Wall Corp. on April 1 follow:
Cash	$ 60,000
Inventory	180,000
Property and equipment (net of accumulated depreciation of $220,000)	320,000
Liabilities	 (120,000)
Net assets	$440,000On April 1, Wall's inventory had a fair value of $150,000, and the property and equipment (net) had a fair value of $380,000. What is the amount of goodwill resulting from the business combination?
A
$150,000
B
$120,000
C
$ 50,000
D
$ 20,000
A

Explanation:
Goodwill is the excess of the investment cost, plus the fair value of any noncontrolling interest, over the fair value of the identifiable net assets acquired. The amount of goodwill resulting from this acquisition method business combination is determined as follows:

Purchase price $620,000
Cash $ 60,000
Inventory 150,000
Property, plant, and equipment (net) 380,000
Fair value of identifiable assets 590,000
Less liabilities assumed 120,000
Less fair value of identifiable net assets acquired: (470,000)
Goodwill resulting from business combination $150,000

353
Q

Wagner, a holder of a $1,000,000 Palmer, Inc. bond, collected the interest due on March 31 of the current year, and then sold the bond to Seal, Inc. for $975,000. On that date, Palmer, a 75% owner of Seal, had a $1,075,000 carrying amount for this bond. What was the effect of Seal’s purchase of Palmer’s bond on the retained earnings and noncontrolling interest amounts reported in Palmer’s March 31 consolidated bal­ance sheet?

Retained earnings	Noncontrolling interest
A	$100,000 increase	$ 0
B	$ 75,000 increase	$ 25,000 increase
C	$ 0	$ 25,000 increase
D	$ 0	$100,000 increase
A

Explanation:
An investment by one member of a consolidated group of companies in the bonds of another member of that group is, in substance, the same thing as the purchase by a member of its own bonds. Although the bonds cannot physically be retired, since two separate entities are involved in the transaction, from a consolidated viewpoint, the transaction is treated as a constructive retirement of the bonds to the extent of the investment in the bonds. Thus, the consolidated financial statements will reflect any gain or loss on the retirement of bonds in the year of purchase. This is true despite the fact that the books of the affiliates involved in the transaction continue to reflect the Investment in Bonds and Bond Payable accounts, respectively. Thus, the consolidated entity in question recognizes a gain of $100,000 (i.e., $1,075,000 carrying amount of bond - $975,000 cost to subsidiary) on the bond retirement. Gains and losses on the early retirement of bonds can only be reflected on the books of the issuer. Thus, Palmer, the parent company, is attributed the entire $100,000 gain on the retirement, thereby increasing consolidated Retained Earnings by the same amount. Since no portion of the gain on retirement is attributed to the subsidiary, the amount reported in the consolidated financial statements for the 25% noncontrolling interest in the subsidiary is unaffected by the intercompany bond transaction.

354
Q

On December 31 of the previous year, Adam Co. leased a machine under Finance Lease for a period of ten years, contracting to pay $50,000 on signing the lease and $50,000 annually on December 31 of the next nine years. The present value of the ten lease payments over the lease term discounted at 10% was $338,000 at December 31 of the previous year. At December 31 of the current year, Adam’s total Finance Lease liability is

A
$303,980
B
$266,800
C
$259,200
D
$243,000
A

Explanation:
The balance of the Finance Lease liability at 12/31 of the current year is computed as follows:

Balance before payment, 12/31 previous year $338,000
Minimum lease payment, 12/31 previous year 50,000
Balance after payment, 12/31 previous year 288,000
Principal reduction from 12/31 current year payment:
Lease payment, 12/31 of the current year $50,000
Interest portion ($288,000 × 10%) 28,800 21,200
Balance after payment, 12/31 of the current year $266,800

355
Q

The correction of an error in the financial statements of a prior period should be reported
A
Net of applicable income taxes, in the current retained earnings statement after net income but before dividends.
B
As a prior period adjustment by restating the prior-period financial statements
C
Net of applicable income taxes, in the current income statement after income from continuing operations and before extraordinary items.
D
Net of applicable income taxes, in the current income statement after income from continuing operations and after extraordinary items.

A

Explanation:
The correction of an error in financial statements of a prior period is reported as a prior-period adjustment by restating the prior-period financial statements.

356
Q

How should the acquirer recognize a bargain purchase in a business acquisition?

A
As negative goodwill in the statement of financial position
B
As goodwill in the statement of financial position
C
As a gain in earnings at the acquisition date
D
As a deferred gain that is amortized into earnings over the estimated future periods benefited

A

Explanation:
A bargain purchase is a business combination in which the fair value of the recognized identifiable net assets acquired exceeds the fair value of the acquirer’s interest in the acquiree plus the recognized amount of any noncontrolling interest in the acquiree. In such cases, the acquirer should reassess whether it has cor­rectly identified all of the assets acquired and all of the liabilities assumed and should recognize any additional assets or liabilities that are noted in that review. The objective of the review is to ensure that the measure­ments appropriately reflect consideration of all available information as of the acquisition date. If the excess remains after reassessment, the acquirer will recognize the resulting gain in earnings on the acquisition date.

Option (A), (B) and (D) are incorrect as per above explanation.

357
Q

Which of the following statements regarding foreign exchange gains and losses is correct?
A
An exchange gain occurs when the exchange rate increases between the date a payable is recorded and the date of cash payment.
B
An exchange gain occurs when the exchange rate increases between the date a receivable is recorded and the date of cash receipt.
C
An exchange loss occurs when the exchange rate decreases between the date a payable is recorded and the date of the cash payment.
D
An exchange loss occurs when the exchange rate increases between the date a receivable is recorded and the date of the cash receipt.

A

Explanation:
Receivable: Rate increase results in a gain (receive more at settlement), rate decrease results in a loss (receive less at settlement). Payable: Rate increase results in a loss (pay more at settlement), rate decrease results in a gain (pay less at settlement).

358
Q

Howe Co. leased equipment to Kew Corp. on January 2, year 1, for an eight-year period expiring December 31, year 8. Equal payments under the lease are $600,000 and are due annually on January 2. The first payment was made on January 2, year 1. The list selling price of the equipment is $3,520,000 and its carrying cost on Howe’s books is $2,800,000. The lease is appropriately accounted for as a sales-type lease. The present value of the lease payments at an imputed interest rate of 12% (Howe’s incremental borrowing rate) is $3,300,000. What amount of profit on the sale should Howe report for the year ended December 31, year 1?

A
$720,000
B
$500,000
C
$ 90,000
D
$0
A

Explanation:
Profit on the sale should be reported for the excess of the present value of the minimum lease payments over the carrying amount of the equipment. The list price of the equipment is irrelevant. The normal sales price would only be used as the fair value if the present value of the lease payments wasn’t given or able to be computed. The fair value of the leased property at the lease inception is the present value of the minimum lease payments at the lessor’s incremental borrowing rate.

The excess of the present value of the minimum lease payments of leased property at the lease inception over its cost or carrying amount is the dealer’s profit from a sales-type lease and recognized fully at the lease inception.

FV of leased property at lease inception $ 3,300,000
Less: Carrying amount of equipment (2,800,000)
Manufacturer’s or dealer’s profit recognized on sale $ 500,000

359
Q

Explanation:
Profit on the sale should be reported for the excess of the present value of the minimum lease payments over the carrying amount of the equipment. The list price of the equipment is irrelevant. The normal sales price would only be used as the fair value if the present value of the lease payments wasn’t given or able to be computed. The fair value of the leased property at the lease inception is the present value of the minimum lease payments at the lessor’s incremental borrowing rate.

The excess of the present value of the minimum lease payments of leased property at the lease inception over its cost or carrying amount is the dealer’s profit from a sales-type lease and recognized fully at the lease inception.

FV of leased property at lease inception $ 3,300,000
Less: Carrying amount of equipment (2,800,000)
Manufacturer’s or dealer’s profit recognized on sale $ 500,000

A

Explanation:
The correct answer is (C).

The notes to the financial statements need to disclose the amount of minimum lease payments due for each of the next five years and the aggregate amount for the period thereafter and the amount totally in aggregate.

The amount for the required period = Amount for 5 years.

West discloses the minimum lease payments (MLP) of $500,000 ($100,000 × 5) pertaining to the next five succeeding fiscal years and the $300,000 [$100,000 × (10 - 2 - 5)] of MLP pertaining to the fiscal years after this 5-year period.

Lease amounts in aggregate is 800,000 (i.e. $80,000 x 8).

360
Q

A foreign subsidiary of a U.S. parent company should measure its assets, liabilities, and operations using

A
The subsidiary's local currency.
B
The subsidiary's functional currency.
C
The U.S. Dollar.
D
The best available spot rate.
A

Explanation:
The correct answer is (B).

Foreign currency transactions are transactions with a foreign entity denominated in a foreign currency. A foreign subsidiary of a US parent company should measure its assets, liabilities, and operations using the subsidiary’s functional currency. For example, a US company with the bulk of its operations in Canada would consider the Canadian dollar its functional currency, even if financial figures on its balance sheet and income statement are expressed in US dollars. For the subsidiary, the US company may use Canada’s currency or the US dollar, whichever it deems to be the functional currency. Generally, a company chooses the currency with the highest number of transactions, as its functional currency.